Sie sind auf Seite 1von 112

Un hombre de 44 años de edad llega a la oficina para el seguimiento después de

una reciente hospitalización. Hace dos semanas, fue al servicio de urgencias con
palpitaciones y fue encontrado para tener fibrilación auricular con respuesta
ventricular rápida. Durante la noche, se convirtió espontáneamente a ritmo sinusal
normal y lo enviaron a casa al día siguiente. El paciente no tiene antecedentes de
hipertensión arterial, diabetes mellitus, hipercolesterolemia o enfermedad cardíaca.
No utiliza tabaco, alcohol o drogas ilícitas. Su presión arterial es 124/70 mm Hg y el
pulso es 78/min y regular. La presión venosa yugular es estimada en 5 cm H2o.
Examen no demuestra ninguna anormalidad. Informe de resultados de laboratorio
de la ingreso en el hospital muestra el nivel normal de creatinina, pruebas de función
hepática, estudios de tiroides y lípidos p un ecocardiograma muestran una leve
dilatación de aurícula izquierda, izquierda normal y función ventricular derecha, y
ninguna anormalidad valvular importante. Cuál de los siguientes es el mejor
tratamiento adicional para ¿Este paciente?

A. Amiodarona 12 %
B. aspirina y clopidogrel 10 %
C. aspirina y warfarina 19 %
D. Ninguna terapia adicional 59 %
E. rivaroxabán 7 %

Este paciente tenía paroxística auricular fibrilación auricular (FA) que


espontáneamente al tratamiento sinusal normal con tromboembolismo es un
importante causa de morbilidad a largo plazo en pacientes con FA y reducir los
anticoagulantes orales warfarina se ha demostrado que significativamente
directrices riesgo de embolización en pacientes con moderada a alta riesgo de
eventos tromboembólicos. Reciente en pacientes han recomendado el uso de un
Asc C modificado puntuación para la evaluación de riesgo de accidente
cerebrovascular con n el término "lone AF" se utiliza ocasionalmente en pacientes
con paroxística, persistente o permanente AF con evidencia de enfermedad
cardíaca estructural. Pacientes con FA solitaria son generalmente mayores de 60 y,
por definición, tienen un ents es extremadamente puntuación de 0. El riesgo de
embolización sistémica en tal (opción A) no está indicada la terapia anticoagulante y
baja. Amiodarona.

Un hombre de 40 años llega al servicio de urgencias debido a dolor de cabeza


severo asociado a fotofobia y náuseas durante las últimas horas varias. El paciente
informa de los síntomas del resfriado leve a la semana hace. Él tiene una historia de
2 años de hipertensión y ha fumado un paquete de cigarrillos diario por 15 años. mm
Hg y el pulso es 92/min y regular. Le parece incómodo y tuvo un episodio de
náuseas y vómitos en el servicio de urgencias. El examen neurológico muestra
fuerza motor 4/5 y 2 reflejos profundos del tendón bilateral en las extremidades
superiores e inferiores. Sensibilidad a la luz pinchazo de tacto es normal.
Tomografía computarizada de la cabeza sin contraste se muestra a continuación. de
las siguientes es ¿causa más probable del dolor de cabeza de este paciente?

A. meningitis aséptica angiopathy amiloideo de


B. La trombosis venosa Cerebral
C. Cerebral malformación arteriovenosa
D. Encefalopatía Hipertensiva
E. Aneurisma rota
F. disección de la arteria vertebral

Una mujer caucásica de 35 años de edad se presenta a la oficina debido a que se


palpa una masa en mama; Después de la se realiza la adecuada aspiración el
Biopsia , la fino-aguja (FNA) de la masa. Los resultados de la FNA volver como
negativa. Como se explica el resultado de la prueba, pide a la paciente, cuáles son
las posibilidades que realmente no tengo cáncer de mama?" Cuál de los siguientes
valores mejor aborda este paciente ¿de la pregunta?

A. sensibilidad (10%)
B. especificidad [11%]
C. positivo valor predictivo
D. Valor predictivo negativo del validez

Respondió correctamente esta explicación: el valor predictivo negativo (VPN) se


define como la probabilidad de estar libre de una enfermedad si el test es negativo.
Una cosa muy importante a recordar es que el VPN puede variar con la probabilidad
pretest de un de la enfermedad. A pacientes con una probabilidad de tener una
enfermedad tiene un bajo valor actual neto y un paciente de alto abajo: con una
probabilidad baja de tener una enfermedad tendrá un alto valor actual neto. Se dan
ejemplos concretos 1) cáncer de mama y FNA prueba resultados A paciente con
alta probabilidad pre-test para tener mama cáncer (por ejemplo, la mujer, tiene
primer grado familiares cáncer de mama, más de 40 años de edad) tiene un bajo
NPV. Un paciente con un pre-test baja probabilidad de tener cáncer de mama (por
ejemplo, menos de 40 años de edad, como en este caso) tiene un alto valor actual
neto. 2) VIH y ELISA prueba de resultados a un paciente que pertenece a un grupo
de alto riesgo (por ejemplo, múltiples parejas sexuales, admite a no usar NPv.
usuario) tiene pre-test probabilidad; Este paciente tendrá baja tiene en la mano, un
paciente que pertenece a un bajo riesgo Grupo (por ejemplo, uno sexual que no
asociarnos a otras parejas sexuales, utiliza condones correctamente todos los
tiempo, sin historia de uso de drogas Intravenosas) tiene baja probabilidad pre-test;
por lo tanto, este paciente tendrá un alto valor actual neto.

Un hombre hispano de 30 años de edad se presenta a la oficina con quejas de


palpitaciones, temblor, nerviosismo y dolor de cabeza. Su última historia es
insignificante. Su madre tiene diabetes tipo 2, que está bien controlada con
medicamentos. Su temperatura es de 37,0 ° C (98,6 ° F), el pulso es de 100/min,
presión es de 150/80 mm Hg, y respiraciones son 16/min. Él aparece ansioso,
sudoroso y tembloroso. La examinación neurológica es no focal, y el examen de
otros sistemas es normal. Su nivel de glucosa en sangre obtenida por punción digital
es de 38 mg/dL. Administración intravenosa de un bolo de dextrosa 50% conduce a
la mejora de sus síntomas. Después es sometida a ayuno prolongado bajo
supervisión. Después de un ayuno nocturno, estudios de laboratorio revelan:
glucemia 40 mg/dL. Insulina de suero 15 microU/L (valor normal es k 6 microU/L con
hipoglucemia) suero Pro-insulina 9 microUIL al valor k 20% del total immunoreactive
de la insulina) nivel de péptido c 0.8 nmol/L (valor normal es menos de 0.2 nmol/L)
sulfonilureas negativo IGF-II negativo basado en la información anterior, cuál es el
más ¿causa probable de la hipoglucemia de este paciente?
A Tumor de la célula Beta
B. Non-beta de la célula tumoral
C. sulfonilurea agentes
D. exógena insulina
E. Glucagonoma

Es normal personas, un nivel de glucosa en sangre por debajo de 60 mg/dL produce


casi completa de la secreción de insulina. El paciente en esta viñeta así presenta
con hipoglucemia e inapropiadamente elevado suero niveles de insulina. Hay dos
importantes causas de hipoglucemia en pacientes no diabéticos con niveles de
insulina elevados.

 Mujer de 52 años acude al servicio de urgencias debido a una semana de fiebre,


disnea y tos con esputo que productiva, huele asqueroso. Ella no tiene escalofríos o
rigores. El paciente era atención de urgencia Centro para estos síntomas hace 4
días y fue prescrito un curso de 5 días de azitromizina para a tratar neumonía. Sin
embargo, ella sigue peor a pesar de tomar la medicina. Ella experimentó una
endoscopia gastrointestinal alta 10 días atrás por una larga historia de pirosis
temperatura es 38,7 C (101.7 F), la presión arterial es 130/80 mm Hg, pulso es 108
por minuto y respiraciones 22/min dentición es pobre. Se oyen crujidos sobre el
campo pulmonar superior derecho. No hay soplos cardiacos. Abdomen es suave
Hemograma muestra leucocitos de 14.500/mm3 radiografía de tórax revela un
derecho infiltrado del lóbulo superior. Cuál de las siguientes terapias adicionales
sería más útil para esto ¿condición del paciente?

A. Ampicilina y gentamicina
B. Ciprofloxacina
C. Clindamicina
D. Doxycycline
E. Trimetoprima-sulfametoxazol

Este paciente probablemente tiene una neumonía por aspiración a anaerobios como
complicación de la endoscopia gastrointestinal superior. Pacientes con neumonía
causada por organismos anaerobios por lo general presentan síntomas indolentes
como fiebre, malestar, un característico mal olor esputo pobres que predispone a
una mayor cantidad de organismos de flora oral, aumenta el riesgo de neumonía
anaeróbica. Otros factores de riesgo para aspiración de incluir formas de
instrumentación de la vía aérea superior (por ejemplo, endotraqueal o tubos
nasogástricos), y con problemas de reflujo de conciencia, disfagia, la localización de
la infiltración depende de la gravedad y difiere según la posición del paciente en el
momento de la aspiración. En decúbito supino el segmentos posteriores de los
lóbulos superiores y segmentos superiores de los lóbulos inferiores son los más
afectados, mientras que en las bases de los lóbulos inferiores y el lóbulo medio
derecho de erectas pacientes son los más afectados. Añadir un antibiótico como
clindamicina, cobertura anaeróbica es el siguiente paso más razonable. Otros
regímenes incluyen metronidazol con amoxicilina.

 Pregunta 2 de 2 en el final del periodo de seguimiento, los investigadores


informaron que el 17% de los pacientes en el Grupo de terapia médica y 91% de los
pacientes en la terapia médica más grupo de CABG experimentaron CABG. La
combinación de ambos grupos y luego analizar los resultados para los pacientes
que realmente experimentaron CABG frente a ¿aquellos que recibieron sólo
tratamiento médico podrían establecer cuál de las siguientes?

A. Plazo de obtención de sesgo de medición


B. Observadores diagonal
D. recordar diagonal
E. Selección sesgo:

El estudio presentado en el resumen es una clínica estudio clínico que comparó el


beneficio de la CABG con tratamiento farmacológico versus tratamiento médico
solo. A pesar de tratamiento médico de alta calidad a todos los pacientes en el
estudio, 17% de los pacientes en el médico Grupo de tratamiento clínico deteriorado
hasta el punto donde CABG se indicó uncontrovertibly. Cuando el régimen de
tratamiento para un paciente depende de la severidad de la condición del paciente,
puede resultar una forma de sesgo de selección conocido como sesgo de
susceptibilidad (factores de confusión por indicación). Esto se produce porque los
pacientes de cirugía del grupo de tratamiento médico pueden tener subyacente
factores de confusión que fueron responsables del empeoramiento de su
enfermedad agruparlas con pacientes de el grupo de cirugía puede conducir a
análisis erróneos y debilitan las conclusiones que pueden extraerse de el estudio
para evitar la selección b en estudios, los pacientes son asignados aleatoriamente a
los tratamientos para minimizar potenciales variables confusoras muchos estudios
también realizan un sesgo electoral. Un análisis ITT compara el análisis (ITT) para
hacer frente a la intención) sin importar la inicial al azar grupos de tratamiento (la
original tratado del tratamiento eventual para evitar contar pacientes crossover. Por
el contrario, identificación como prueba: análisis (vástago en cuestión sugerida)
compara los grupos basados en el real tratamiento recibido. Un análisis como
tratamiento se realiza para medir la efectividad del tratamiento.

 Una mujer de 36 años llega a la oficina después de un mes de dolores de cabeza


casi a la manera de una banda, mareos y fatiga. Ella ha experimentado irritabilidad,
incapacidad para concentrarse y dormir mal de la misma duración. Antes del inicio,
tuvo un accidente en el trabajo; una caja cayó sobre su cabeza, y ella perdió el
conocimiento brevemente. El paciente fue llevado a un departamento de
emergencia y todas las pruebas fueron normal, entonces ella fue dada de alta. Ella
no ha tenido pesadillas ni flashbacks del evento. Ella no tiene problemas médicos
conocidos. Ella bebe 1-2 vasos de vino todas las noches pero no usa tabaco o
drogas ilícitas. El paciente está afebril. La presión arterial es 125/85 mm Hg y el
pulso es 76 / min. Los exámenes cardiopulmonares y abdominales no son
notables. Ella parece ansiosa pero su neurológica la evaluación que incluye
atención y memoria es normal. ¿Cuál de los siguientes es el más probable
¿diagnóstico?

A. Trastorno de estrés agudo


B. Hematoma subdural retardado
C. Trastorno facticio
D. Síndrome de Korsakoff
E. Síndrome posconmocional
F. Trastorno de estrés postraumático
G. Trastorno de síntomas somáticos

Explicación: Este paciente muestra signos de síndrome postconcusivo después de


una lesión cerebral traumática leve TBI) TBI de cualquier gravedad puede conducir
a (unas pocas horas o días después) el síndrome postconcussi, que está
clínicamente descrito por el siguiente constelación de síntomas: dolor de cabeza,
confusión, amnesia, dificultad estos con multitarea, vértigo, alteración del estado de
ánimo, alteración del sueño y ansiedad. Por lo general, los síntomas se resuelven
con tratamiento sintomático en unas semanas o meses después de una TBI; Sin
embargo, algunos pacientes pueden tienen síntomas persistentes que duran 26
meses.

 Un hombre de 62 años acude a la oficina con un historial de 4 semanas de


picazón y sequedad en las manos. Él tiene tiene una piel escamosa y agrietada
experimentada en el dorso de ambas manos que se extiende hasta los dedos. El
paciente ha probado lociones humectantes con cierto alivio, pero los síntomas son
persistentes. El historial médico es notable por alergias estacionales, hipertensión,
hiperplasia prostática benigna y una erupción por herpes en el labio superior
después de infecciones respiratorias superiores. Él trabaja como dentista y lo hace
no use tabaco, alcohol o drogas ilícitas. El examen muestra piel seca, costrosa e
irritada con eritema en los dedos, las redes de los dedos y el dorso de las manos,
como se muestra en la imagen. Unos pocos las vesículas se observan en las redes
de los dedos. No se ven otras lesiones en la piel. ¿Cuál de los siguientes es el
diagnóstico más probable?

A. Dermatitis de contacto
B. Herpetic whitlow
C. Psoriasis
D. sarna
E. tiña manum

 Una mujer de 34 años acude al médico que se queja de diarrea, pérdida de peso
y fatiga durante el año pasado. La diarrea ocurre 2 o 3 veces al día y se acompaña
de dolor abdominal tipo cólico. Ella tiene diarrea de vez en cuando por la noche. Ella
no tiene tenesmo o sangre en las heces. El paciente describe sus evacuaciones
como muy malolientes y flotantes. Ella también se queja de dolor óseo difuso. El
examen físico no tiene nada especial. Los resultados de laboratorio muestran una
hemoglobina de 9.8 g / dL con una media volumen corpuscular de 72 fL. ¿Cuál de
los siguientes hallazgos adicionales se esperaría en este ¿paciente?

Hormona paratiroidea alta de fosfato baja de calcio baja (59%)


Baja Baja Baja [1%]
Baja Alta Baja [5%]
Bajo Alto Alto [14%]
Alto Bajo Alto [19%]
Explicación: La presentación de este paciente de un año de diarrea maloliente,
pérdida de peso y fatiga es compatible con probable esteatorrea y malabsorción. La
esteatorrea previene la emulsificación grasa habitual y altera la vitamina absorción
de vitamina D en el intestino, lo que resulta en deficiencia de vitamina D. D media
intestinal absorción de calcio y fósforo, los pacientes desarrollan hipocalcemia y
disminución de los niveles de fósforo. La hipocalcemia conduce a niveles
secundarios con niveles elevados de hormona paratiroidea. Vitamina D la
deficiencia puede conducir a anormalidades, hallazgos que incluyen dolor o
sensibilidad en los huesos, músculo debilidad o calambres, marcha, educación y
aumento del riesgo de fracturas. Sin embargo, algunos pacientes pueden ser
asintomático.

Una mujer de 23 años es llevada al servicio de urgencias debido a una hora de


dificultad para respirar. Ella tiene un historial de asma y ha sido hospitalizada varias
veces en el último año con asma exacerbación. El paciente no cumple con un
corticosteroide inhalado pero toma albuterol como necesario para la falta de aliento
Ella dice que la dificultad para respirar se siente como su asma ataque. Ella acaba
de ser diagnosticada con sinusitis aguda después de una infección de las vías
respiratorias superiores por que tomó un antibiótico esta mañana. La paciente ha
usado su inhalador de albuterol varias veces desde el inicio de los síntomas. Ella no
usa tabaco, alcohol o drogas ilícitas. Ella es alérgico al polen pero no tiene alergias
medicamentosas conocidas. En el examen, el paciente está en dificultad respiratoria
moderada. La temperatura es 37.8 c 100 F), la presión arterial es 82/50 mm Hg, el
pulso es 118 / min, y las respiraciones son 30 / min. No hay distensión venosa
yugular visible. Tórax revela sibilancias audibles de forma bilateral y expiración
prolongada. El es suave y no sensible. Una urticaria erupción se ve sobre el tronco y
las extremidades superiores, y las extremidades son cálidas para el toque. de lo
siguiente ¿Cuál es el siguiente paso más apropiado en el manejo de este paciente?
A. Albuterol inhalado continuo
B. Cricotomía
C. Intubación endotraqueal
D. Fluticasona inhalada
E. Epinefrina intramuscular 61%
l Antibióticos por vía intravenosa
G. Difenhidramina intravenosa
H. Epinefrina intravenosa
I. Metilprednisolona intravenosa

Una mujer de 49 años llega a la oficina debido a la dificultad para tragar alimentos
sólidos. Los síntomas empeoran con los panes y las galletas, pero no tiene dificultad
con los líquidos, y ella usa sorbos de agua para ayudar a tragar. Ella piensa que
puede haber perdido peso. Los pacientes el historial médico es significativo para la
hipertensión y el hipotiroidismo tratados con amlodipina y levotiroxina,
respectivamente. Ella también usa gotas de solución salina sin receta para la
sequedad ocular. Paciente nunca ha usado tabaco o alcohol. Los antecedentes
familiares son negativos para el cáncer. Ella ha estado monógamo con su esposo
durante 15 años. La presión arterial es 140/90 mm Hg y el pulso es 60 / min. BMI es
19 kg / m2. El examen físico muestra aftas orales, caries dentales prominentes y
bilateral, firme nódulos submandibulares. No hay linfadenopatía. El examen
cardiopulmonar no muestra anormalidades. El abdomen es suave y no sensible, sin
organomegalia. No hay edema periférico ¿Cuál de los siguientes es el mejor paso
para confirmar el diagnóstico de este paciente?
A. Anticuerpos a RolsSA y Lal ssB
B. Tomografía computarizada del cuello
C. Evaluación del trastorno alimentario
D. Endoscopia esofágica con biopsia
E. biopsia de glándula submandibular
F. videofluoroscópica estudio de deglución

Un hombre de 26 años sometido a quimioterapia para seminoma metastásico


llega al servicio de urgencias con fiebre y escalofríos durante las últimas 24 horas.
No tiene dolor en el pecho, dificultad para respirar, tos, dolor abdominal, diarrea o
erupción cutánea. Su temperatura es de 38.9 C (102 F), la presión arterial es de
118/70 mm Hg, el pulso es 102 / min, y las respiraciones son 18 / min. El examen
muestra palidez de la mucosa. Ahi esta eritema gingival leve pero sin evidencia de
sangrado. El examen cardiopulmonar es normal. los el abdomen es suave y no
sensible a la palpación. El área perianal no tiene lesiones. Laboratorio los resultados
son los siguientes: 690 / uL con 20% de leucocitos neutrófilos Hemoglobina 8.6 g /
dL Hematocrito 25% Plaquetas 74,000 / uL Creatinina 0.9 mg / dL Radiografía de
tórax no muestra infiltrados. El análisis de orina es normal. El paciente ingresa en el
hospital. ¿Cuál de los siguientes es el inicial más apropiado? terapia para este
paciente?

A. Aciclovir
B. Ceftriaxona
C. Ciprofloxacina
D. Linezolid
E. Piperacilina-tazobactam [42%]
F. Vancomicina
G. Voriconazol

Una mujer de 45 años acude al servicio de urgencias con vómitos y dolor severo
en el flanco derecho que irradia a su ingle. El paciente ha estado en el
departamento de emergencia dos veces en el pasado debido a episodios similares
de cólico renal. Las imágenes previas mostraron cálculos radiopacos. Las piedras
pasaron espontáneamente, y no buscó atención médica adicional Las imágenes
ahora muestran otra piedra ureteral. Una recolección de orina de 24 horas muestra
la excreción urinaria de calcio de 350 mg (normal es y lt; 250 mg en mujer). Los
resultados de laboratorio muestran una concentración sérica de calcio de 8.9 mg /
dL; paratiroides en suero la hormona es normal. Las investigaciones posteriores no
revelan una causa de la hipercalciuria. Aparte de aconsejando un mayor consumo
de líquidos, ¿cuál de las siguientes intervenciones beneficiará a este paciente?

A. Antagonista de aldosterona
B. Ingesta alta de sodio
C. Furosemida en dosis baja
D. Restricción de calcio dietario
E. Diuréticos tiazídicos [76%]

Un estudiante universitario de 20 años es llevado al departamento de


emergencias por su novia después de que él ingirió grandes cantidades de
paracetamol en un intento de suicidio. Hace dos días, la novia le dijo a la paciente
que ella quería romper con él. Él se angustió y tomó aproximadamente 30 pastillas
de 500 mg de paracetamol para terminar con su vida. Esta mañana, la novia
descubrió lo que había hecho y lo trajo para buscar atención. El paciente informa
náuseas y dolor abdominal leve. Él no tiene historial de intentos de suicidio previos o
enfermedad psiquiátrica. La temperatura es 36.7 C (98 F), sangre la presión es
114/68 mm Hg, y el pulso es 94 / min. El examen abdominal revela una leve
molestia hepatomegalia. El nivel de paracetamol en suero es alto y el paciente está
hospitalizado por Terapia de N-acetilcisteína. Durante la estadía en el hospital, se
vuelve marcadamente confundido e incoherente. El examen repetido muestra
escleral icterus y asterixis. resultados de laboratorio actuales y aquellos obtenidos
en el momento de la admisión son los siguientes: Al ingreso Estudios actuales de
función hepática Total bilirrubina 1.2 mg / dL 4.1 mg / dL Aspartato aminotransferasa
96 U / L 8.456 U / L Alanina aminotransferasa 70 U / L 9,634 U / L Tiempo de
protrombina 18 seg 120 s Creatinina sérica 1,1 mg / dL 3,5 mg / dL. De los cuales
siguiente es el mejor siguiente paso en el manejo de este paciente?

A. Solo control de cierre


B. Iniciar la suplementación con vitamina K
C. Realizar una biopsia hepática
D. Referir a centro de trasplante de hígado [55%]
E. Iniciar glucocorticoides

Un hombre de 54 años acude a la clínica con fiebre, disnea de esfuerzo y tos no
productiva una semana. Él fue diagnosticado con infección por VIH hace 3 años,
pero ha estado asintomático desde entonces. Él tiene no ha estado tomando sus
medicamentos constantemente. Su temperatura es de 38.9 C (102 F), la presión
arterial es 120/80 mm Hg, el pulso es de 100 / min y las respiraciones son de 28 /
min. La oximetría de pulso del paciente muestra 80% en el aire de la habitación.
Con el uso de una máscara 100% no respirable, su saturación de oxígeno aumenta
al 92%. Los resultados de laboratorio son los siguientes: Conteo sanguíneo
completo Hemoglobina 9.6 g / dL Plaquetas 120,000 / mm3 8,000 / mm3 (sin
bandas de leucocitos Gases arteriales en el aire ambiente pH 7.45 PaO 54 mm Hg
PaCO2 44 mm Hg La radiografía de tórax muestra infiltrados intersticiales bilaterales
difusos. Su recuento de CD4 es de 190 / mm3 y lactato el nivel de deshidrogenasa
es de 400 U / l. ¿Cuál es el siguiente paso más apropiado en la gestión de este
¿paciente?
A. Iniciación del tratamiento antirretroviral
B. Ceftriaxona intravenosa
C. Pentamidina intravenosa y corticosteroides
D. Trimetoprim-sulfametoxazol y corticosteroides [61%]
E. Trimetoprim-sulfametoxazol solo

Una niña de 15 años previamente sana viene a la oficina debido a "manchas" en


su rostro. Como un año Hace poco, la paciente comenzó a tener pequeñas
protuberancias en la frente, que han aumentado progresivamente en número y
tamaño Ella usó varios productos de limpieza facial de venta libre sin ningún alivio.
Ella no usa tabaco, alcohol o drogas ilícitas. El examen físico muestra una tez
grasosa y otros hallazgos. ¿Cuál de los siguientes es el mejor paso siguiente en
manejo de este paciente?
A. Tera antiviral
B. Limpiadores a base de crema
C. Dieta modificación
D. Antibióticos sistémicos
E. Corticosteroide tópico
F. Tópico metronidazol
G. Retinoides tópicos (69%)

Se descubre que un niño de 15 años tiene un soplo durante un examen físico
deportivo de rutina. Él no tiene quejas y tiene una excelente capacidad de
ejercicio. Su tío materno murió mientras nadaba a los 24 años. La presión arterial
del paciente es de 122/70 mm Hg y el pulso es de 63 / min. Un soplo mesosystolic
débil es se escucha a lo largo del borde esternal inferior izquierdo cuando el
paciente está en decúbito supino. Cuando se le pide al paciente que de pie, se
escucha un murmullo de crescendo decrescendo sistólico 3/6 en el mismo lugar. De
los cuales siguiente es la anormalidad de válvula mitral más probable en este
paciente?

A. Prolapso mitral anormal movimiento [56%]


B. Anillo de válvula mitral dilatada
C. Calcificaciones del anillo mitral
D. Prolapso de la valva de la válvula mitral
E. Ruptura de las cuerdas tendinosas

Un hombre de 27 años acude al médico debido a la orina roja. No ha tenido dolor
o ardor en micción. Tiene tuberculosis pulmonar infiltrativa (diagnosticada
recientemente). Él toma isoniazida, rifampina y pirazinamida. Él fuma dos paquetes
al día y consume alcohol de vez en cuando. Vital los signos son estables El
examen físico no muestra anormalidades. ¿Cuál de los siguientes es el más
diagnóstico probable?

A. Cistitis aguda
B. Tuberculosis renal
C. Reacción farmacológica [95%]
D. Nefrolitiasis
E.Glomerulopatía

El diagnóstico más probable en este paciente es la decoloración de la orina


debido a la rifampicina. Es importante recordar que las causas de la rifampicina
decoloración de rojo a naranja de los fluidos corporales, incluida la orina, la saliva,
el sudor y las lágrimas. También puede causar decoloración de las lentes de
contacto blandas. Conocer este hecho lo ayudará a obtener un "fácil"
diagnóstico. Por supuesto, se deben descartar afecciones más serias, comenzando
con un simple análisis de orina. Ausencia de cambios significativos en el análisis de
orina (proteinuria, hematuria y piuria estéril) esencialmente descarta la tuberculosis
renal o la glomerulopatía asociada a la tuberculosis (Elección E). (Opciones A y D)
La hematuria durante la cistitis aguda o la nefrolitiasis suele ir acompañada de dolor
y síntomas disuria. Objetivo educativo: la rifampicina causa decoloración del cuerpo
de rojo a naranja fluidos. La "orina roja" en un paciente que toma rifampina
generalmente es un efecto benigno de la droga.
Un hombre de 53 años llega al médico quejándose de empeoramiento de la
debilidad y disnea del exertional durante los últimos 2 días. Él tenía una enfermedad
respiratoria superior 3 semanas pero los síntomas no resuelto totalmente. No tiene
ningún dolor en el pecho o palpitaciones. Su padre murió de un infarto infarto en la
edad 55. Su presión arterial es 87/60 mm Hg y el pulso es de 109 por minuto,
saturación de oxígeno es 95% en aire ambiente. Su índice de masa corporal es de
23 kg/m2. Pulsación venosa yugular interna es 11 cm H20 Los pulmones están
claros a la auscultación. Son amortiguados sonidos de corazón. Cuál de las
siguientes es la más ¿causa probable de las quejas de este paciente?

A. disminución de contractilidad cardiaca


B. disminución de la precarga [52%]
C. aumento ventricular derecho cumplimiento
D. izquierda ventricular respuesta correcta salida obstrucción
E. hipertensión pulmonar

Este paciente Presentación sugiere derrame pericárdico y taponamiento


cardiaco. La etiología es probablemente infecciosa (pericarditis viral) debido a su
reciente enfermedad viral respiratoria superior. Este paciente tiene todas las 3
características de la tríada de Beck asociados con taponamiento cardiaco:
hipotensión, distensión de cuello las venas y sonidos de corazón ahogado.
Paradoxus del pulsus 10 mm Hg de la gota en la presión arterial sistólica durante el
la inspiración) es también un hallazgo común. El examen abdominal puede mostrar
también hepatoyugular positivo reflujo taponamiento cardiaco es debido a la
acumulación de líquido en la cavidad pericárdica que aumenta la presión
intrapericárdica por encima de la presión ventricular diastólica. Esto restringe el
retorno venoso a el corazón y baja derecha e izquierda de llenado ventricular. El
resultado es disminución de precarga, volumen sistólico y gasto cardiaco. El
examen pulmonar típicamente muestra pulmones claros a auscultación debido a
disminución de llenado (precarga) en lugar de sobrecarga de volumen ventricular.

Una mujer de 44 años de edad se evalúa por debilidad generalizada, fiebre baja y
dolor de las articulaciones. Su las actividades diarias están limitadas debido a la
rigidez articular, especialmente en la mañana. Ella tiene bilateral hinchazón de sus
articulaciones de mano, principalmente las articulaciones metacarpofalángicas. La
punta del bazo es palpable en la inspiración profunda. Su hematocrito es de 34%.
Hígado y pruebas de función renal son normales. El paciente diagnosticada con
artritis reumatoide y comenzó el tratamiento inicial. Dos meses más tarde, han
mejorado sus síntomas comunes, pero ella vuelve al médico con úlceras orales
dolorosas. Resultados de laboratorio son los siguientes: hematocrito 33% aspartato
aminotransferasa 120 U/L alanina aminotransferasa 90 U/L alcalina fosfatasa 90
U/L. bilirrubina 1.1 mg/dL creatinina 0.8 mg/dL. Tarifa de sedimentación de eritrocito
de sangre urea nitrógeno de 16 mg/dL es 22 mm/hr, cuál de las siguientes es
¿causa más probable de estado actual de este paciente?

A. Anticytokine agente terapia 19%]


B. antimetabolito de agente [59%]
C. tratamiento con corticoides
D. Felty el síndrome
E. linfoma
F. Hepatitis vírica.

Una mujer de 31 años de edad llega a la oficina con una historia de 6 semanas de
edema periorbitario y abdominal distensión. Su temperatura es de 37,1 C (98,9 F),
presión arterial 125/75 mm Hg, pulso es 80/min y respiratorio son 14/min examen
muestra moderadas ascitis y edema de la extremidad inferior. Análisis de orina
muestra proteinuria 24 horas y la excreción urinaria de proteínas es de 4 g/día,
proteínas séricas totales es de 5 g/dL, y albúmina sérica es de 2,5 g/dL. El
ultrasonido renal es irrelevante . Se realiza una biopsia renal. El paciente se inicia
en diuréticos y consumo de sal y proteínas está restringido. El edema comienza a
mejorar. Sin embargo, el paciente desarrolla repentinamente bruto, fiebre y dolor
abdominal lado derecho severo hematuria. ¿Cuál de las siguientes es el diagnóstico
más probable es revelado por la biopsia renal?

A. Focal glomerulosclerosis segmentario


B. IgA nefropatía
C. glomerulopathy membranoso [49%]
D. Enfermedad con cambios mínimos
E. amiloidosis sistémica respuesta

Trombosis de la vena Renal (RVT) y otros tromboembolismo son importantes


complicaciones del síndrome nefrótico. Pérdida de antitrombina Mal (un inhibidor de
múltiples factores de la coagulación) en la orina aumenta el riesgo de venosa y
trombosis arterial. Trombosis de la vena renal puede ser aguda y con dolor
abdominal, fiebre y hematuria, como en este paciente. Sin embargo, RVT es más
comúnmente progresiva, causando empeoramiento gradual de la función renal y
proteinuria en un paciente asintomático. RVT puede ocurrir en cualquier etiología de
síndrome nefrótico, pero es más comúnmente visto con glomerulopathy
membranoso. (Opciones A, D y E) Enfermedad, con glomerulosclerosis
segmentario focal, un mínimo cambio y sistémica la amiloidosis pueden causar el
síndrome nefrótico y trombosis de la vena renal posterior; sin embargo, es la RVT
menos frecuente con estos trastornos la nefropatía por IgA (opción B) presenta
generalmente con bruto o hematuria microscópica con proteinuria mínima después
de una infección del tracto respiratorio superior. Pacientes raramente desarrollan
rápidamente progresiva ymptoms de síndrome nefrótico o renal. Este paciente
presentación de síndrome nefrótico y el infarto no es compatible con nefropatía lgA

Una mujer de 32 años de edad llega a la oficina debido a la tos persistente y


dificultad para respirar. Ella ha tuvo 3 episodios de neumonía en los últimos 3 años.
El paciente tenía sinusitis severa hace un año y un episodio de diarrea con sangre
que requirió hospitalización hace 6 meses. Generalmente responde a antibióticos
pero lleva varios días para despejar la infección. No hay otros miembros de la
familia tienen similar problemas. El paciente no ha viajado fuera de Estados Unidos.
Ella no usa tabaco, alcohol o drogas ilícitas y en la actualidad no hay medicamentos.
Todas sus vacunas son fecha. La presión arterial es 130/80 mm Hg y el pulso es de
90/min índice de masa corporal es de 22 kg/m2. Examen físico revela finos
crepitantes en el campo más bajo correcto del pulmón. No hay edema de la
extremidad inferior está presente. Cuello la palpación no mostrar cualquier
ampliación del nodo de linfa. Recuento de leucocitos es 14.000/mm3. La prueba del
VIH es negativo. Radiografía de tórax revela un lóbulo más bajo derecho infiltrarse.
Cuál de los siguientes es más probable ¿para revelar la causa del proceso
subyacente de la enfermedad de este paciente?

A. medición de cD4 TIymphocyte contar


B. medición del suero alfa-1 antitripsina nivel
C. metacolina reto de prueba para la medición cuantitativa de
D. Niveles séricos de inmunoglobulina [73%]
E. sudor prueba del cloruro

Se realizó un estudio para evaluar la asociación entre el uso de L-triptófano y el


desarrollo Síndrome de eosinofilia-mialgia (el ccsme). Pacientes con EMS se les
preguntó sobre el uso de productos que contiene L-triptófano durante los últimos 6
meses. Al mismo tiempo, era gente sin EMS seleccionadas al azar de la misma
población donde vino de los pacientes y le preguntó sobre su experiencia con
productos que contienen L-triptofano en los últimos 6 meses. El estudio mostró que
el uso de L-triptófano se asocia significativamente con EMS. Cuál de las siguientes
medidas de ¿Asociación son los investigadores más probable al informe?
A. la supervivencia mediana
B. riesgo relativo de
C. exposición odds ratio [65%]
D. relativa tarifa
E. prevalencia odds ratio

Este caso describe un diseño de estudio de casos y controles típicos. Pacientes con
la enfermedad de interés (casos) y personas sin la enfermedad (controles) se
preguntó sobre la exposición anterior a la variable ser estudiado (uso de L-
triptófano). La principal medida de asociación es el ratio de exposición, en que la
exposición de las personas con la enfermedad (casos) se compara la exposición de
los que no la enfermedad (controles) medidas de incidencia (opciones A y D) (p. ej.,
riesgo relativo o velocidad relativa) no se puede medir directamente en los estudios
de casos y controles, porque las personas estudiadas son las que ya han
desarrollado la enfermedad. Riesgo relativo y la tasa relativa se calculan en cohorte
estudios, donde las personas se siguen en el tiempo para la ocurrencia de la
enfermedad (opción B) mediana supervivencia se calcula en estudios de cohorte o
ensayos clínicos y se utiliza generalmente para comparar la tiempos de
supervivencia media de dos o más grupos de pacientes (por ejemplo, que recibe un
nuevo tratamiento o placebo

 Un hombre de 60 años viene a la oficina debido a varios episodios de mareos


durante los 3 últimos meses. Experimenta estos episodios mientras trataba de
caminar demasiado rápidamente o subir las escaleras. "casi desmayado"durante el
último episodio. El paciente describe su estilo de vida como activo pero
recientemente ha se sentía más cansado que de costumbre durante sus actividades
diarias. No ha tenido ningún dolor en el pecho o palpitaciones. Su historial médico
es importante para la diverticulosis, que toma suplementos de fibra. El paciente no
está en ningún otro medicamento. Su temperatura es de 37,1 C (98,8 F), la presión
arterial es 110/98 mm Hg, pulso 88/min y regular, y respiraciones 14/min. En el
examen, tiene un murmullos mediados de-sistólico 3/6 que es mejor escuchar sobre
el borde esternal superior derecho. Los pulsos carotídeos son levantamiento lento y
retrasado bilateralmente. Los campos del pulmón son claros a la auscultación. No
hay ninguna edema periférico. Cuál de los siguientes hallazgos adicionales es más
probable ser visto en física ¿examen?

A. Apical mediados de y último soplo diastólico


B. Un pico temprano del soplo sistólico 41%
C. fuerte primer corazón sano (S1) 19%
D. suave segundo sonido de corazón (S2)
E. tercer sonido de corazón 29% (S3)

fatiga progresiva de presentación clínica de este paciente y del exertional mareos y


presíncope, retrasado pulsos carotídeos y soplo mesosistólico sobre la derecha
borde esternal es sugestivo de estenosis aórtica (EA). Pacientes con AS son
asintomáticos por un período prolongado y clásico por lo general síntomas en
aquellos con severa como (área de la válvula aórtica y lt; 1 cm2). Algunos resultados
de examen físico sugestivos de severo, como son: disminuir y retraso del pulso
carotídeo (pulsus parvus y tardus") debido a la obstrucción del flujo de sangre 2. un
pico tardío soplo sistólico de turbulencia debido a la estenosis 3. Presencia de suave
y único segundo corazón sonido (S2) engrosamiento y calcificación de las valvas de
la aorta provoca movilidad reducida y causa un suave S2, S2 es debido
principalmente al cierre repentino de la válvula aórtica (A2). Además, como
resultado de la Movilidad reducida, A2 se retrasa y se produce simultáneamente con
el cierre de la válvula pulmonar (P2), llevando a un solo S2

Una mujer de 66 años es traída al servicio de urgencias por su hijo preocupado


debido a la creciente confusión, pérdida de movilidad y cambios de comportamiento.
El paciente tiende a gritar sin aparente razón y recientemente exclamó "un León
estaba durmiendo en el patio trasero." Ella ve a menudo en los gatos su habitación
cuando ninguno está presente. Tiene pérdida de memoria y con frecuencia se
pierde en su barrio. El paciente es un no fumador y no usar alcohol o drogas ilícitas.
Ella tiene no significativo historia psiquiátrica. Ella aparece alerta pero desorientado
y muy agitado. Ella es afebril; sangre presión es 136/72 mmHg, pulso es 98 por
minuto y respiraciones son muestra de la examinación física de 16/min. aumento del
tono muscular, reflejos normales y gruesos temblores en las extremidades.
Completa hemograma, electrólitos, creatinina, pruebas de función hepática de
glucosa, TSH y B12 niveles están dentro de rango normal. Reagina plasmática
rápida es negativa, y es de absorción de anticuerpos treponémicos fluorescentes no
reactivo. Cuando el paciente se vuelve combativo, recibe una pequeña dosis de
risperidona, que sólo agrava su rigidez. ¿Cuál de los siguientes es el diagnóstico
más probable?

A. Alzheimer enfermedad
B. demencia de cuerpos Lewy 89%)
C. Frontotemporal demencia
D. neurosífilis Demencia Vascular

Un hombre afroamericano de 35 años llega al médico quejándose de 2 semanas


de dolor abdominal ha sido en un programa de rehabilitación de drogas durante 2
años debido a abuso de la heroína antes de la distensión. Perfiles de VIH y hepatitis
viral reciente eran negativos. Su temperatura es de 37,1 c. (98,7), sangre presión es
145/82 mm Hg, es de pulso 80 por minuto y respiraciones son 14/min. El paciente
es obeso con una Índice de masa corporal de 40 kg/m2. El examen muestra tal
edema, ascitis y 2 picaduras edema en ambos piernas hasta las rodillas. Sonidos de
la respiración están disminuidos en base de pulmón derecho. Orina de la veinte-
cuatro-hora excreción de proteínas 7,5 g de muestra. Cuál de las siguientes es más
probable que esté presente en el riñón ¿biopsia?

A. nefropatía de IgA
B. glomerulosclerosis segmentario focal [70%]
C. membranosa
D. nefropatía mínima, enfermedad con cambio
E. amiloidosis sistémica

Mujer de 34 años llega al servicio de urgencias con una erupción y prurito. Ella
tenía una hace la garganta para varios días y tomó algún medicamento (amoxicilina)
la picazón que queda de año 1 erupción se convirtió aproximadamente 20-30
minutos después de que ella tomara la primera píldora hace 4 horas. No tiene
ninguna disnea, mareos, náuseas, diarrea o calambres abdominales. Ella no tiene
ninguna historia previa de erupciones en la piel o alergias. Su temperatura es de
36,7 ° C (98° F) la presión arterial es 124/82 mmHg y pulso 80/min saturación de
oxígeno es del 98% en aire ambiente. El examen físico muestra múltiples urticaria y
suave excoriaciones sobre las extremidades superiores e inferiores y parte superior
del cuerpo. Las amígdalas son ligeramente eritematosa, pero sin exudado o
hinchazón. No hay ninguna ampliación del nodo de linfa. Los pulmones están claros
en la auscultación sin wheezes. No hay estridor. El examen abdominal es
irrelevante. Que ¿de los siguientes es el mejor siguiente paso en la gestión de este
paciente?
A. administrar por vía intravenosa aspirina
C. recomendamos de glucocorticoides
B. administrar epinefrina intramuscular con la próxima dosis de amoxicilina
D. enviar para anticuerpos IgM contra el virus de Epstein - Barr
E. tratamiento con el antihistamínico 72%

Los síntomas de este paciente son típicos de la leve reacción de hipersensibilidad


inducida por drogas, tipo 1. Algunos pacientes pueden formar la inmunoglobulina
específica de drogas E (IgE) en la exposición a un medicamento, aunque la mayoría
no. Una vez formada, la lgE específicos de drogas ocupa los receptores de
mastocitos y basófilos. Si la droga se encuentra otra vez, estas células pueden
activar, resultando en síntomas. Inicio es rápido (segundos a minutos) y los
síntomas puede variar de suave (por ejemplo, urticaria, prurito, rubor) a más severa
(por ejemplo, angioedema de laringe, anafilaxia). Más frecuentemente implicados en
reacciones de tipo 1 son drogas de beta lactámicos, neuromusculares bloqueo de
agentes, quinolonas, platino agentes quimioterapéuticos y proteínas extrañas (por
ejemplo, quimérico anticuerpos) Urticaria y prurito sin síntomas sistémicos
generalmente son tratados con antihistamínicos y tratamiento con el fármaco que
está descatalogado (opción C)
El anuncio de la droga en el botón de exposición se aplica a los 2 a continuación
artículos en que el anuncio de drogas se centra kalgatran (Kalaxin), un nuevo
anticoagulante por vía oral para el tratamiento de la no-valvular atrial fibrilación
auricular. Artículo 1 de 2 riesgo de hemorragia es la principal preocupación cuando
los pacientes con anticoagulante fibrilación auricular no valvular. El anuncio
promueve el Kalaxin como un medicamento seguro. En estudio de comparación
Kalaxin y warfarina, cuál de los siguientes riesgos específicos de sangrado fueron
más ¿similares entre los 2 grupos? Mostrar anuncio de drogas

A. Gastrointestinal sangrado [6%]


B. intracraneal sangrado 18 %l
C. mortal sangría 25%)
D. mayor sangrado (38 %l
E. Total sangrado [20%]

El anuncio de drogas es comparar el efecto de Kalaxin versus warfarina para la


prevención del accidente cerebrovascular en no - fibrilación auricular valvular.
Compara las tarifas de peligro para eventos adversos de ambas drogas en forma de
cocientes de riesgos instantáneos. Una tasa de riesgo es la probabilidad de un
evento que ocurre en uno de los grupos de estudio durante un periodo establecido.
Un cociente de riesgo es la probabilidad de un evento que ocurre en el grupo de
tratamiento comparado con la probabilidad de ese evento que ocurre en el control
Grupo durante un periodo establecido. Un cociente de riesgo y lt; 1 indica que un
evento es más probable que ocurra en el grupo de control. Un cociente de riesgo y
gt; 1 significa que un evento es más probable que ocurra en el grupo de tratamiento.
Un cociente cercano a 1 implica poca diferencia entre los 2 grupos. En este estudio,
el cociente de riesgo para sangrado mayor fue o.96, que está más cerca de la 1 a la
otra respuesta Opciones. Además, intervalo de confianza (0.84 1.10) contiene el
valor null 1. que no existe ninguna diferencia significativa en el riesgo de sangrado
mayor de entre los 2 grupos.

Artículo 2 de la 2 cuando la asignación de pacientes a los grupos tratamiento y


control, aleatorización adecuada es importante para evitar el efecto de variables
extrañas en los resultados del estudio. En el estudio actual, Cuál de las siguientes
formas de información adicional ayudaría más en la determinación de asignación al
azar ¿éxito? Muestran tasas de movimiento anual de drogas Ad

A. características de los pacientes


B. base 71 %
C. Cumplimiento del paciente tabla
D. Tasas de seguimiento paciente
E. análisis de subgrupo de 4%de l
F. las tablas [16%]
Un proceso de aleatorización ideal minimiza el sesgo de selección, resultados en
cerca de igual tratamiento y control de tamaños de grupo y logra una baja
probabilidad de tener variables confusoras. Una tabla de características de
referencia de pacientes para tratamiento y control los grupos mostrarían si los 2
grupos incluyeron a pacientes con características similares y ayudaría a confirmar
adecuada aleatorización. (Opción A) Tasa anual de ictus es un punto final del
estudio que puede ser influenciado por aleatorización de pacientes, pero no
determina el éxito de la aleatorización. (Opción C) Cartas de cumplimiento del
paciente indican cuán bien un paciente tolera el tratamiento y seguido el régimen de
tratamiento, pero no indica directamente el éxito de la aleatorización. (Opción D) Las
tasas de seguimiento paciente indican cuántos pacientes seguidos o abandonaron
el estudio. Estos los datos no son un marcador directo del éxito de la asignación al
azar.

Mujer de 32 años acude a la oficina debido a dolor abdominal intermitente y sin


sangrado; diarrea durante los últimos 3-4 meses. Ella describe el dolor tipo cólico y
situado en la parte central del abdomen más bajo derecho cuadrante. El paciente
piensa que ha perdido algo de peso durante este período. Ella modificó su dieta
varias veces en un intento de disminuir los síntomas, creyendo que las comidas
pesadas exacerban el dolor y diarrea. El paciente experimentó la apendicectomía
con el desbridamiento del absceso hace un año. Ella no tiene otros problemas
médicos y ninguna medicación que no sea anticonceptivos orales. Ella no ha viajado
fuera de Estados Unidos y no uso tabaco, no alcohol o drogas recreativas.
Temperatura es de 36,7 C (98 F), la presión arterial es 120/70 mm Hg, pulso es de
85 por minuto y respiraciones son 14/min. Algunas úlceras poco profundas están
presentes en la boca. El examen abdominal muestra suave ternura en el cuadrante
inferior derecho sin rebote. Resultados de laboratorio son: hemoglobina 10.2 g/dL de
sedimentación de eritrocito de leucocitos 14.500/mm3 plaquetas/mm3 530.000 48
mm/hr de velocidad ¿la siguiente es la diagnosis más probable para este paciente?

A. la enfermedad celíaca
B. Crohn enfermedad (85%]
C. Giardia infección [0%]
D. síndrome del intestino Irritable
E. intolerancia a la lactosa
E. Infección por Yersinia g.
F colitis ulcerosa

Una mujer de 32 años de edad llega al Departamento de emergencia debido a la


hinchazón del brazo izquierdo en los últimos 24 horas. El paciente no tiene
problemas médicos crónicos y no lleva medicamentos. Ella bebe alcohol 2 - 3 veces
a la semana y utiliza intravenosa de la heroína diariamente. Temperatura es de 37.8
C (100 F), presión arterial es 105/62 mm Hg, pulso es de 92 por minuto y
respiraciones son 16/min revela de auscultación de pecho claro Campos
pulmonares y ruidos cardíacos normales de primero y segundo. El abdomen es
suave. Examen de las extremidades muestra varias marcas de aguja. El brazo
izquierdo está eritematosa e inflamada así como cálidos y sensibles al tacto. No hay
ningún empalme hinchazón y rango de movimiento es normal en todas las
articulaciones. El paciente se inicia en clindamicina intravenosa. A la mañana
siguiente la inflamación y se mejora el dolor, pero el paciente informes "sensación
miserable." Tiene congestión nasal, náuseas, y los calambres abdominales además
de múltiples episodios de vómitos heces sueltas. En repetición examen, el paciente
está inquieto y dolor de músculos y articulaciones. La presión arterial es 132/88 mm
Hg, pulso 102min y regular, y respiraciones 16/min resultados de laboratorio de
tiempo de admisión (0 horas) y 24 horas más tarde son los siguientes: hemograma
completo 0 horas 24 horas hemoglobina 12 g/dL 11,4 g/dL plaquetas/mm3 280.000
240.000/mm leucocitos 13.800/mm 8.500 mm Neutrófilos 80% 62%, linfocitos 15%
24%, cuál de los siguientes es el más apropiado siguiente paso ¿gerencia de este
paciente?

A. administrar morfina intravenosa


B. deje clindamicina
C. iniciar naltrexona intravenosa
D. iniciar tratamiento para la abstinencia de opiáceos 81%
E comienzo de E. Flumazenil
F. prueba para toxina de Clostridium difficile

Una mujer de 34 años de edad llega a la oficina debido a una historia de


moretones con 6 meses mínimo o ningún trauma. Ella también ha tenido subir
escaleras dificultad debido a la debilidad muscular y frecuentes Calambres
nocturnos. El paciente ha experimentado una ganancia de peso de 7 kg (15 lb) a
pesar de sigue un dieta restringida en calorías vegano. Su historial médico es
notable para una endodoncia sin complicaciones procedimiento hace 6 meses pero
es de otra manera irrelevante. Ella no toma receta o de venta libre medicamentos. El
paciente no está sexualmente activo y nunca ha estado embarazado. Su último
periodo menstrual fue hace 3 meses. No hay ninguna historia familiar de trastornos
hemorrágicos. Su sangre presión es 146/94 mm Hg y el pulso es 78/min índice de
masa corporal es de 29 kg/m2. El examen muestra plétora facial con pelo terminal
oscura en su labio superior y barbilla. Hay inflamatorio del acné en su cara y la
espalda, amplia tiña versicolor en el tronco y disperso contusiones en las
extremidades. El ayuno resultados de laboratorio son las siguientes: cuenta de
sangre completa 14 g/dL. Hemoglobina 160.000/mm3 plaquetas 9.500/mm3
leucocitos 80% neutrófilos eosinófilos 0% 17% linfocitos 2% monocitos 1% basófilos
suero química 140 mEq/L sodio potasio 3.6 mEq/L, 100 mEq/L cloruro 28 mEq/L.
bicarbonato creatinina 0.8 mg/dL. Calcio 9 mg/dL. Glucosa 122 mg/dL, cuál de las
siguientes es la más probable causa de ¿síntomas de este paciente?

A. Cushing síndrome (70%)


B. vasculitis de vaso pequeño cutánea
C. ovario síndrome poli quístico
D. vitamina C deficiencia
E. vitamina K la deficiencia

Un hombre de 67 años de edad con enfermedad pulmonar obstructiva crónica


leve es traído a la emergencia Departamento de su hija con la queja principal de
ataque. El paciente ha tenido episodios de confusión y letargo durante la semana
pasada. Él se queja de algunos disnea del exertional y tos no productiva. Su hija
cree que está perdiendo peso y ha disminuido el apetito. Su temperatura es de 37,2
c. 99 es de la presión arterial 134/88 mm Hg y el pulso es 104/min e irregular. Las
mucosas son húmedas y no hay ningún edema periférico. El examen neurológico es
irrelevante. Resultados de laboratorio son las siguientes: realizar hemograma
hematocrito 34% suero química sodio 5,4 mEq/L potasio mEq/L 22 bicarbonato
mEq/L 1.3 creatinina mg/dL rápido corrección de anormalidades metabólicas p de
este paciente en mayor riesgo de que de la ¿siguientes?

A. Cerebral edema
B. ictus embólico
C. hidrocefalia
D. Osmótica arritmias ventriculares
E. Desmielinización
Una mujer de 67 años de edad llega a la oficina para el seguimiento. Cuatro
semanas atrás, ella experimentó el inicio repentino adormecimiento del lado derecho
y la evaluación revelan un infarto cerebral isquémico. El paciente se siente como
algunos de sus sensación regresa pero experimenta dolor ardor transitorio en las
extremidades superiores e inferiores puede ser inducida incluso por un ligero toque.
Su historial médico es importante para la hipertensión y el tipo 2 mellitus de la
diabetes. Ella fumaba un medio paquete de cigarrillos diario durante 30 años, pero
paró después de su movimiento. Presión arterial es 125/70 mm Hg y el pulso es
74/min y regular. Examen neurológico muestra derecha hemianesthesia y atetosis
suave de la mano derecha. Hay hiperestesia en el lado derecho del cuerpo
demostrado por dolor exagerado en la luz a fuerza del Motor es normal en todos 4
extremidades. Cuál de las siguientes es la ubicación más probable del trazo
experimentan 4 ¿hace semanas por este paciente?
A Cápsula interna
B. médula
C. mesencéfalo
D. Convolución del cerebro central posterior
E putàmen.
F. tálamo [56%] este paciente probablemente experimentado un
movimiento lagunar del tálamo posterolateral, que normalmente se produce debido
a obstrucción atherothrombotic de la pequeña, penetrando (thalamogeniculate)
ramas de la parte posterior de la arteria cerebral. El ventral posterolateral y el núcleo
ventral posteromedial del tálamo transmitir la información sensorial del lado
contralateral del cuerpo y la cara, respectivamente. En consecuencia, pacientes con
accidente cerebrovascular talámico en esta región a menudo presente con inicio
repentino pérdida sensorial contralateral que involucra todas las modalidades
sensoriales (es decir, puro movimiento sensorial). Los síntomas pueden
acompañada de Hemiparesia transitoria, atetosis o movimientos balísticos debido a
la interrupción de la estructuras vecinas de los ganglios basales y las fibras
corticoespinales en la extremidad posterior de la interna cápsula - unidad de uso de
producto (calificador)

Un varón de 65 años llega al servicio de urgencias debido a una pérdida


repentina, indolora de la visión en su ojo derecho produjo una media hora atrás.
Hace cinco horas que experimentó una similar pero con pérdida de la visión en el
ojo mismo, que se prolongó durante cinco minutos. Tiene hipertensión, diabetes,
hipercolesterolemia y enfermedad vascular periférica. Él tenía un infarto miocardio
de pared anterior hace años. Sus medicaciones incluyen glibenclamida, captopril,
atenolol, simvastatina y aspirina. Su temperatura es de 36,7 C (98,0 ° F),
respiraciones son 16/min, pulso es de 88/min y la presión arterial es 146/88 mmHg.
Examen del ojo derecho revela la agudeza visual de 20/60 y sutil de la retina
hitening. Bien se oye un soplo en la carótida. Cuál de los siguientes es el paso
siguiente más apropiado ¿en el manejo de este paciente?
B. masaje ocular y alto flujo oxígeno
A. los corticosteroides sistémicos,
D. tópicos beta bloqueador
E. trombolíticos (respuesta correcta 45%
El diagnóstico más probable es la oclusión de la arteria retiniana central (OACR)
secundario a la embolia. Pérdida indolora de la visión monocular es el síntoma que
se presenta generalmente. Factores de riesgo incluyen enfermedad de la arteria
carótida, enfermedad valvular cardiaca, endocarditis, huesos largos fractura,
condiciones hipercoagulables, vasculitis, Mixoma auricular, etcetera. Se produce
cuando un severo, disminución brusca del flujo sanguíneo a través de la arteria
retiniana central causa isquemia de la interna retina. (Recordemos que la arteria
oftálmica es la primera rama intracraneal de la interna de la arteria carótida.
Suministra sangre a la vista a través de la arteria retiniana central (1), que es un fin
la arteria que suministra la retina interna y las ramas ciliares (2), que suministran los
choroids y porción anterior del globo). Embolia de la arteria retiniana es la causa
más común de ocular accidente cerebrovascular. Los émbolos pueden viajar a las
ramas distales de la arteria de la retina, causando la pérdida de sólo un sección del
campo visual. Se asocia comúnmente con amaurosis fugaz antes de la oclusión
(como en este paciente). La agudeza visual suele ser 20/800 (6/240) o peor.
Fundoscopia revela blanqueamiento retiniano isquémico difuso y manchas rojo
cereza (típicas pero no específicas para la OACR)

Un hombre de 80 años con cáncer de próstata avanzado y metástasis ósea se


queja de severo dolor de espalda; dolor que ha empeorado progresivamente
durante las últimas dos semanas. Su dolor es tan severo que lo restringe a jugar al
golf. Hace ocho meses, se sometió a orquiectomía, después de lo cual fue desde el
dolor de huesos hasta ahora. El examen físico revela sensibilidad en dos sitios en
la región lumbar La gammagrafía ósea con radionúclidos muestra un aumento en la
absorción en estas áreas. ¿Cuál de los siguientes es el más apropiado siguiente
paso en el manejo de este paciente?

A. Cordotomía cervical
B. Etidronato Terapia disódica
C. Terapia con flutamida
D. Hipofisectomía
E. Radioterapia [68%] La radioterapia es la más adecuada para el
tratamiento del dolor progresivo en un paciente con cáncer de próstata y metástasis
ósea después de la ablación androgénica (orquiectomía). Focal la radioterapia
externa es una excelente opción en este paciente, ya que la metástasis se localiza
en pocos sitios. (Elección A) la cordotomía cervical no es efectiva para controlar el
dolor o dolor axial en el extremidades superiores (Elección B) Etidronato disódico y
otros bisfosfonatos pueden ser útiles en pacientes con metástasis ósea, ya que
reduce la resorción ósea. Es útil en el control de enfermedades crónicas dolor de
metástasis ósea, pero su inicio de acción para aliviar el dolor óseo agudo no es tan
rápido como radioterapia. (Opción C) La flutamida es un antiandrógeno no
esteroideo que proporciona andrógeno bloqueo mediante la unión competitiva a los
receptores de dihidrotestosterona. Este medicamento, cuando se combina con un
Agonista de LHRH, prolonga la supervivencia de pacientes con enfermedad
limitada. Sin embargo, no ha habido beneficios demostrables en pacientes
sometidos a orquiectomía, como en este paciente. Estramustina, una combinación
de mosto de estrógeno y nitrógeno, tiene una tasa de respuesta del 40% en
hombres castrados, pero no es Bien tolerado en personas con trastornos
cardiovasculares o hematopoyéticos.
Una mujer de 29 años acude a la consulta debido a episodios frecuentes de dolor
de cabeza. Ella tiene moderada dolor severo, de lado izquierdo, palpitante asociado
con náuseas y vómitos ocasionales. Su dolor de cabeza es a menudo precedido
por una sensación de hormigueo en la mano derecha que gradualmente involucra el
brazo derecho y cara y es seguido por entumecimiento duradero y lt; 1 hora. Se
siente irritable mientras experimenta el dolores de cabeza El dolor generalmente
mejora después de varias horas de descanso en una habitación oscura y silenciosa.
El paciente ha tenido estos episodios cada 1-2 meses durante los últimos 5
años. Ella ha tomado según sea necesario sumatriptán con cierto alivio, pero los
síntomas le han hecho perder varios días de trabajo. Ella no tiene otro historial
médico y no usa tabaco, alcohol o drogas recreativas. Hay un antecedentes
familiares de depresión Los signos vitales están dentro de los límites normales y el
examen físico es normal. ¿Cuál de los siguientes es más probable que sea útil para
prevenir este paciente? dolores de cabeza?

A. Beta-interferón
B. Glucocorticoide
C. Levetiracetam
D. Metoclopramida
E. Sertralina
F. Topiramato 68%

Una mujer de 65 años llega a al consultorio debido a una hemorragia


gastrointestinal. Ella ha tenido 3 o 4 episodios de heces de color marrón oscuro en
las últimas 2 semanas, con intestino intermedio normal movimientos. El paciente no
presenta dolor abdominal o rectal asociado, náuseas o vómitos. Ella tiene un
antecedentes de hipertensión, diabetes mellitus tipo 2 e hipercolesterolemia. Su
temperatura es 36.7 C (98 F), la presión arterial es 140/80 mm Hg, el pulso es 95 /
min, y las respiraciones son 16 / min. La exploración física revela un soplo de
eyección sistólica 3/6 en el segundo espacio intercostal derecho. Los pulsos
carotídeos se retrasan a la palpación bilateral. Los exámenes abdominales y
rectales son benignos. Los resultados de laboratorio son los siguientes:
Hemoglobina 11.1 g / dL Volumen corpuscular medio 90 um Sangre urea nitrógeno
34 mg / dL. La colonoscopia con creatinina 1,6 mg / dl 6 meses antes no fue
notable, pero fue algo limitado en el colon ascendente debido a h intestinal inferior a
la óptima de los siguientes es el más probable causa de los síntomas de este
paciente?
A. Angiodisplasia [50%]
B. Cáncer de colon
C. Diverticulosis
D. Hemorroides
E. Colitis isquémica

 Se Trata de una mujer de 54 años con una larga historia de hipertensión arterial y
estenosis mitral reumática leve multiples visitas al Departamento de la emergencia
de cambios severos de la piel. Ella fue diagnosticada recientemente con la fibrilación
auricular y se inició terapia de warfarina su médico hace varios días. El paciente ' s
otros medicamentos incluyen hidroclorotiazida y metoprolol. En el examen físico, su
la temperatura es 36.7 c (98 F), la presión arterial es 130/80 mm Hg, el pulso es 80 /
min e irregular, y respiraciones son 16 / min. Los cambios en la piel se muestran a
continuación. El examen cardíaco revela un desmayo soplo mesodiastólico. El resto
del examen no tiene nada especial. ¿Cuál de los siguientes es la causa principal de
la presentación clínica de este paciente;
A. Anticuerpos anti-factor plaquetario 4 [6%]
B. Deficiencia de antitrombina III [4%]
C. Factor V Leiden 13%]
D. Deficiencia del factor VII [4%]
E Deficiencia de proteína C 80%

Una mujer de 65 años acude al servicio de urgencias debido a 1 día de hinchazón


dolorosa de la izquierda lado de su cara, asociado con fiebre alta y escalofríos. Ella
no ha tenido trauma facial o lesión a esa área. El paciente fue diagnosticado con
diabetes mellitus tipo 2 hace 3 años y logra buenos resultados control glucémico con
dieta, ejercicio y agentes hipoglucemiantes orales; ella no tiene complicaciones
conocidas. La temperatura es de 39.2 C (102.6 F), la presión arterial es de 125/75
mm Hg, y el pulso es de 90 / min. Físico El examen muestra una erupción
eritematosa, cálida y sensible con bordes elevados y bien demarcados en el lado
izquierdo de la cara, incluida la oreja externa izquierda. El canal auditivo no tiene
secreción y audición está intacto Linfadenopatía regional leve está presente. ¿Cuál
de los siguientes es el más probable organismo causante de la infección de este
paciente?

A. Clostridium perfringens [1%]


B. Enterococci [0%]
C. Streptococcus del grupo A [34% l
D. Haemophilus influenzae
E. Pseudomonas aeruginosa
F. Staphylococcus aureus
G. Streptococcus agalactia
H. Streptococcus pneumoniae

La siguiente viñeta se aplica a los siguientes 2 artículos. Los elementos del
conjunto deben responderse en orden secuencial. Una vez que haga clic en
Proceder al siguiente artículo, no podrá agregar o cambiar un responder. Este
elemento tiene medios asociados que pueden requerir el uso de auriculares. Por
favor asegúrate de el volumen del sistema / altavoz está configurado a un nivel
audible. Un hombre de 73 años acude al médico para evaluación de un temblor. en
los últimos meses, ha notado que se pone trémulo cuando tratando de sostener un
periódico y al beber su taza de café de la mañana. El temblor no es notablemente
peor durante cualquier momento del día. Él no tiene más problemas médicos que
hipertensión, por lo que toma clortalidona. Sus signos vitales son normales. Examen
físico incluida la evaluación de su modo de andar, es normal. El paciente muestra un
temblor, que se muestra en el video. Artículo 1 de 2 ¿Cuál de los siguientes es el
diagnóstico más probable en este paciente?

A. Temblor esencial 70%


B. corea de Huntington
C. enfermedad de Parkinson
D. efecto secundario de cafeína
E. Ataxia espinocerebelosa
Ítem 2 de 2 ¿Cuál de los siguientes es el siguiente mejor paso en el manejo de
este paciente?
A. Carbidopa / levodopa
B. Cesación de la ingesta de café
C. Estimulación cerebral profunda
D. Consejo genético
E. Propranolol 83% El tratamiento de los esenciales por lo
general, el temblor comienza con el betabloqueante propranolol, que es
especialmente útil si el paciente también tiene hipertensión coexistente. otras
posibilidades de tratamiento incluyen anticonvulsivos, como primidona y topiramato.
Aunque las benzodiazepinas como el alprazolam son efectivas en reduciendo los
síntomas del temblor esencial, su uso debe restringirse debido a la posibilidad de
dependencia. (Opción A) carbidopa / levodopa es el tratamiento de primera línea
para pacientes con Parkinson enfermedad. (Opción B) Cualquier agente que
aumente la actividad adrenérgica (p. Ej., Cafeína) puede exacerbar temblor
fisiológico. Sin embargo, los estudios han demostrado que el uso de cafeína no está
necesariamente correlacionado con la severidad del temblor esencial. (Opción C)
opciones quirúrgicas como la estimulación cerebral profunda y la talamotomía se
consideraría solo si el temblor tuvo un impacto significativo en la función diaria y
después de no controlar el temblor con varios medicamentos. (Opción D) El
asesoramiento genético puede ser beneficioso en el tratamiento de enfermedades
como la enfermedad de Huntington y la ataxia espinocerebelosa debido a su
profundo efecto en la calidad de vida y la duración de la vida. Sin embargo, el
asesoramiento genético no es indicado en este paciente

Una mujer de 52 años acude al médico con una erupción en la cara que comenzó
hace unas semanas. Últimamente, el paciente también ha tenido dificultades para
levantarse de una posición sentada y subir escalera. en el examen, una erupción
eritematosa en la parte superior del tórax y un edema periorbitario violáceo están
presentes. Sus signos vitales están dentro de los límites normales. El examen
muestra músculo proximal simétrico debilidad en las piernas. ¿La condición de este
paciente se asocia con más frecuencia a cuál de los siguientes?

A. Hemorragia alveolar
B. Aneurismas aórticos
C. Síndrome del túnel carpiano
D. Glomerulonefritis
E. Malignidad 57%
F. Pérdida visual repentina

Un hombre de 65 años es evaluado en el departamento de emergencias después


de haber estado involucrado en un accidente moto. Él describe una breve pérdida
de conciencia durante el choque. Desde que recuperó la conciencia, ha marcado
debilidad en ambas extremidades superiores, pero ha sido capaz de mover sus
extremidades inferiores. El paciente tiene una historia de osteoartritis y espondilosis
cervical secundaria a cambios degenerativos en el columna cervical, pero por lo
demás es saludable. Los signos vitales son estables. Radiografías de película
simple del cuello la columna vertebral no muestra anomalías, excepto las que
concuerdan con la artritis degenerativa leve. De los cuales siguiente es el
diagnóstico más probable?

A. Síndrome de la médula anterior


B. Síndrome de Brown-Séquard
C. Síndrome del cordón central [49%]
D. Contusión cerebral
E. Síndrome del cordón posterior

El síndrome del cordón central (ccs) típicamente ocurre con lesiones por
hiperextensión en ancianos pacientes con cambios degenerativos preexistentes en
la columna cervical. Este tipo de traumático lesión causa daño selectivo a la porción
central de la médula espinal anterior, específicamente el partes centrales de los
tractos corticoespinales y las fibras decussating del lateral El CCS del tubo
espinotalámico se caracteriza por una debilidad que es más pronunciada en la parte
superior extremidades que el inferior. Este déficit motor único se produce porque las
fibras motoras que sirven al los brazos están más cerca de la parte central del tracto
corticoespinal. Un paciente con CCS también puede tener pérdida selectiva
ocasional de dolor y sensación de temperatura en los brazos debido a daños en
tracto espinotalámico. (Elección A) El síndrome del cordón anterior (ventral) se
caracteriza por bilateral paresia motora espástica distal a la lesión. Por lo general,
se debe a la oclusión de la espina anterior artería. (Opción B) La hemisección del
cordón umbilical, la mayoría de las veces debido a una lesión penetrante, puede
ocasionar Ndrome Brown-Séquard, que se caracteriza por Ipsilateral: debilidad,
espasticidad, pérdida de sentido de vibración y propiocepción Contralateral: pérdida
de dolor y sensación de temperatura

Un hombre afroamericano de 22 años se presenta en urgencias con fiebre,


ictericia, dolor abdominal y orina oscura. Su frecuencia cardíaca es de 100 / min y la
presión arterial es de 100/60. Frotis de sangre periférica revela las células de
mordida y las inclusiones de glóbulos rojos se observan después de la tinción con
cristal violeta. El paciente más Probablemente sufre de cuál de las siguientes
condiciones?

A. Hepatitis viral aguda


B. Agudo glomerulonefritis
C. Deficiencia de enzima [75%]
D. Talasemia menor
E. Rasgo de células falciformes
F. Esferocitosis hereditaria

Este paciente sufre de una enfermedad aguda crisis hemolítica, con células de
mordida y cuerpos de Heinz vistos en el frotis de sangre periférica. Esto es más
compatible con la deficiencia de glucosa-6-fosfato deshidrogenasa (G6PD) G6PD es
una enzima involucrada en creando NADPH, un cofactor requerido para crear
glutatión y prevenir la oxidación de la hemoglobina. Sin G6PD, la hemoglobina se
oxida y se desnaturaliza en cuerpos de Heinz. La desnaturalización h interrumpe
membranas de glóbulos rojos (RBC) y causa hemólisis. Esta reacción ocurre
típicamente en respuesta a fármacos oxidantes antipalúdicos (por ejemplo, sulfas) e
infección. Las habas también pueden precipitar hemólisis La deficiencia de G6PD es
un trastorno recesivo ligado a x común en hombres afroamericanos (Elección A) La
hepatitis viral aguda, secundaria a la hepatitis A o B, se caracteriza por náuseas
vómitos, dolor abdominal e ictericia. La fiebre y la confusión también pueden ocurrir.
Orina oscura puede ser vista debido a la hiperbilirrubinemia. Las células de mordida
y Heinz no se ven en la hepatitis viral aguda. (Elección B) La glomerulonefritis aguda
se caracteriza por el rápido desarrollo de hematuria microscópica, disminución de la
TFG e hipertensión. Las células de mordida y los cuerpos de Heinz no se ven en
glomerulonefritis aguda (opción D) Talasemia menor describe a las personas que
son heterocigóticas para B-talasemia. Las personas con talasemia menor suelen ser
asintomáticas y la sangre periférica el frotis muestra microcítico.

Un hombre de 23 años viene a la oficina debido a una lesión genital. El paciente
notó un pequeño golpe en el eje de su pene hace 2 días que creció en tamaño antes
de ulcerarse. La lesión no es dolorosa y el paciente no tiene fiebre u otras lesiones
en la piel. Él nunca ha tenido síntomas similares. El paciente tiene no hay problemas
médicos crónicos, pero es muy alérgico a la penicilina. Recientemente comenzó a
salir con un nuevo pareja sexual, y su uso de condones ha sido inconsistente. Los
signos vitales están dentro de los límites normales. El examen genital muestra una
úlcera de 2 cm en el eje medio del pene con bordes elevados e indurados y una
base lisa. La palpación no provoca dolor y no hay exudado presente. Sin piel
adicional se encuentran lesiones y el examen oral es normal. Leve inguinal bilateral,
no dolorosa linfadenopatía está presente. La prueba de suero VDRL y VIH es
negativa. ¿Cuál de los siguientes es ¿Es más probable que genere el diagnóstico en
este paciente?
A. Absorción de anticuerpos treponémicos fluorescentes 75%
B. Aspiración de nódulo linfático
C. Prueba de Pathergy
D. biopsia de úlcera y cultivo

Una mujer de 32 años acude a la clínica para recibir atención continua de su


infección por el VIH. Ella informa moderada pérdida de peso. Ella fue diagnosticada
con infección por VIH hace 8 años y ha tenido poca adherencia a ella terapia
antirretroviral. El paciente "se siente bien" y no ha tenido quejas recientes,
enfermedades o hospitalizaciones. Los signos vitales están dentro de los límites
normales. El examen no muestra anomalías, excepto linfadenopatía difusa leve. Su
recuento de CD4 es de 30 células / UL y la carga viral es de 300,000 copias / mL.
Ella acepta cumplir con la terapia. Además de la terapia antirretroviral, ¿cuál de los
siguiente está indicado para este paciente?

Trimethoprim Sulfametazol Azithromycin


Fluconazole Acyclovir
Corticoides

Un hombre de 51 años acude al médico con dificultad para tragar sólidos pero no
líquidos. Su la historia médica es significativa para la enfermedad por reflujo
gastroesofágico durante los últimos 12 años. Él se sometió a una endoscopia y se le
diagnosticó esófago de Barrett hace 6 meses. Su acidez resuelto 3 meses después
de este diagnóstico. Los signos vitales del paciente están dentro de los límites
normales. Su índice de masa corporal es de 38 kg / m2. La deglución de bario ahora
muestra un área simétrica, circunferencial estrechamiento que afecta el esófago
distal. ¿Cuál de los siguientes explica mejor este hallazgo?

A. Acalasia
B. Adenocarcinoma esofágico
C. Estenosis esofágica [62%]
D. Hernia hiatal
E. Anillo vascular

La presentación de este paciente crónica El reflujo gastroesofágico con nueva


disfagia y estrechamiento esofágico inferior simétrico sugiere estenosis esofágica
(péptica). El reflujo gastroesofágico crónico y (ERGE) predispone a Barrett s de la
crónica inferior Ambas condiciones son consecuencias de la respuesta reparadora
del cuerpo a pacientes con exposición al ácido gástrico y pueden ocurrir
simultáneamente. Las estenosis benignas afectan del 5% al 15% de cáustico con
ERGE. Otras causas de estenosis péptica incluyen radiación, esclerosis sistémica y
Las estenosis típicamente provocan disfagia lentamente progresiva a los alimentos
sólidos sin anorexia o pérdida de peso A medida que avanzan, en realidad pueden
bloquear el reflujo, lo que lleva a la mejora de síntomas de acidez estomacal (como
se ve en este paciente). Las escisiones tienden a aparecer como simétricas, el
estrechamiento circunferencial en el bario lo permite. No obstante, en cualquier caso
de estenosis en el entorno de El esófago de Barrett, la biopsia es necesaria para
descartar un adenocarcinoma. Esto usualmente se logra a través de una
endoscopia que puede ser diagnóstica y terapéutica (la dilatación se realiza si no
hay malignidad detectado). (Opción A) La acalasia es una causa esofágica

Se realizó un estudio para evaluar la relación entre el nivel sérico de HDL2 (una
subfracción de HDL y grosor íntima-media carotídeo como marcador de
aterosclerosis. Interpretación de los resultados reveló una relación lineal entre estas
2 variables, con un coeficiente de correlación r -0.25 (p 0.005). ¿Cuál de las
siguientes afirmaciones sobre los resultados del estudio es la más correcta?

A. Como El nivel de HDL2 aumenta, hay una disminución asociada en el


grosor de la íntima media carotídea
B. La disminución del nivel de HDL2 es la causa del engrosamiento de la
íntima-media carotídea
C. La asociación no es estadísticamente significativo
D. Existe una fuerte correlación entre el nivel de HDL2 y la carótida espesor
íntima-media
E. Existe una correlación positiva entre el nivel de HDL2 y la carótida espesor
íntima-media [4%]
La Coeficiente de correlación Negativo Sin correlación Correlación de
correlación positiva 1.0 -0.5 Fuerte 0.5 OUMundo débil Débil 1.0 La correlación
Fuerte la correlación (r evalúa una relación lineal entre 2 variables. El valor nulo
para positivo o el coeficiente es 0 (sin asociación), y el signo del coeficiente de
correlación indica un esta asociación negativa. Cuanto más cerca esté r de sus
márgenes (-1 o 1), más fuerte será la asociación (estudio dado, ya que hubo una
correlación negativa entre HDL2 y el grosor de la íntima media carotídea e -0.25 y lt;
0), cuando hubo un aumento en una de las variables (p. Ej., HDL2), hubo un
dirección en la otra variable (p. ej., grosor íntima-media carotídeo). Esta declaración
con respecto al La elección del cambio no implica causalidad (Opción B) C) El

Una mujer de 34 años viene a la oficina debido a la fatiga durante los últimos
meses. Ella se convierte cansado fácilmente después de caminar distancias cortas.
También tiene dificultad para peinarse debido a problemas sosteniéndole las manos
sobre su cabeza por largos períodos. El paciente informa ansiedad, irritabilidad y
una pérdida de peso involuntaria de 3,6 kg (8 lb) en los últimos 2 meses. Ella no
tiene fiebre o pérdida de apetito. El paciente no usa tabaco ni alcohol. Su padre
murió de un derrame cerebral a los 54 años y su madre tiene diabetes mellitus. En
el examen, el paciente está afebril con un pulso de 115 / min. El examen cardíaco
revela taquicardia regular sin soplo. Los pulmones son claro para la auscultación. La
marcha del paciente es normal, pero ella se deja caer en la silla torpemente pidió
sentarse lentamente. La paciente parece tener una masa muscular disminuida en
sus hombros, pero su los músculos no son sensibles Los reflejos tendinosos
profundos son normales. ¿Cuál de los siguientes es el más probable causa de los
síntomas de este paciente?

A. Lesión de los ganglios basales


B. Disfunción cerebelosa
C. Enfermedad inflamatoria del músculo
D. Trastorno de la unión neuromuscular
E. Polineuropatía
F. Enfermedad de la tiroides [43%]
G. Enfermedad de la neurona motora superior

Un hombre de 65 años acude a la clínica debido a un dolor de pecho subesternal


periódico en los últimos 6 meses. Los episodios de dolor ocurren durante una
emoción fuerte, duran de 10 a 15 minutos y se resuelven espontáneamente. El
paciente tiene una larga historia de hipertensión y diabetes mellitus tipo 2. Su pie
derecho fue amputado hace 2 años debido a complicaciones de la diabetes. La
imagen de perfusión miocárdica se realiza para evaluar la posible enfermedad
obstructiva de la arteria coronaria. La imagen en reposo revela uniforme distribución
del isótopo radiactivo en todo el miocardio. Imágenes después de la administración
de un análogo de adenosina revela la captación isotópica radiactiva no uniforme en
el miocardio, consistente con m isquemia ¿Cuál de los siguientes describe mejor el
mecanismo del cambio observado? porción de estrés durante el estudio?

A. Aumento del flujo en vasos no obstruidos (51%)


B. Mejora de la contractilidad del miocardio
C. Incremento de la poscarga cardíaca
D. Aumento retorno venoso sistémico
E. Vasoconstricción de vasos estenóticos

Se solicita a un residente médico de turno que vea a una mujer de 72 años con
dolor de cabeza. El médico registro indica que fue admitida hace 5 días después de
una caída que causó una fractura de cadera que requirió corrección urológica El
paciente desarrolló dolor de pecho postoperatorio y se transfirió a la servicio médico
para mayor cuidado. Ella está estabilizada y recibe morfina por su dolor, pero
frequentemente llama a las enfermeras para decirles que se siente incómoda. De
acuerdo con la enfermera, el paciente desarrolló un dolor de cabeza intermitente
más temprano en el día que se repitió hace una hora después de que su hija
llamado para decir que no podría visitar hoy. Cuando ve al residente, el paciente
chasquea, "Te ves tan joven. Espero que sepas lo que estás haciendo". El residente
hace algunas preguntas sobre ella dolor de cabeza, pero ella da muy poca
información. Cuando el residente intenta realizar un básico examen físico, el
paciente grita: "No me toques. Me voy a quejar al hospital". administradores sobre
usted porque no tiene idea de cómo tratar adecuadamente a un paciente ". la
siguiente es la respuesta más apropiada?

A. Como quizás sepan, este es un hospital de enseñanza; yo puedo


asegurarle que estoy bien entrenado y puedo consultar a un médico tratante si es
necesario ". 13%]
B. "Lamento que no esté satisfecho con su atención: puede presentar una
queja como se describe en el carta de derechos del paciente. "[0%]
C." Veo que estás molesto, imagino que podrías estar decepcionado porque
su hija no pudo visitar. "12%
D." Veo que estás molesto; usted ¿Prefiere ser visto por mi médico de
cabecera supervisor? "[6%]
E." Entiendo que esté molesto, pero puedo ayudarlo mejor si me permite
continuar con el examen y hacer un diagnóstico ". [13%]
F. "¿Puedo preguntar qué te molesta para poder ayudarte lo mejor que
pueda?" [73%] probablemente irritable, enojado y verbalmente abusivo con el
médico residente. Aunque el inmediato ira, cólera no está clara, su curso médico
complicado, incomodidad general y decepción que su hija no visitó probablemente
contribuya a su angustia. Aunque y desafiante, es responsabilidad del médico ser no
defensivo, desactivar al paciente intento de tiempo de bloqueo preservar la relación
médico-paciente. Reconociendo que el paciente 92065249 no es defensivo, y hacer
preguntas abiertas con respecto a lo que está molesto

Un hombre de 53 años se queja de un temblor ocasional de su mano derecha


durante los últimos 3 meses. Él con frecuencia nota el temblor mientras descansa
en un sillón y mira la televisión. A menudo se detiene cuando alcanza el control
remoto para cambiar el canal. La sacudida se vuelve más pronunciada cuando el el
paciente se dedica a tareas mentales. Toma café y ocasionalmente toma una copa
de vino en cena; estos no tienen ningún efecto sobre el temblor. Su mano izquierda
no tiene síntomas. El paciente no tiene debilidad, entumecimiento, parestesias o
disfunción de la marcha. No hay otro pasado médico importante historia y él no toma
medicamentos. ¿Cuál de los siguientes es el responsable más probable de esto?
¿la condición del paciente?
A. Disfunción de los ganglios basales [63%]
B. Disfunción cerebelosa
C. Lesión del tracto corticoespinal
D. Temblor esencial
E. Temblor ortostático
F. Neuropatía periférica
G. Temblor fisiológico

Una mujer de 52 años con cirrosis hepática conocida llega al servicio de


urgencias con un sangrado de color rojo brillante por el recto durante 2 días. Ella
también tiene una creciente distensión abdominal y malestar. el paciente ha estado
tomando diariamente furosemida y espironolactona, así como restricción de agua y
sodio. Tenía banding de varices después de un episodio de hemorragia esofágica
debido a varices esofágicas 3 meses hace. La temperatura del paciente es 36.5 C
(97.7 F), la presión arterial es 91/55 mm Hg, el pulso es 116 / min, y las
respiraciones son 18 / min. Su abdomen está distendido y no sensible con
desplazamiento positivo embotamiento y tiene sonidos intestinales normales. Hay 2
edemas por picadura periférica. Los resultados de laboratorio son como sigue:
Hemoglobina 8.2 g / dL Leucocitos 4.200 / mm Suero sérico 128 mEq / l. Potasio
sérico 5.5 mEq / L Cloruro sérico 103 mEq / l nitrógeno ureico en sangre 72 mg / dl
creatinina sérica 2,1 mg / dl después de la sangre transfusión y celebración de
diuréticos por un día, la producción de orina del paciente disminuye a pesar
resucitación del volumen intravenoso adecuado. La tira reactiva de orina es negativa
para proteínas y sangre. Orina el sodio es de 5 mEq / l. ¿Cuál de las siguientes es la
causa más probable del riñón agudo de este paciente? ¿lesión?

A. Enfermedad glomerular
B. Nefritis intersticial
c. uropatía obstructiva
D. Hipoperfusión renal
E. Trombosis de la vena renal

Una mujer de 37 años acude al servicio de urgencias por falta de aliento y de lado
izquierdo dolor en el pecho que comenzó hace 2 días. Los síntomas comenzaron
repentinamente y empeoraron a pesar del uso de acetaminofén de venta libre. Ella
no ha sido febril. El paciente fue diagnosticado con esclerosis múltiple hace 5 años.
Ella usa una silla de ruedas debido a paraparesia espástica y tiene movimientos
sacádicos habla. La temperatura es 36.7 C (98 F), la presión arterial es 120/70 mm
Hg, el pulso es 110 / min, y respiraciones son 18 / min. La oximetría de pulso es
94% en aire ambiente. El examen pulmonar es normal, excepto para sonidos
reducidos de la respiración y embotamiento a la percusión en la base del pulmón
izquierdo. El abdomen es suave y nontender. No hay edema periférico Los
resultados de laboratorio son los siguientes: Hemograma completo Leucocitos mm
Neutrófilos 80% Linfocitos 20% Radiografía de tórax revela un pequeño pleural
izquierdo efusión. No se ven infiltrados. ¿Cuál de las siguientes es la causa más
probable de este paciente? efusión pleural?

A. Neumonía por aspiración


B. Insuficiencia cardíaca congestiva
C. Hipoalbuminemia
D. Derrame maligno
E. Embolia pulmonar [56%] este paciente joven con inmovilización
prolongada presenta dolor de pecho repentino, disnea y taquicardia que sugiere una
embolia pulmonar Los pacientes con EP desarrollan comúnmente pleural pequeña
derrames debido a la inflamación de la hemorragia. Las efusiones tienden a ser
exudativas y extremadamente sangrientas, y pueden asociarse con dolor debido a la
irritación pleural. Los pacientes con sospecha de EP deben obtener pruebas de
confirmación (por ejemplo, angiografía pulmonar por tomografía computarizada).
Un hombre de 32 años de edad es intubado y ventilado mecánicamente después
de una sobredosis de opioide. Él es sedado y pasivo El ventilador activa 14
respiraciones por minuto, cada una con 500 ml de marea volumen a un flujo máximo
de 60 LAmin. Una retención inspiratoria se realiza durante 2 segundos después
entrega del volumen tidal, y la presión de la vía aérea se mide a 30 cm H20
Asimiento inspiratorio Presión de 30 mmH20 Tiempo ¿Es muy probable que la
presión medida refleje cuál de los siguientes?
A. Resistencia de la vía aérea superior
B. Resistencia de la vía aérea total
C. Conformidad pulmonar [51%]
D. Resistencia muscular espiratoria
E. Presión espiratoria final

Explicación: Medida de las presiones de la vía aérea pueden ser útiles en pacientes
ventilados mecánicamente. La presión pico de la vía aérea (la presión máxima
medida a medida que se está entregando el volumen tidal) es igual a la suma de la
resistencia presión (flujo x resistencia) y la presión de meseta. Presión resistiva de
presión de la vía aérea máxima.

Una maestra jubilada de 54 años acude al médico debido al empeoramiento de la


lumbalgia. El dolor comenzó hace 3 semanas. Es continuo, no mejora con el
descanso y es peor por la noche. los el paciente ha tenido poco alivio con
analgésicos no esteroideos de venta libre. Él no tiene otro síntomas. Él tuvo una
resección quirúrgica de un tumor de pulmón hace 1 año para el carcinoma de
células no pequeñas de el pulmón. En ese momento, la tomografía de emisión de
positrones (PET) preoperatoria no reveló ninguna evidencia de metástasis Sus
signos vitales están dentro de los límites normales. El examen muestra la columna
vertebral local ternura en el nivel L4-L5. ¿Cuál es la causa más probable de este
paciente dolor de espalda?

A. Espondilitis anquilosante [1%]


B. Estenosis del canal medular central [1%]
C. Hernia discal 13%)
D. ac trombosis arterial (0%)
E. cepa lumbar [2%]
F. enfermedad metastásica 78%
G. osteoporótica fractura por compresión (9%)
H. osteomielitis vertebral [1%]

Un hombre de 32 años de edad es intubado y ventilado mecánicamente después


de una sobredosis de drogas opioides. Lo es sedada y pasiva. El ventilador activa
14 respiraciones por minuto, cada entrega 500 mL de marea volumen con un caudal
pico de 60 LAmin. Un asimiento inspiratorio se realiza durante 2 segundos después
de entrega del volumen tidal y la presión de la vía aérea se mide para ser 30 cm
H20 inspiratorio mantenga ¿Presión de 30 tiempo mmH20 la presión medida
probablemente refleja cuál de las siguientes? (repetida)
A. Vía aérea superior [10%]
B. vía aérea Total B.
C. pulmonar
D. Cumplimiento de normas [51%]
E. D. Fuerza de músculo espiratorio
F. E. final de la espiración la presión [12%]

Maestra jubilada de 64 años viene al médico debido a empeoramiento de dolor de


espalda. El dolor Iniciado hace 3 semanas. Es continuo, no mejora con el reposo y
es peor por la noche. El paciente ha tenido poco de alivio con analgésicos
antiinflamatorios no esteroides de venta libre. No tiene ningún otro síntomas. Él
tenía una resección quirúrgica de un tumor de pulmón carcinoma de células no
pequeñas de hace 1 año el pulmón. En aquel momento, preoperatorio emisión del
positrón (animal doméstico) análisis no reveló ninguna evidencia de metástasis. Sus
signos vitales están dentro de límites normales. El examen muestra local espinal
sensibilidad en el nivel L4-L5. ¿Cuál es la causa más probable de este paciente
dolor de espalda? (repetida)

A. Espondilitis anquilosante [1%]


B. canal Central espinal estenosis [1%]
C. hernia discal 13%]
D. ac trombosis de la arteria (0%]
E. tensión Lumbar [2%]
F. metástasis [78%]
G. osteoporosis compresión de la fractura (9%]
H. osteomielitis vertebral [1%]

Una mujer de 68 años acude al servicio de urgencias debido a la aparición


repentina de dolor en el pecho asociado con 2 episodios de vómitos. Historial
médico pasado incluye hipertensión por la cual ella toma hidroclorotiazida. La
presión arterial es 80/50 mm Hg, el pulso es 60 / min y las respiraciones son 14
lmin. El examen muestra diaforesis, presión venosa yugular elevada y un signo
positivo de Kussmaul. Sus pulmones están limpios para la auscultación. El ECG
muestra una elevación del segmento ST de 2 mm en las derivaciones ll, ll y avF y
depresión del segmento ST de 1 mm en las derivaciones l y avL. ¿Cuál de las
siguientes es la causa más probable? de la hipotensión de este paciente?

A. Rotura del tabique interventricular


B. Volumen intravascular agotamiento
C. embolia pulmonar
D. infarto ventricular derecho 80%
E angina variante

Explicación: la presentación de este paciente es consistente con aguda inferior


Infarto de miocardio con elevación del ST (STEMI) con infarto de miocardio del
ventrículo derecho (RVMI). La RVMI se observa en el 30% -50% de los pacientes
con IAM agudo debido a la oclusión de la arteria coronaria derecha. proximal al
origen de las ramas del VD Los pacientes suelen tener dolor en el pecho, signos
autonómicos vómitos) y los hallazgos ECG de ST- cuya elevación en las
derivaciones inferiores ll, III y avF. En adición, pueden tener una vena yugular aguda
que sugiera el inicio de la VD junto con un pulmón claro campos, MI inferior debido a
la falla. También puede ocurrir bradicardia transitoria o bloqueo atrioventricular con
tono vagal mejorado con nitroglicerina. La hipotensión puede empeorar mucho
después de la terapia. los El diagnóstico de RVMI se confirma con una elevación del
segmento ST de 21 mm en el precordial derecho 4R-v6R. La falla Rv conduce a la
precarga y, por lo tanto, a la terapia de MI estándar, lo que resulta los pacientes (sin
congestión pulmonar) generalmente se tratan con bolos de (líquidos de Choice)
(solución salina sotónica) para mejorar la precarga Rv y facilitar el llenado del
ventrículo izquierdo A) Interventricular ruptura del tabique puede conducir a un
tourinh izquierda.

Una mujer de 25 años acude al médico con un historial de 3 meses de pérdida


de peso, insomnio irritabilidad, y palpitaciones. Su historial médico pasado es
insignificante. Ella no está tomando ningún medicamento. El paciente no usa
tabaco, alcohol o drogas ilícitas. La presión arterial es 155/70 mm Hg y el pulso
es 110 / min. El examen físico muestra la retracción de la tapa, la piel seca y el
temblor fino de las manos. ¿Cuál de las siguientes es la causa más probable de
la hipertensión de este paciente?

A. Nivel de calcio sérico elevado 12%]


B.Aumento de la contractilidad miocárdica [37%]
C.Aumento resistencia vascular periférica
D.Aumento de la síntesis de catecolaminas [35%]
E. Nocturno Insuficiencia respiratoria hipercápnica

Un hombre de 34 años está siendo evaluado por una posible enfermedad renal en
etapa terminal. Él tiene una larga historia o diabetes, tipo 1. Previamente desarrolló
insuficiencia renal crónica a pesar de estar con enalapril e insulina. Su función renal
está empeorando día a día. nefrólogo está actualmente manejando su condición
renal ¿Cuál de los siguientes tratamientos a largo plazo brindaría la mejor tasa de
supervivencia? para este paciente?

A. Hemodiálisis 15%
B. Diálisis peritoneal 1%
C. El trasplante renal de un cadáver [1%]
D. Trasplante renal de un donante vivo relacionado 89%
E. Trasplante renal de un donante vivo no relacionado 12%

Explicación: La enfermedad renal en etapa terminal es una enfermedad


progresiva condición que es fatal si no se trata. Una vez que se desarrolla la
enfermedad renal en el estadio final, solo hay dos opciones de tratamiento
disponibles: diálisis o trasplante renal. La elección depende de condiciones de
pacientes y comorbilidades; Sin embargo, si ambas opciones están disponibles, el
trasplante renal es preferido, ya que se asocia con una mejor supervivencia y
calidad de vida Las ventajas de renal el trasplante durante la diálisis es una mejor
supervivencia y calidad de vida 2. Anemia, enfermedad ósea y la hipertensión
persiste a pesar de la diálisis; estos están mejor controlados con el trasplante 3. Los
pacientes de trasplante tienen un retorno de las funciones endocrinas, sexuales y
reproductivas normales, y niveles de energía mejorados; Volviendo al empleo de
tiempo completo y la actividad p más vigorosa somos nosotros, posible. 4. En
diabéticos, la neuropatía autonómica persiste o empeora después de la diálisis;
mientras que, se estabiliza o mejora con el trasplante 5. La tasa de supervivencia
esperada después del trasplante es del 95% en un año y 88% en cinco años Las
principales desventajas del trasplante son dificultad en encontrar un donante, riesgo
quirúrgico y costo nd inmunosupresión. Trasplante de una vida relacionada el
donante tiene la menor injección y el mejor donante no relacionado vivo, y el injerto
de cadáver.

Una mujer de 84 años es llevada al departamento de emergencia debido a 2


semanas de tratamiento progresivo Confusión. Ella tiene un historial de demencia
leve y vive en un centro de vida asistida. El cuidador informa que durante los últimos
meses se ha vuelto progresivamente más débil, y ella se ha caído en varias
ocasiones y ahora está usando un andador. En su estado habitual, el paciente
reconoce a la mayoría del personal, conversa apropiadamente y realiza actividades
diarias con un mínimo asistencia; sin embargo, durante la última semana, ella ha
estado más confundida y durmiendo la mayor parte del tiempo. Ella también ha
desarrollado un leve dolor de cabeza. El paciente no ha tenido fiebre, vómitos ni
problemas urinarios síntomas. Sus otros problemas médicos incluyen hipertensión y
osteoartritis. La presión arterial es 138/76 mm Hg y el pulso es 74lmin y regular. Ella
es somnolienta pero excitable. El paciente no reconocer a su cuidador y da varias
respuestas inexactas, pero es capaz de seguir simples instrucciones. La fuerza
muscular es 4/5 en el lado derecho y 3/5 en el izquierdo. El reflejo plantar es positivo
a la izquierda. ¿Cuál de los siguientes es el diagnóstico más probable?

A. progresión de la demencia 18%]


B. Infarto de cápsula interna [16%]
C. Encefalopatía metabólica [7%]
D. Presión normal hidrocefalia [3%]
E. paresia postictal [0%]
F. hematoma subdural (63%)

Un hombre de 53 años acude al departamento de emergencias quejándose de la


repentina aparición de intensos dolor epigástrico. También vomitó una vez y notó un
abdomen repentino, sordo, adolorido y difuso dolor. Él ha tenido dolor abdominal
inespecífico episódico durante los últimos meses. Abdominal La ecografía realizada
hace 2 semanas mostró cálculos pequeños en la vesícula biliar. El paciente es otro
los problemas médicos incluyen estreñimiento, diabetes mellitus tipo 2 e
hiperlipidemia. El tiene un 45-pack-year fumando historia y bebe 1 o 2 cervezas
diariamente Su temperatura es 38.0e C (100.4 ° F), la presión arterial es de 160/95
mm Hg, el pulso es de 110 / min y las respiraciones son de 26 / min. El examen
muestra un abdomen difuso sensible con rebote Hay dolor incrementado en la parte
superior del abdomen cuando se palpa después de inspiración profunda, pero no
hay protección o masa palpable. El examen rectal muestra una bóveda vacía. La
radiografía de tórax vertical se muestra a continuación. UPRIG ¿Cuál de los
siguientes es el más probable diagnóstico en este paciente?

A. Colecistitis aguda [10%]


B. Pancreatitis biliar aguda (13%)
C. Disección aórtica 12%)
D. Cólico biliar [1%]
E. Úlcera péptica perforada 72%
F. Neumotórax [0%]
Un hombre sin hogar de 34 años de edad es traído a urgencias en un estado
confuso. Él se queja de dolor epigástrico, visión borrosa y vómito. Su temperatura es
de 36,8 ° c. (98,2), presión arterial es de 110\/60 mm Hg, pulso es 110 por minuto y
respiraciones son 22\/min el examen físico muestra oral pobre higiene y seca las
membranas mucosas. Los pulmones están claros a la auscultación. Examen
abdominal muestra leve malestar epigástrico. La examinación funduscopia revela
hiperemia del disco óptico. Estudios de laboratorio Ver suero sodio 136 mEq\/L
suero potasio 3.0 mEq\/L. cloruro 93 mEq\/L bicarbonato 6 mEq\/L sangre creatinina
del suero de 30 mg\/dL de urea nitrógeno (BUN) 0,8 mg\/dL. Glucemia 80 mg\/dL.
Cetonas de suero ¿negativo el Plasma lactato 2.2 mEq\/L, cuál de las siguientes es
la causa más probable de sus síntomas?

A. el glicol de etileno envenenamiento [24%]


B. isquemia mesentérica [1%]
C. shock séptico [0%]
D. diabética cetoacidosis [1%]
E. metanol envenenamiento (68%)
F el. aspirina sobredosis 13%]

Un hombre de 60 años llega a la oficina debido a una disfunción eréctil que ha
progresado lentamente el año pasado. Le resulta difícil lograr una erección y ha
notado una disminución en la nocturnidad erecciones El historial médico es
significativo para la diabetes mellitus tipo 2 y la próstata benigna
hiperplasia evaluación oftalmológica hace 6 meses reveló retinopatía diabética que
fue tratada con fotocoagulación. El paciente también tuvo una prueba de esfuerzo
cardíaco negativa en la cinta de correr hace 4 meses después de presentar
molestias torácicas atípicas. Los medicamentos actuales incluyen metformina,
sitagliptina, rosuvastatina y doxazosina. La presión arterial es 122/70 mm Hg y el
pulso es 76 / min. El examen cardiopulmonar no es notable excepto por pulsos de
pedal ligeramente disminuidos a la derecha. Hay una disminución de la sensación
de vibración en ambos pies. La hemoglobina A1c medida hace 2 semanas fue del
8%. El paciente solicita una receta para sildenafil. ¿Cuál de los siguientes es el más
importante? preocupación en la prescripción de sildenafil a este paciente?

A. Retinopatía diabética [4%]


B. Interacción con doxazosina [74%]
C. Interacción con metformina [1%]
D.Neuropatía periférica [1%]
E. Periférico enfermedad vascular 18%]

Una mujer de 24 años llega a la oficina después de 6 meses de dolor ardiente en


la mano derecha. El dolor es leve e intermitente, pero peor por la noche, y se siente
principalmente en el primero, segundo y tercer dígitos de la mano. Estrechar las
manos o dejarlas colgando brevemente alivia el dolor. Ahi esta sin dolor asociado
en el cuello o el antebrazo. El paciente intentó el tratamiento con over-the-counter
analgésicos y una "crema para la artritis" tópica sin alivio. Su historial médico es
notable por enfermedad renal en etapa terminal, por la cual el paciente se sometió a
un trasplante de donante vivo hace 2 años. Ella no usa tabaco, alcohol o drogas
ilícitas. Los signos vitales son normales. En el examen, hay no hay atrofia tenar o
hipotenar y la oposición del pulgar es normal. Percusión sobre el aspecto volar de
la muñeca reproduce el dolor. ¿Cuál de los siguientes es el siguiente paso más
apropiado en manejo de este paciente?

A. Cirugía de descompresión
B. Indometacina
C. Local inyección de glucocorticoides
D. glucocorticoides orales
E. Férula de muñeca [75%]

Un hombre caucásico de 73 años es llevado a la oficina por su hija, a quien le


preocupa que él podría estar deprimido Es un cirujano retirado, y ha vivido solo
desde que su esposa murió un año hace. Su hija lo visita cada 6 meses; ella se
siente mal por no poder visitarlo más con frecuencia porque su trabajo y su familia la
mantienen ocupada. Él niega tener sentimientos de tristeza, culpabilidad, pérdida de
peso, pérdida de apetito, ideación suicida, sordera, vértigo y disminución o visión
borrosa. Sus problemas médicos incluyen hipertensión, diabetes mellitus tipo 2 y un
miocardio infarto 10 años es medicamentos actuales son gliburida, aspirina y
enalapril. Él niega el uso de alcohol de tabaco, o drogas. Sus signos vitales están
dentro de los límites normales. Parece retirado, menos enérgico que y camina
rígidamente. Él se sienta con una postura encorvada. Él tiene una expresión facial
fija, y su voz suena monótona. Los reflejos tendinosos profundos son 2.
Sensaciones y la fuerza del motor son normal. Hay una mayor resistencia a la
flexión pasiva. ¿Cuál de los tipos de marcha es más probable que ser el siguiente
presente en este paciente?

A. Ataxia cerebelosa 11%]


B. Marcha hipocinética [61%]
C. Waddling Walk [7%]
D. Spistic step (6%)
E. Gait desequilibrio [1%]
F. Ataxia sensorial [0% O
G. Ataxia vestibular (0%)
H. Andar de base ancha [16%] O
I. Andar distónico 13%

Explicación: La mayoría el diagnóstico probable de este paciente es parkinsonismo.


Los signos distintivos de esta enfermedad incluyen una especie de máscara,
bradykinesia inmóvil, temblor, rigidez y marcha. Hay una desaceleración general de
toda actividad motora (marcha hipocinética). Los pacientes asumen una postura
encorvada y caminan en pasos cortos y acelerados, tales que el piso de raspado La
ID de Prueba restante: y no se balancee durante la caminata. El subyacente la
patología implica la degeneración del bloqueo n de la sustancia negra, lo que
conduce a la disminución actividad dopaminérgica .

Un hombre con sobrepeso de 42 años de edad está siendo evaluado para su
segundo episodio de vena profunda trombosis en 2 años. Ambos episodios parecen
haber sido no provocados. Él no tiene una prolongada reciente inmovilidad, viajes
de larga distancia o traumatismos en las extremidades inferiores. El paciente no
tiene antecedentes médicos historial de diabetes, cáncer o enfermedad hepática. Se
ordena un estudio de trombofilia. Uso de cual de las siguientes drogas es más
probable que conduzcan a bajos niveles de proteína S?
A. Aspirina [0%]
B. Clopidogrel [1%]
C. Heparina 4%
D. Simvastatina 12%
E. Warfarina 90%.

Una mujer de 38 años acude a la consulta con debilidad muscular progresiva en


los últimos 6 meses. Le cuesta subir escaleras y peinarse, y no puede ni siquiera
caminar distancias sin descanso. No hay dolor muscular asociado. El paciente fue
diagnosticado con hipotiroidismo hace un año cuando comenzó a aumentar de peso
y ha estado tomando levotiroxina desde entonces. Hace cuatro meses, ella
desarrolló dolor de espalda agudo. La radiografía en ese momento reveló
desmineralización de los huesos vertebrales. La temperatura es 37 C (98.6 F), la
presión arterial es 155/100 mm Hg, el pulso es 76 / min, y las respiraciones son 15 /
min. El IMC es 32 kg / m2. El examen físico muestra hirsutismo facial y debilidad
muscular proximal leve en las extremidades. Los músculos del paciente no son
sensibles palpación, y no se observan fasciculaciones. Los reflejos tendinosos
profundos y el examen sensorial son normal. TSH es normal. ¿Cuál de los
siguientes es más probable responsable de este paciente? ¿debilidad muscular?

A. Anormalidad del electrolito [31%]


B. Disfunción mitocondrial [4%]
C. Motor enfermedad neuronal (8%)
D. atrofia muscular [30%]
E. inflamación muscular [21%]
E. Nervio periférico desmielinización [4%]

Un hombre de 60 años llega a la oficina después de que se descubrió que tenía
un "recuento sanguíneo bajo al intentar para donar sangre El paciente se encuentra
en su estado de salud habitual y se siente bien. Él no tiene crónico problemas
médicos, pero ha sido tratado por neumonía dos veces durante el año pasado. El
paciente toma sin medicamentos y no usa tabaco ni alcohol. La temperatura es 37 c
(98.6 F), la presión arterial es 120/80 mm Hg, y el pulso es 76 / min. La exploración
física revela una mucosa normal. No hay una ampliación. ganglios linfáticos El
abdomen es suave y no sensible. La punta del bazo es palpable 5 cm por debajo de
la izquierda margen costal: el hígado es de tamaño normal. El guayaco fecal es
negativo. Resultados completos de conteo de sangre son los siguientes: 9.6 g / dL.
Hemoglobina Volumen corpuscular medio 83 fL 130,000 / mm3 Plaquetas 14,000 /
mm Leucocitos Frotis de sangre periférica muestra linfocitos predominantes
¿Cuál de los siguientes es el diagnóstico más probable?

A. linfoblástica aguda leucemia [1%]


B. Leucemia linfocítica crónica [15%
C. Enfermedad de Gaucher [0%]
D. Leucemia de células pilosas 74%
E. Linfoma de Hodgkin (0%
F. Mononucleosis infecciosa [1%]
Un hombre de 65 años acude al servicio de urgencias con dolor de pecho
subesternal, falta de sensibilidad severa respiración y diaforesis que comenzaron
repentinamente hace 40 minutos. El dolor se irradia a su brazo izquierdo y no remite
con nitroglicerina sublingual. El paciente ha vomitado dos veces desde que
comenzó el dolor. El electrocardiograma muestra aumentos de ST de 2 mm en las
derivaciones anteriores. En el examen físico, el la temperatura del paciente es 36.9
C (98.4 F), la presión arterial es 110/70 mm Hg, el pulso es 60 / min, las
respiraciones son 32 / min, y la saturación de oxígeno es del 90% con 4 l / min de
oxígeno por cánula nasal. Cardíaco el examen muestra un S3. El examen pulmonar
muestra crepitantes basilares que se extienden hasta la mitad del pulmón campos
bilateralmente Se le administran 325 mg de aspirina, 600 mg de clopidogrel y 80 mg
de atorvastatin. ¿Cuál de los siguientes es el mejor paso siguiente en el manejo de
este paciente?

A. Amlodipina [1%
B. Digoxina 12%]
C. Furosemida [55%]
D. Solución salina normal intravenosa (4%)
E. Metoprolol [30%]
F. Espironolactona (4%
G. Verapamil [1%

Una mujer de 54 años acude a la consulta como un paciente nuevo. Ella emigró a
los Estados Unidos 3 Hace semanas para el asilo político. El paciente tiene
antecedentes de hipertensión y diabetes tipo 2 mellitus Ella también fue
diagnosticada con tuberculosis pulmonar activa hace 4 meses. Ella completó 2
meses de terapia antituberculosis intensiva con 4 medicamentos y ahora está en la
isoniazida y la rifampicina solo. Repita las pruebas de esputo para el bacilo ácido-
ácido son negativos. El paciente se siente bien en general pero tiene hormigueo y
entumecimiento de manos y pies bilaterales que comenzaron hace algunas
semanas; no tiene tos, fiebre, escalofríos ni sudores nocturnos. Ella no usa tabaco,
alcohol o drogas ilícitas drogas. La temperatura es 37 C (98.6 F), la presión arterial
es 126/84 mm Hg, y el pulso es 84 / min. Pulmón la auscultación revela finos
crepitantes en el campo pulmonar superior izquierdo. Los sonidos cardíacos son
normales y regular. El examen neurológico muestra una fuerza motriz normal pero
disminución del tacto y la sensación de dolor en las extremidades superiores e
inferiores bilaterales, el signo de Romberg es positivo. el examen de la piel no
muestra anormalidades. El nivel de hemoglobina A1C es del 7%. La radiografía de
tórax revela cambios fibróticos en la parte superior izquierda. pulmón. ¿Cuál de las
siguientes es la causa más probable de los síntomas actuales de este paciente?

A. Degeneración de las columnas vertebrales dorsal y lateral [6%]


B. Desmielinización inflamatoria de los axones [1%]
C. Pérdida de neuronas motoras [0%]
D. Efecto adverso a la medicación [89%]
E. Lesión del nervio microvascular 12%]
F. Paraproteinemia neuropatía [0%]

Explicación: La tuberculosis activa se trata típicamente con 2 meses de isoniazida


(INH), rifampicina, los efectos son pacientes y la transición a varios meses de INH y
común con el tratamiento, especialmente rifampina sola. Drogas INH-inducida
inducida por INH hepatotoxicidad y neuranasa periférica periférica

Una mujer de 47 años llega a la oficina debido a 2 días de falta de aire y pecho
derecho dolor. Ella también tiene un dolor de espalda progresivo que recientemente
ha limitado su movilidad. El paciente tiene un antecedentes de cáncer de mama
tratado con quimioterapia neoadyuvante y mastectomía hace 6 meses. Su los
medicamentos incluyen tamoxifeno y un multivitamínico. La temperatura es 38.1 C
(100.6 F), presión arterial es 100/60 mm Hg, el pulso es 110 / min, y las
respiraciones son 18 / min. Ella no está angustiada. Existen disminución de los
sonidos de la respiración en la base derecha. El examen neurológico muestra una
fuerza normal y reflejos en sus extremidades superiores e inferiores bilaterales.
Radiografía de tórax muestra un infiltrado que oscurece el borde derecho del
corazón y un derrame pleural del lado derecho. La toracocentesis se realiza; pleural
los resultados de fluidos son los siguientes: Células nucleadas mm Glóbulos rojos
1500 / mm Glucosa 38 mg / dL Proteína 3.9 g / dL. pH 6.9 Lactato deshidrogenasa
620 U / L Las químicas séricas son notables para un lactato deshidrogenasa de 300
U / L y proteína total de 6.1 g / dL. ¿Cuál de los siguientes es el más probable causa
de su condición?

A. Lupus inducido por fármacos 12%]


B. Insuficiencia cardíaca 13%)
C. Hipoalbuminemia 13%)
D. neumonía 74%
E. Embolia pulmonar [15%]

Se realizó un estudio prospectivo de cohortes para evaluar los efectos a largo
plazo de una dieta alta en grasas en la incidencia de cáncer de colon Los
participantes del estudio fueron seleccionados al azar de la población de interesar.
Los patrones dietéticos se evaluaron mediante el uso de cuestionarios
autocompletados periódicos. Los investigadores informaron un riesgo relativo de 5
años de 1.60 para las personas que consumieron una dieta alta en grasas en
comparación con las personas que consumieron una dieta baja en grasas. El
intervalo de confianza del 95% fue de 1.02 a 2.15 ¿Es más probable que este
estudio tenga cuál de los siguientes valores de p?

A. 0.04 [85%]
B. 0.06 [5%]
C. 0.09 13%]
D. 0.11 12%]
E. 0.20 13%]

Explicación: El riesgo relativo (RR) es la probabilidad de que resultado de interés


que ocurre en el grupo expuesto en comparación con la probabilidad de que ocurra
en el grupo no expuesto. El valor nulo del RR es 1.0; un RR de 1.0 significa que se
produce el resultado con la misma frecuencia en ambos grupos y que no hay
asociación entre la exposición y el Salir. Un RR 1.0 significa que el resultado ocurre
con más frecuencia en el grupo expuesto (positivo asociación). El RR no dice nada
sobre la significación estadística de un estudio. La estadística puede ser expresado
con valores de p o intervalos de confianza, pero ambos están nulos eufóricos. Por
ejemplo, p corresponde a un intervalo de confianza del 95% que no contiene el
valor. Del mismo modo p es equivalente a un intervalo de confianza del 99% que no
contiene el alor nulo. Por el contrario, si el valor nulo es dentro de un intervalo de
confianza dado, entonces el valor de p es el intervalo de confianza equivalente de
0.01. 95% de confianza 99% de confianza Valor nulo fuera de la confianza intentnl
ID de la prueba.

Un hombre de 38 años acude a la consulta para una visita de seguimiento


después de estudios de laboratorio anuales de rutina mostró una prueba positiva de
VIH Actualmente está asintomático y nunca ha tenido una enfermedad relacionada
con el VIH El paciente cree que adquirió la infección a través de relaciones sexuales
sin protección. Tratamiento no se ha iniciado debido a resultados pendientes de
resistencia a los medicamentos antirretrovirales. El paciente no tiene otros
problemas médicos. Inmigró a los Estados Unidos hace 15 años y no sabe si
recibió todas sus vacunas infantiles. El paciente recibió tétanos, difteria, tos ferina y
vacunas contra la hepatitis B durante su visita previa a la consulta. Los signos
vitales están dentro de los límites normales, y el examen físico no tiene nada
especial. El recuento de CD4 es de 500 / mm3 y la carga viral plasmática es de
42,000 copias / mm3 La prueba del título de anticuerpos muestra inmunidad contra
sarampión, paperas y rubéola, pero bajos títulos de anticuerpo antivaricella. ¿Cuál
de las siguientes es la recomendación más adecuada con respecto a una vacuna
viva de varicela para este paciente?
A. No administre la vacuna tal como es contraindicado en pacientes con VIH
[21%]
B. No administrar la vacuna ya que no es necesaria para adultos. [1%]
C. Administre la vacuna una vez que comience la terapia
antirretroviral. [11%]
D. Administrar la vacuna si el recuento de células CD4 cae por debajo de 200
/ mm3.
E. La administración de la vacuna ahora es recomendado. [64%]

Explicación: La vacuna contra la varicela contiene virus vivos atenuados. La


vacunación produce una infección por virulencia extremadamente baja que
representa un riesgo mínimo para individuos inmunocompetentes pero riesgo
significativo para aquellos que están severamente inmunocomprometidos. Como
por lo tanto, los pacientes con VIH con recuentos y lt; 200 / mms no deben recibir
vacunas vivas atenuadas (varicela, zóster, sarampión, paperas y rubéola
MMR). Aquellos con recuentos de CD4 200 / mm3 son inmunocompetentes
suficiente para eliminar las infecciones atenuadas y deben aplicarse vacunas contra
la varicela y la vacuna triple vírica si los títulos son bajos Todos los pacientes con
VIH deberían recibir las vacunas inactivadas modificadas para el bien general como
aquellos recomendado específicamente para personas con (neumococo, hepatitis B)
(Opciones A y D) Varicela la vacunación está contraindicada solo en pacientes con
VIH con recuentos de cD4 y lt; 200 mm

Un hombre de 48 años acude al servicio de urgencias debido a un dolor de pecho


severo que comenzó durante 2 horas hace. Tiene dificultad para localizar el dolor y
dice que "me duele todo el pecho". Él no tuvo dificultad para respirar, aturdimiento,
síncope o palpitaciones. El paciente no tiene conocimiento médico problemas. Él
es un no fumador de por vida y no usa drogas ilícitas. Su padre murió de repente a
los 52 años de un presunto ataque al corazón. Su temperatura es de 36.7 C (98 F),
la presión arterial es 178/102 mm Hg, el pulso es 105 / min y regular, y las
respiraciones son 16 / min. En el examen físico, el los pulmones están libres de
auscultación y los sonidos cardíacos son normales. El ECG muestra taquicardia
sinusal, voltaje los criterios para la hipertrofia ventricular izquierda y la inversión de
la onda T en las derivaciones V5 y V6. Una imagen de CT de su cofre se muestra a
continuación. Además del control del dolor, ¿cuál de los siguientes es el más
¿farmacoterapia inmediata apropiada para este paciente?

A. Furosemida [1%)
B. Hidralazina [8%
C. Labetalol (66%)
D. Nitroprusiato (13%
E. Activador de plasminógeno tisular [3%)
F. No fraccionado heparina [4%]

Un varón caucásico de 30 años viene a la oficina debido a la debilidad simétrica


de su menor extremidades. También tiene parestesias en sus dedos de los pies y
los dedos y dolor lumbar. El examen neurológico muestra debilidad simétrica,
disminución de los reflejos y la sensación intacta en su extremidades inferiores.
También se observó hipotensión ortostática. Los estudios electrofisiológicos
muestran lentos velocidades de conducción nerviosa. La punción lumbar revela
presión de apertura normal. Muestra de la examinación del CSF pocas células y una
concentración de 90 mg\/dL de proteína. Cuál de los siguientes organismos participa
en ¿la patogenesia de este desorden?

A. Campylobacter jejuni [87%]


B. Chlamydia [4%] C. Shigella 12%]
D. Salmonella 12%]
E. coli (0157: H7) 13%]
Explicación: pacientes de más de dos tercios de Síndrome de Guillain-Barré (GBS)
tiene una antecedente infección respiratoria o gastrointestinal. Campylobacter jejuni
es el precipitado más frecuente de GBS. Otros precipitantes incluyen humana
Herpes virus, micoplasma y Haemophilus influenzae. GBS se presenta con más
frecuencia en pacientes con linfoma, sarcoidosis y lupus eritematoso sistémico.
Infección reciente por el VIH y recientes También han sido asociados con la
infección por VEB. síndrome urémico hemolítico. (opciones B, C y D) Salmonella,
Shigella, Campylobacter Yersinia y Clamidia están implicados en la patogenia de la
artritis reactiva. Objetivo educativo: Campylobacter jejuni es el precipitado más
frecuente de síndrome de Guillain-Barré (GBS).

Una mujer de 38 años acude a la consulta debido a dolor crónico y rigidez a su


alrededor cuello, hombros, espalda baja y caderas. Incluso un esfuerzo menor
empeora el dolor. El paciente también tiene dificultad para dormir y con frecuencia
se despierta en el medio de la noche. Incluso después de una buena noche dormir,
ella siempre se siente fatigada por la mañana. El paciente no tiene debilidad
muscular, fiebre, malestar general, pérdida de peso o sarpullido. Intentó hacer
ejercicios aeróbicos en el agua 3 veces por semana pero no tuvo ningún síntoma
mejora. Ella tiene un historial de síndrome de intestino irritable. Los signos vitales
están dentro de los límites normales. El examen muestra múltiples puntos sensibles
en los puntos de inserción muscular y prominencias óseas. Su músculo la fuerza es
de 5/5 en todas las extremidades, y los reflejos tendinosos profundos son 2 en todas
partes. Sin sensorial las anormalidades son notadas Los resultados de laboratorio
son los siguientes: Hematocrito 43% Leucocitos 7.000 / mm Plaquetas 200,000 /
mm3 Creatina quinasa, suero 60 U / L TSH 3.0 HU / mL Tasa de sedimentación
globular 9 mm / h ¿Cuál de las siguientes es la terapia más apropiada para este
paciente? A. Amitriptilina [71%]
B. Colquicina [0%]
C. Metotrexato 12%]
D. Naproxeno [15%]
E. Oxicodona (0%)
F. Prednisona 19%)

Un hombre de 47 años de edad desarrolla una repentina aparición de dolor


torácico en el esternón y diaforesis durante una reunión en su oficina. Mientras
espera que llegue el personal médico de emergencia, se queja de mareo y deja de
responder. Sus compañeros de trabajo realizan resucitación cardiopulmonar y
recupera conciencia después de 60 segundos. El paciente tiene un historial de
diabetes tipo 2 controlada por la dieta mellitus, hipertensión e hiperlipidemia En el
servicio de urgencias, su presión arterial es 142/88 mm Hg y el pulso es 92 /
min. Electrocardiograma muestra ritmo sinusal normal, ventricular complejos
prematuros, y una elevación del segmento ST de 3 mm en las derivaciones V1-
V3. Cuál es el más probable mecanismo fisiopatológico primario responsable del
episodio sincopal de este paciente?

A. Asístole [12%]
B. Fibrilación auricular 15%]
C. Bloqueo de conducción auriculoventricular [19%]
D. Paroxismo taquicardia supraventricular [8%]
E. actividad eléctrica sin pulso [16%]
F. reentrada arritmias ventriculares 37%]

Explicación: Arritmias ventriculares, incluida la ventricular latidos prematuros,


taquicardia ventricular no sostenida o sostenida y fibrilación ventricular, son bastante
comunes en el período inmediato posterior al infarto de miocardio (IM). La fibrilación
ventricular es la arritmia subyacente más frecuente responsable de un paro cardíaco
repentino en el entorno de MI agudo; más del 50% ocurre dentro de la primera hora
del inicio de los síntomas. La reentrada es la predominante mecanismo responsable
de las arritmias ventriculares en el período inmediato posterior al infarto. los
mecanismo subyacente responsable de las arritmias ventriculares periinfarto varía
de acuerdo con el tiempo transcurrido desde el inicio de MI. Arritmias que ocurren
dentro de los 10 minutos de la oclusión coronaria se conocen como arritmias
ventriculares "inmediatas" o de fase 1a. La isquemia aguda causa heterogeneidad
de conducción con áreas de marcada disminución de la velocidad y activación
retardada, que a su vez predispone a las arritmias reentrantes. Por el contrario,
ocurren arritmias "retardadas" o de fase 1b aproximadamente 10-60 minutos
después del infarto agudo y se cree que es el resultado de una automaticidad
anormal.
Una mujer de 35 años viene a la oficina para el seguimiento en pruebas genéticas
para el síndrome de Lynch. Ella no tiene síntomas o problemas de salud y nunca ha
tenido la cirugía. Una prueba de Papanicolaou fue normal última mes. El paciente
toma un multivitamínico diario y no otros medicamentos. Ella no utiliza
anticoncepción como ella no ha sido sexualmente activa desde la separación de su
esposo el año pasado. El la madre del paciente, edad 54, está recibiendo
quimioterapia para el carcinoma de colon. Su tío materno fue diagnosticado con
cáncer de colon en la edad de 49 años, pero es en la remisión; su primo murió de
colon cáncer en la edad de 36 años. Además de cáncer de colon, el paciente está
en mayor riesgo de que de la ¿siguientes condiciones? A. de mama
carcinoma de
B. carcinoma renal de la célula
C. endometrio el carcinoma
D. carcinoma medulares hepatocelular
E. carcinoma de la tiroides
f el. OG de adenoma pancreático
Feocromocitoma

Un hombre de 30 años viene a la oficina debido a una erupción que ha estado
presente durante 3 semanas. La erupción es asociada a prurito significativo que no
ha mejorado con una crema tópica de antihistamínicos. El síntomas comenzaron
después de que el paciente adquirió un reloj de un vendedor ambulante mientras
estaba de vacaciones. Médicos la historia es irrelevante, y él no está tomando algún
medicamento. El paciente consume alcohol sólo en social oportunidades y no utilizar
tabaco o drogas ilícitas. Temperatura es de 36,9 C (98,4 F), sangre presión es
118/76 mm Hg, pulso es de 64 por minuto y respiraciones son 12/min
cardiopulmonar y exámenes abdominales son normales. No hay ninguna
linfadenopatía cervical o axilar. Piel el examen muestra una erupción localizada en
la muñeca izquierda,Que de la ¿después de los metales en la joyería es más
probable causar los síntomas de este paciente?
A. cobre
B. Oro
C. níquel
D. platino
E. plata

Un hombre de 45 años de edad llega a la oficina con una historia de 6 meses de
recurrente, dolor epigástrico urente y la diarrea. Sus heces son espumosas e
inusualmente fétido y flotan. El paciente ropa ajuste libremente, y él cree que
pueden haber perdido algo de peso. Él ha probado varios medicamentos, como
antiácidos, bloqueadores H-2, inhibidores de la bomba de protones con éxito
moderado. signos vitales son normales. Examen no demuestra ninguna anormalidad
excepto para midepigastric sensibilidad a la palpación profunda. Prueba de las
heces para sangre oculta es positiva, y grasa materia fecal también es positivo.
Endoscopia gastrointestinal revela dos úlceras duodenales y un yeyunal ¿úlcera
cuál de las siguientes es la mejor explicación para este los pacientes con problemas
de absorción de grasa?
A. Deficiencia de enzima pancreática
B.Lesión autoinmune de la mucosa
C.sobrecrecimiento bacteriano
Enzimas pancreáticas
D. inactivación [57%]
E. reducida del ácido de bilis absorción.

Una mujer de 60 años llega al médico con varios meses de la hinchazón de la


extremidad inferior. Su antecedentes es importante para la hipertensión, diabetes
mellitus tipo 2 y hepatitis C infección. La trataron para la tuberculosis hace 10 años.
Su presión arterial es 120/80 mm Hg y el pulso es 90/min el examen físico muestra
simétrico picaduras edema de las extremidades inferiores. El hígado es palpable a
4 cm por debajo del reborde costal y la ascitis está presente. La punta del bazo es
palpable en inspiración profunda. Presión abdominal muestra distensión persistente
de las venas yugulares ¿positivo sugerente de causa cardiaca edema de este
paciente?
A. pulmones B. ascitis C. Reflujo hepatoyugular D.
hepatomegalia (84%] E. esplenomegalia [1%] segundos o reflujo se
produce mediante la aplicación de presión firme y sostenida durante 10-15 en
la parte superior del abdomen. Una respuesta positiva se define por una
elevación sostenida de la presión yugular 3 cm durante la continua
compresión abdominal. El reflujo hepatoyugular no es trastorno particular
pero es un reflejo de un ventrículo derecho falla que no puede acomodar más
bien un aumento en retorno venoso con la compresión abdominal.
Constrictivo, infarto ventrículo derecho, y miocardiopatía restrictiva son las
causas más comunes de un reflujo y positivo. Este paciente historia de la
tuberculosis es sugestivo de pericarditis constrictiva como causa de su
presentación La pericarditis constrictiva es una causa importante de
insuficiencia cardíaca derecha. Los pacientes presentan con elevada presión
f yugular, edema periférico, ascitis y congestión hepática con hepatomegalia y
eventual progresión a la cirrosis (cirrosis cardiaca). Resultados del examen
físico sugestivos de constrictiva incluyen disminución de la presión venosa
yugular elevada o un incremento en la yugular venoso reflujo, signo de
Kussmaul (calcificatio de ricardial de falta de inspiración).

 Mujer 21 años acude a la oficina con 5 meses de fatiga, sudoración excesiva, y


palpitaciones. El paciente presenta un poco sobrepeso y previamente había tenido
éxito en perder peso con restricción calórica y ejercicio. Sin embargo, ella ha
perdido peso durante los últimos 6 meses con un remedio de pérdida de peso sin
receta médica. El paciente no ha tenido ninguna menstruación durante los últimos 3
meses. Ella no es sexualmente activa. Su presión arterial es de 136/70 mm Hg y el
pulso es de 100/min su IMC es de 26 kg/m2. El paciente tiene retraso de tapa pero
ninguna protuberancia del globo ocular. Examen de la tiroides muestra un pequeño
glándula sin nódulos o ternura. Los resultados del laboratorio son los siguientes:
TSH baja T4 libre alta B-hCG negativa absorción de yodo radiactivo por la glándula
tiroides es difusamente disminuida. Cuál de las siguientes ¿probablemente se
encontrarían en este paciente?

A. título de anticuerpo peroxidasa antitiroideo elevada [14%]


B. elevada de eritrocito sedimentación tasa 15%]
D. Elevado TSH receptor anticuerpos título [20%]
E. suero baja nivel de tiroglobulina (45%]

 Un hombre de 84 años de edad llega al Departamento de emergencia con una


historia de 4 meses de disnea que ha sido cada vez peor últimamente. No puede
acostarse plano y duerme en una silla por la noche. Médicos del pasado la historia
incluye hipertensión y tipo 2 mellitus de la diabetes. Lo hospitalizaron con infarto
agudo de miocardio hace 1 año pero no se ha seguido para arriba con un médico
desde entonces. Su medicamentos incluyen aspirina y metformina. La presión
arterial del paciente es de 144/82 mm Hg y el pulso es 98/min y regular. El examen
físico muestra extremidad inferior bilateral, edema de las picaduras. El punto de
máximo impulso se desplaza a la izquierda y un soplo holosistólico se oye en el
pulmón el examen muestra crepitantes bibasilares. Electrocardiograma (ECG)
muestra ritmo sinusal con inespecífica Segmento ST y T cambios de la onda.
Evaluación ecocardiográfica y cateterismo cardíaco son para determinar el índice
cardiaco (IC), la resistencia vascular sistémica (svR) y ventrículo izquierdo volumen
telediastólico (VTDVI). Cuál de las siguientes pautas es más probable que ver en
esto ¿paciente? A.Resistencia de índice cardiaco
B.xtremo ventricular izquierdo
C.vascular sistémica (SVR) diastólica volumen (VTDVI)
12%] [12%) [78%] [2%] Normal Normal [4%]
No está clara la respuesta

Un hombre de 76 años con demencia por infarto múltiple está siendo evaluado
para la tos y fiebre baja. Él fue tratada por pulmonía dos veces en el último año.
Durante los últimos 6 meses, ha tenido dificultad para deglución y en ocasiones
regurgita comida no digerida. El paciente tiene una larga historia de hipertensión y
fibrilación auricular crónica. Su índice de masa corporal es de 22 kg/m2. Su
temperatura es examen 38.5 destaca por aliento fétido y una masa fluctuante en el
cuello izquierdo. La auscultación muestra base crepitaciones en el pulmón derecho.
Radiografía de tórax muestra un infiltrado sin cavitación en el campo más bajo
correcto del pulmón. El paciente es hospitalizado, cultivos de esputo y sangre son
enviado, y se comienzan antibióticos. Cuál de los siguientes es el paso siguiente
más adecuado en su ¿gestión de la?

A. broncoscopia [4 %l
B. contraste esophagrama [63%]
C. gástrica sonda de alimentación respuesta correcta colocación [10%] cuello
D. biopsia de la masa [5%] superior
E. endoscopia gastrointestinal [16%]

La diagnosis subyacente más probable en este paciente es el divertículo de Zenker


(ZD). ZD es más común en pacientes ancianos, particularmente de los hombres. Se
presenta en la parte inferior posterior cervical esófago cerca de tener músculo que
huele mal. Los pacientes pueden quejarse de y a menudo aliento grande, el los
divertículos pueden ser palpables. o acumulación de material en el divertículo. Si
particularmente se produjeron en Esta pacientes con ZD corren el riesgo de
aspiración p como paciente un esophagram del contraste, que claramente muestra
el divertículo, es la prueba diagnóstica de elección. Administración de contraste oral
pacientes con antecedentes de aspiración se asocia con un riesgo de umonitis. Sin
embargo, el riesgo puede reducirse por un número de técnicas de posicionamiento y
alize indicado se ZD a menos que el paciente tiene severa dificultad para deglutir.
Además, otras opciones a (opción del esófago, incluyendo endoscopia, se asocian a
riesgos muy graves de la perforación del esófago de las complicaciones. ZD es
generalmente reparados quirúrgicamente.

 Una mujer de 38 años de edad es traída al servicio de urgencias por su hermana


después de desarrollar agudo ansiedad, dolor de cabeza, náuseas y vómitos al
comer una cena de mariscos. El paciente no tiene antecedentes migraña,
hipertensión o diabetes. Ella no tiene alergias conocidas pero tienen una historia de
alergias tratada con difenhidramina. El paciente tiene una larga historia de depresión
mayor desorden y ataques de pánico, y ha tenido mala respuesta a los varios
antidepresivos. Durante los últimos año, ella ha sido tratada con una alta dosis de
fluoxetina, que ella continuó hace 2 semanas. Ayer, ella comenzó a tomar fenelzina
para síntomas depresivos. A su llegada, el paciente es agitado, desorientado un
115lmin y respiraciones son 24/min el examen físico muestra dilatada alumnos y
temblores bilaterales. Los reflejos tendinosos profundos son 3 bilateral. Cuál de las
siguientes es ¿la causa más probable de estos hallazgos?

A.Interacción de fármacos anticolinérgicos


B. toxicidad [6%]
C. fluoxetina retirada síndrome 53%
E. IgE-mediada de interacción [34%]
D. alimentos-medicamentos ataque de pánico de
F. alergia [0%]

 Un hombre de 49 años acude al médico con una historia de fiebre de 3 semanas,


También informa dolores musculares. No tiene tos ni dificultad para respirar. El
paciente tiene un año de fumar 15-pac historia. Su temperatura es de 38.9 C (102
F), la presión arterial es de 120/76 mm Hg, el pulso es de 90 / min y respiraciones
son 16 / min. Los pulmones son claros para la auscultación. El resto del examen
físico es nada especial Ambos hemocultivos crecen Streptococcus gallolyticus
(Streptococcus bovis biotype I). El ecocardiograma revela vegetaciones en la válvula
mitral. Además del tratamiento con antibióticos, ¿qué paso adicional se recomienda
en este paciente?
A. Broncoscopia [1%]
B. Colonoscopia [90%]
C. Tomografía computarizada de la cabeza [3%]
D. Cistoscopia [0%]
E. Retrógrado endoscópico colangiopancreatografía [0%]
F. Análisis de sangre oculta en heces [3%]
Explicación: Este paciente tiene endocarditis infecciosa de la válvula mitral debido a
la infección con Streptococcus gallolyticus. S gallolyticus (S bovis biotipo 1) es 1 de
las 4 especies principales que pertenecen a los estreptococos del grupo D (también
conocido como complejo S bovisis equinus). El metanálisis mostró un riesgo
significativamente mayor de cáncer colorrectal y endocarditis en pacientes con
infección por S gallolyticus (s bovis) biotipo 1) en comparación con pacientes con
infección por S bovis biotipo ll. Debido a esta asociación, el el paciente debe
someterse a una colonoscopia para buscar malignidad oculta subyacente. (Opción
A) Este paciente tiene un historial de tabaquismo, lo que lo pone en un mayor riesgo
de carcinoma broncogénico. Sin embargo, no se recomienda la evaluación de rutina
o la detección con broncoscopia en pacientes sin síntomas o signos sugestivos de
malignidad pulmonar. Infección por S gallolyticus (S bovis biotipo I) tampoco se
asocia con malignidad pulmonar (Elecciones c y E) Retrógrado endoscópico la
colangiopancreatografía o la tomografía computarizada de la cabeza no se inculpa
en estos pacientes, ya que sin asociación significativa con patología biliar / cerebral
y endocarditis de este organismo

 Un hombre de 40 años llega a la oficina debido a un mes de ardor y hormigueo


que empeora progresivamente sensaciones en las manos y los pies. Él restaura
muebles antiguos como un hobby, pero recientemente ha tenido dificultad para
manejar herramientas pesadas. El paciente no tiene problemas médicos y no toma
medicamentos. Él no usa tabaco ni alcohol La presión arterial es 100/70 mm Hg, y
el pulso es 70 / min. La piel sobre el cuello tiene áreas irregulares de
hiperpigmentación e hipopigmentación. La hiperqueratosis y la incrustación son
presente en las palmas y plantas de los pies. Hay una mayor sensibilidad al
pinchazo y un toque ligero sobre el dedos de manos y pies. La flexión plantar y la
dorsiflexión son débiles en el tobillo, y hay debilidad de los flexores y extensores
interóseos y de la muñeca. Los reflejos tendinosos profundos de las extremidades
superiores e inferiores son 1. Los resultados de laboratorio son los siguientes:
Conteo sanguíneo completo Hemoglobina 10.4 g / dL Leucocitos 4.100 / mm
Plaquetas 130,000 / mm3 Estudios de función hepática Aspartato aminotransferasa
50 U / L Alanina aminotransferasa 62 U / L ¿Cuál de los siguientes es el diagnóstico
más probable para este paciente?

A. Envenenamiento por arsénico [30%]


B. Síndrome de Guillain-Barré [0%]
C. Porfiria intermitente [14%]
D. Plomo envenenamiento (40%)
E. Deficiencia de vitamina A [5%]
F. Deficiencia de vitamina B12 [7%]

 hombre 78 años es llevado al médico debido a caídas recurrentes. Según su hija,


él perdió el equilibrio y caído varias veces en los últimos 6 meses. Él no sostener
cualquier grave lesiones. Ella también ha notado algunos "temblores" de su mano
derecha. El paciente toma amlodipino hipertensión y lansoprazol para la acidez
estomacal. Ha vivido solo desde que su esposa murió hace 5 años, su hija le ayuda
con las compras y gestión de la casa. Su presión arterial es Hg 141/68 y el pulso es
de 72 por minuto el examen neurológico revela leve rigidez de los músculos de la
extremidad y un marcha arrastrando los pies. Cuál de los siguientes efectos
secundarios pueden esperarse después de la iniciación de ¿terapia de
levodopalcarbidopa?

A. coreiformes discinesia [14%]


B. alucinaciones (39%]
C. Livedo reticularis 15%]
D. involuntarios movimientos [30%]
E. la retención urinaria [10%]
Un hombre de 58 años acude al médico y se queja de "problemas con la
erección". Él tiene erecciones recurrentes y persistentemente dolorosas. Sus otros
problemas médicos incluyen colitis ulcerosa cálculos renales, insomnio, depresión,
hipertensión, diabetes inducida por medicamentos, obesidad y
hipercolesterolemia Él no usa tabaco, alcohol o drogas. Toma prednisona,
mesalamina, insulina, 6-mercaptopurina, simvastatina, gliburida, enalapril, trazodona
y fluoxetina. Él no tiene alergias conocidas a los medicamentos. Sus signos vitales
son estables. El examen físico general no tiene nada especial. ¿Evitar cuál de los
siguientes medicamentos podría haber evitado su condición?
A. Fluoxetina [11%]
B. Trazodona 82%]
C. Enalapril [1%]
D. Gliburida 10%]
E. Simvastatina [0%]
F. Prednisona 10%]
G. 6-Mercaptopurina 12%
Explicación: Trazodona es un antidepresivo que se utiliza principalmente para
trastornos del sueño. Su efecto secundario perturbador es el priapismo, que es una
erección persistente y dolorosa que se desarrolla sin estimulación sexual y tiene una
larga duración. Recuerde las causas comunes de priapismo: 1. enfermedad de
células falciformes y leucemia generalmente en niños o adolescentes 2. Perineal o
el trauma genital produce laceración de la arteria cavernosa 3. Lesiones
neurogénicas como la médula espinal lesión del cordón umbilical, compresión de la
cola de caballo, etc. 4. Medicamentos como trazodona y prazosín Ninguno de los
otros medicamentos enumerados pueden causar priapismo

Un hombre de 43 años acude a la consulta debido a la frecuente quema


epigástrica que no se alivia con antiácidos durante los últimos 4 meses. La
sensación generalmente es provocada por el trabajo pesado y el trabajo 10-15
minutos para irse. El paciente no ha tenido dolor asociado en el brazo o el cuello,
tos, falta de aliento o dificultad para tragar. Su historial médico es significativo para
el lupus sistémico eritematoso diagnosticado hace 5 años, para el cual toma
prednisona e hidroxicloroquina en dosis bajas. Él es un no fumador de por
vida. Hace un año, a la esposa del paciente le diagnosticaron úlcera péptica
enfermedad que requirió tratamiento con antibióticos. En el examen físico, la
presión arterial es 140/90 mm Hg y el pulso es 80 / min y regular. Los sonidos de la
respiración son iguales en ambos lados. No hay sibilancias o se escuchan
crepitaciones. Primer y segundo sonido del corazón están presentes. No se notan
murmullos ni frotaciones. El abdomen es suave y no sensible a la palpación
profunda. No hay erupción cutánea ni edema periférico. ECG es normal. ¿Cuál de
los siguientes es el mejor paso siguiente en el manejo de este paciente?

A. Abdominal Tomografía computarizada con / sin contraste 13%]


B. Ecografía abdominal [1%]
C. Tomografía computarizada del tórax sin contraste [1%]
D. Angiografía coronaria [1%]
E. Ecocardiograma 13%]
F. Estudios de motilidad esofágica [4%]
G. Ejercicio ECG [56%]
H. Prueba de antígeno Helicobacter pylori en heces [14%]
I. Gastrointestinales superiores endoscopia [13%]

 Un hombre de 66 años de edad se queja de disnea del exertional que ha


progresado en los últimos dos años. Como un resultado que ha tenido que limitar
sus actividades físicas para evitar convertirse en dificultad para respirar. De
cualquiera dolor torácico importante, pero hace Nota tener una recurrente tos
productiva de esputo blanquecino. Su historia clínica PAS es significativo para la
hipertensión controlada con hidroclorotiazida. Él tiene un historia de tabaquismo de
años 40 pac. Su historia familiar es importante para un movimiento en su madre. Su
presión arterial 160/90 mmHg y su frecuencia cardiaca es de 80 por minuto el
examen físico revela una leve paciente con sobrepeso no hay señal de socorro
aguda. Notablemente aumenta su diámetro anteroposterior de tórax. Sonidos de la
respiración son disminuidos bilateralmente con dispersos wheezes espiratorios.
Cuál de las siguientes ¿agentes es más probable reducir los síntomas de este
paciente?
A. bloqueador alfa-adrenérgico de 15%
B. Agonista de la dopamina de
D. de bloqueador beta-adrenérgico [11%]
C.antagonista muscarínico [70%] Alpha
E. agonistas 2-adrenérgicos 10
Explicación: historia y presentación de este paciente son clásico para la enfermedad
pulmonar crónica (EPOC) típicamente por el consumo de cigarrillo, causa de EPOC
gradualmente disminuir las tasas de flujo espiratorio, aumentar la compliance del
pulmón y aumenta volúmenes pulmonares. Las terapias sólo demostradas para
prolongar la supervivencia en pacientes con EPOC son fumadores cese, oxígeno
suplementario y la cirugía de reducción pulmonar en algunos pacientes que el resto
de terapias son encaminadas a disminuir los síntomas respiratorios, mejorar la
calidad de vida y disminuyendo hospitalizaciones. Los pilares del tratamiento son los
broncodilatadores inhalados, especialmente los fármacos anticolinérgicos como el
ipratropio y tiotropio. Agentes anti - muscarínicos pueden ser combinado con
agonistas beta de acción corta como el salbutamol. Los esteroides inhalados y beta
de acción prolongada agonistas pueden ser utilizados para los pacientes más
graves (opción A) se utilizan para los bloqueadores alfaadrenérgicos tratar la
hipertensión y la hipertrofia prostática benigna. No desempeñan un papel en el
tratamiento de coPD. (Bloqueadores beta-adrenérgicos de la opción B pueden
exacerbar síntomas pulmonares en pacientes con severa enfermedad reactiva de
las vías respiratorias son seguros en la mayoría de los pacientes con leve a
moderada de EPOC y asma,

Un hombre de 32 años llega al servicio de urgencias debido a un día de empeorar


el dolor y la inflamación en su pierna derecha. No tiene ninguna fiebre, dolor de
pecho o dificultad para respirar. El paciente fue hospitalizado 2 hace semanas para
derecha inferior de la extremidad la trombosis venosa profunda después de que un
menor de edad divierte lesiones; fue descarga sobre la warfarina. Cuando salió del
hospital, su INR fue de 2,2 (meta: 2-3). El paciente tiene estado tomando warfarina
diariamente; sin embargo, debido a un horario de trabajo ocupado, su ingesta ha
sido variable y la última semana que él faltó a su cita de la clínica para la
monitorización de la anticoagulación. Él no tiene ningún otros problemas médicos.
La presión arterial es 120/70 mm Hg, pulso es 70 por minuto y respiraciones son
14/min examen muestra moderado edema pretibial de la pierna derecha. Hoy en
día, su INR es 1.3. Conteo de plaquetas, creatinina y pruebas de función hepática
están dentro de los límites de la norma. Doppler venoso ultrasonido muestra un
trombo de la vena poplítea derecha que se extiende a la vena femoral y es peor que
en la ecografía anterior. ¿Cuál es el mejor siguiente paso en la gestión de este
paciente?

A. aumento de warfarina de la dosis y el seguimiento en 3 días [12%]


B. iniciar la terapia trombolítica 22%
C. vena cava inferior filtro [25%]
D. sustituir warfarina con rivaroxabán 38%]
E. iniciar dosis bajas aspirina [1%]

Un hombre de 26 años acude a la consulta debido a dolor e hinchazón en el talón


derecho. Él accidentalmente una uña oxidada mientras trabajaba hace 2
semanas. El clavo perforó su zapatilla y entró en su talón derecho. Él crema
antibiótica sin receta médica y tomó paracetamol oral pero continuó teniendo el
paciente no tiene otros problemas médicos y no toma medicamentos. Él no tiene
alergias conocidas a los medicamentos. Él fuma 2 o 3 cigarrillos por día y bebe
alcohol ocasionalmente. La temperatura es 38.3 C (101 F), la presión arterial es de
140/90 mm Hg, el pulso es de 84 / min y las respiraciones son de 14 lmin. El talón
derecho es hinchado, rojo, cálido y sensible a la palpación. Una pequeña herida por
punción es visible. Resultados de laboratorio muestra leucocitosis leve La
radiografía del pie derecho revela cambios óseos sugestivos de osteomielitis. ¿Cuál
de los siguientes es el organismo más probable que causa la infección de este
paciente?

A. Estreptococos B-hemolíticos [10%]


B. Candida albicans [0%]
C. Clostridium tetani [18%]
D. Escherichia coli [0%]
E. Klebsiella pneumoniae 10%]
F. Mycobacterium tuberculosis (0%)
G. Pseudomonas aeruginosa (45%)
H. Staphylococcus epidermidis [24%]

Explicación: Staphylococcus aureus y Pseudomonas aeruginosa son responsables


de la mayoría de las infecciones profundas después de heridas punzantes. Las
infecciones por Pseudomonas son particularmente prevalentes después de heridas
punzantes a través de la suela de un zapato ya que el ambiente cálido y húmedo es
bastante hospitalario para este microorganismo. Muchas heridas punzantes las
lesiones permanecen asintomáticas, pero se estima que el 50% desarrollan celulitis
superficial o más profunda infecciones tales como osteomielitis. La evidencia de
una infección activa aumenta en aquellos con piel hallazgos de infección (edema,
calor, hema, sensibilidad), leucocitosis y fiebre. Las radiografías son generalmente
se requiere para evaluar la osteomielitis subyacente cuando ocurren lesiones de
penetración profunda; sin embargo, los cambios óseos consistentes con
osteomielitis a menudo toman y gt; 2 Los hemocultivos son usualmente extraído, y
se puede requerir una biopsia ósea con cultivo para identificar el
microorganismo. Tratamiento incluye antibióticos por vía intravenosa (p. ej.,
ciprofloxacina, piperacilina-tazobactam).

Un hombre de 55 años viene a la oficina debido a llagas en la piel. Desarrolló


ulceraciones dolorosas en su boca hace 3 semanas, seguido de un comienzo difuso
de dolor grande, que afecta el tronco y las extremidades. La temperatura es de 3 C
(98.2 F), la presión arterial es de 1 mm Hg, el pulso es de 82 / min y las
respiraciones son 14 / min. En el examen, la mucosa oral muestra varias erosiones
y ulceraciones. La piel muestra esparcidas, grandes erosiones y algunas ampollas
flácidas. El roce ligero de la piel no afectada hace que sea fácil separación de la
epidermis. Se realiza biopsia con sacabocados de la piel e inmunofluorescencia la
microscopía muestra depósitos intercelulares de IgG. ¿Cuál de los siguientes es el
más probable ¿diagnóstico?

A. Impétigo ampolloso 1%
B. Penfigoide bulloso 17%
C. Dermatitis herpetiforme 0%
D. Eritema multiforme 0%
E. Pemphigus vulgaris 80%

Un hombre de 64 años de edad sufre una emergencia resección colónica por
colitis isquémica extensa. H vida solo y consume grandes cantidades de alcohol. Su
historia familiar es no contributiva. El paciente es extubados en el 4 º día
postoperatorio. Después de la extubación, tiene episodios de confusión y agitación
tratado con lorazepam y haloperidol. Él también está recibiendo piperacilina con
tazobactam. El paciente ha tenido nada por boca desde la cirugía. En el día
postoperatorio 7, la avisos de enfermera sangrado desde un sitio de venopunción.
Su temperatura es de 36,7 C (98 F), presión arterial es 121/76 mmHg, pulso 80/min,
y respiraciones 16/min resultados de laboratorio son los siguientes: Hemoglobina
11,5 g/dL. Volumen corpuscular medio 88 fL plaquetas 160.000/uL leucocitos
7.500/UL Neutrófilos segmentados 68% bandas 1% eosinófilos 1% linfocitos 24%
monocitos 6% el tiempo de protrombina 24 tiempo de tromboplastina parcial seg
(INR 2.2) 44 segundos (normal 25-40 cuál de las siguientes es la más ¿causa
probable de la condición actual de este paciente?

A. la coagulación intravascular diseminada aguda de [28%]


B. Factor V Leiden 12%]
C. Factor VIII deficiencia 12%)
D. Lupus de E. hiperesplenismo [0%]
anticoagulante [0%]
F. púrpura trombocitopénica trombótica 12%]
G. vitamina K deficiencia (60%]
H. enfermedad de von Willebrand (concepto no activo)

Un hombre de 45 años de edad se evalúa de otitis y sinusitis recurrente. Él ha


sido tratado con antibióticos y corticosteroides intranasales y para los últimos 6
meses, pero él todavía tiene escasa secreción nasal amarillo mezclado en
ocasiones con sangre. En la revisión de los sistemas, tiene fatiga, pero no fiebres,
escalofríos, tos o síntomas abdominales. El paciente es un ex fumador con un 15-
embalar-años historia. Toma sin receta ibuprofeno y aspirina para dolores de
espalda y conjuntos. Es afebril. Pecho y exámenes abdominales son irrelevantes.
Otoscopía muestra eritema y una pequeña ulceración en el canal auditivo derecho.
Resultados de laboratorio son los siguientes: hemograma completo hemoglobina
10,8 g/dL de sangre de orina proteína 2 sangre 2 blanco de plaquetas 410.000/mm3
leucocitos 10.700/mm3 células 1-2/hpf glóbulos rojos 20-30/hpf VIH su prueba es
negativa. Cuál de las siguientes es la ¿mejor siguiente paso en la gestión de este
paciente?
A. aspirina reto y desensibilización 15 %1
B. O
C. cualitativa inmune
d. cuantitativa de suero autoanticuerpos [56%]
E. biopsia de mucosa nasal [13%] cloruro del sudor
F. globulina niveles 122%] pruebas [1%]

Un hombre de 67 años de edad llega a la clínica por primera vez. Habla y camina
muy lentamente la síntoma principal del paciente es "olvido extrema" durante los
últimos 6 meses y teme que es deve demencia como mi padre." Dice: "Yo solía ser
una persona muy brillante y aguda, pero ahora puede l ' t incluso se concentran para
trabajar o leer un libro o un periódico. Me siento muy baja e inútil". El paciente
llorando acciones que él ha sido perder el sueño, tiene poca energía y
recientemente tomó una licencia de ausencia de trabajo como ya no podía
concentrarse en los detalles y completar su documentación. Historial médico
significativo para la hipertensión, hipercolesterolemia, diabetes, hiperplasia benigna,
y ataque isquémico transitorio. Historia familiar es importante para la hipertensión y
la diabetes en su madre de una enfermedad de Alzheimer de su padre él no fuma y
bebe vino sólo de vez en cuando. El paciente ha vivido solo durante los últimos 6
meses puesto que su hijo que se mudó. El examen físico es normal a excepción de
movimientos notablemente lentos. Tomografía computarizada de la cabeza es
normal. Cuál de las siguientes es ¿causa más probable de deterioro cognitivo de
este paciente?

A. Alzheimer Enfermedad 12%)


B. principales trastorno depresivo (82%]
C. envejecimiento Normal 12%]
D. enfermedad [8%]
E. vascular enfermedad de Parkinson (3%)

Explicación: deterioro cognitivo de este paciente, bajo estado de ánimo,


sentimientos de inutilidad, disturbio del sueño, baja energía y retraso psicomotor
(habla lenta, pensamiento y movimientos) son compatibles con desorden depresivo
importante (MDD). Ancianos que están gravemente deprimidos a menudo presente
con pérdida de la memoria de los pacientes que a veces se conoce como
pseudodemencia. Déficit en la atención, concentración, memoria y funcionamiento
ejecutivo pueden verse y a menudo se ven agravadas por pobre esfuerzo durante la
prueba. Aunque estos pacientes tienen mayor riesgo de desarrollar demencia más
tarde su depresión relacionados con el deterioro cognitivo es generalmente
reversible y mejora con tratamiento de la depresión subyacente
Un hombre de 34 años de edad se está recuperando del trauma en la cabeza
sufrido en un accidente de vehículo de motor. Lo es actualmente en la unidad de
cuidados intensivos con ventilación mecánica. Su más reciente gas de sangre
arterial muestra de análisis: pH 7.54 124 Pao2 mmHg PaCO2 20 mmHg bicarbonato
17 mEq/L cuál de las siguientes ¿resultados adicionales lo que más esperan en este
paciente? Excreción de bicarbonato

A. baja orina [14%]


B. alta orina pH [65%]
C. suero alto aldosterona nivel [8%] alta
D. suero anión gap [10%] alto
E. nivel de albúmina sérica 12%]

Explicación: al ser ventilados mecánicamente este paciente es vulnerables a forzado


hiper- o hipoventilación ventilación se define como el volumen de marea multiplicado
por la tasa de respira. Hiperventilación (causada por el aumento del volumen tidal y
resultados de la frecuencia respiratoria) en las pérdidas excesivas de CO2 y
alcalosis respiratoria, mientras que la hipoventilación (causada por disminución del
volumen tidal y frecuencia respiratoria) resultados en la retención de co2 y
respiratoria acidosis. El paciente descrito sufre alcalosis respiratoria según lo
evidenciado por su mayor pH de sangre arterial (norma 7.4) y su disminución de
Paco2 (normal 40 mmHg). Su nivel de bicarbonato del suero es disminuida debido al
intento compensación renal de la alcalosis respiratoria. Específicamente, la los
riñones retener mayores cantidades de H (protones) y excretan mayores cantidades
de (Hco3 en un intento de normalizar el pH del suero. El aumento en la cantidad de
HCo en la orina alkalinizes la orina. 3-

Un hombre de 65 años acude al servicio de urgencias debido a fiebre, escalofríos


y tos productiva durante los últimos 3 días. También tiene dificultad para respirar y
dolor en el lado derecho del pecho que es peor con respiración profunda. Sus otros
problemas médicos incluyen enfermedad arterial coronaria, diabetes tipo 2 mellitus e
hiperlipidemia. Los medicamentos del paciente incluyen dosis bajas de aspirina,
atorvastatina, metformina y sitagliptina. Él no usa tabaco, alcohol o drogas
ilícitas. Su temperatura es 39.2 C (102.5 F), la presión arterial es 95/60 mm Hg, el
pulso es 112 / min, y las respiraciones son 24 / min. La saturación de oxígeno es del
96% en el aire ambiente. El examen muestra membranas mucosas
secas. Resultados de laboratorio son los siguientes: 17,0 g / dL. Plaquetas de
Hemoglobina 250,000 / uL Leucocitos 16,500 / uL de Sodio 135 mEq / L Potasio 4.9
mEq / L Nitrógeno en la sangre en sangre 48 mg / dL Creatinina 2.0 mg / dL Calcio
10.3 mg / dL Sangre glucosa 128 mg / dL. La radiografía de tórax revela un
infiltrado del lóbulo inferior derecho. Se obtienen hemocultivos y se administran
líquidos por vía intravenosa y antibióticos. ¿Cuál de los siguientes es el más
apropiado siguiente paso en el manejo de este paciente?

A. Suspender la aspirina 18%] O


B. Descontinuar atorvastatin [3%]
C. Interrumpir la metformina [59%]
D. Iniciar la infusión de insulina intravenosa 19%]
E. Comience con lisinopril [11%
F. tart zoledronic acid 16%
Un hombre de 34 años de edad se está recuperando de un traumatismo
craneoencefálico sufrido en un accidente automovilístico. Él es actualmente en la
unidad de cuidados intensivos en ventilación mecánica. Su gas arterial más
reciente el análisis muestra: pH 7,54 PaO 124 mm PaCO2 20 mmHg Bicarbonato 17
mEq / l. Cuál de los siguientes hallazgos adicionales que usted espera más en este
paciente? Repetida

A. Baja excreción de bicarbonato en la orina [14%]


B. Orina alta excreción de bicarbonato [65%]
C. Nivel de aldosterona sérica alta 18%]
D. Hendidura aniónica alta en suero [10%]
E. Alta nivel de albúmina sérica 12%]

Explicación: Mientras está siendo ventilado este paciente es vulnerable a hiper o


hipoventilación forzada. la ventilación se define como el volumen tidal multiplicado
por el la frecuencia respiratoria. Hiperventilación (causada por el aumento del
volumen tidal y / o la frecuencia respiratoria) da como resultado pérdidas de CO2
excesivas y alcalosis respiratoria, mientras que la hipoventilación (causada por la
disminución del volumen corriente y / o respiratorio da como resultado la retención
de CO2 y la acidosis respiratoria. los paciente descrito está experimentando
alcalosis respiratoria como lo demuestra su aumento arterial el pH sanguíneo
(normal 7.4) disminuyó la Pacoa (40 normales y su mmHg). Su nivel de bicarbonato
sérico es disminuyó debido a la compensación renal por la alcalosis respiratoria.
Específicamente, la los riñones retienen cantidades aumentadas de H (protones) y
excretan mayores cantidades de bicarbonato (Hco3- en un intento de normalizar el
pH del suero. La mayor cantidad de Hco en la orina alcaliniza la orina. 3-

Un hombre de 46 años de edad se somete a una reparación de hernia inguinal


electiva. Después de la inducción de general anestesia, se nota que es pálido y
taquicárdico. El historial médico del paciente es significativo para dolores de cabeza
frecuentes, hipertensión y un trastorno de ansiedad. Sus medicamentos para
pacientes ambulatorios incluyen lisinopril, alprazolam y, según sea necesario,
naproxeno. El paciente regularmente bebe un paquete de 6 cervezas fines de
semana pero no usa tabaco ni drogas ilícitas. El historial familiar no tiene nada
especial. Su temperatura es 36.9 C (98.4 F), la presión arterial es ahora 250/140
mm Hg (era 144/90 mm Hg antes de la inducción), y el pulso es 125 / min. El ECG
muestra taquicardia sinusal sin cambios isquémicos. ¿Cuál de los siguientes es el
diagnóstico más probable?

A. Abstinencia alcohólica [6%]


B. Anafilaxia [5%]
C. Trastorno de pánico [2%]
D. Feocromocitoma [52%]
E. Síndrome serotoninérgico [11%]
F.Tormenta tiroidea [22%]

Explicación: En luz de la historia de este paciente de hipertensión y dolores de


cabeza frecuentes, su presentación con La hipertensión severa, la palidez y la
taquicardia sinusal son preocupantes para un feocromocitoma oculto con una
oleada de catecolaminas debido a la anestesia. Los feocromocitomas y para son
tumores productores de catecolaminas que surgen de las células cromafines de la
médula suprarrenal o paraganglia extra suprarrenal, respectivamente. La
hipertensión en el feocromocitoma puede ser intermitente o sostenido. Los
paroxismos de hipertensión severa pueden precipitarse por aumentos en los niveles
intraabdominales presión (por ejemplo, cambios de posición de la palpación del
tumor), procedimientos quirúrgicos y una cantidad de medicamentos, en particular la
adición de anestésicos, bloqueadores beta no selectivos pueden causar un estado
de estimulación adrenérgica sin oposición que conduce a la hipertensión paradójica
vasoconstricción Para esto razón, los bloqueadores alfa y adrenérgicos (p. ej.,
phenoxybenza se administran bloqueadores en pacientes con Feocromocitoma

Una mujer de 23 años de edad llega a la clínica debido a un dolor y tiesura de la


mano intermitente de 3 meses. Ella tiene antecedentes de dolor de rodilla y agudo
también se sintió excesivamente cansado y ha tenido episodios ocasionales Dolor
de pecho. El paciente se quema fácilmente con una exposición mínima, que ella
atribuye a la efectos adversos de su medicación para el acné. Ella tiene un historial
de acné que ha sido muy vulgar, controlado por minociclina oral. El paciente no usa
tabaco, alcohol o drogas ilícitas. La temperatura es 37.2 C (98.9 F), la presión
arterial es 130/70 mm Hg, y el pulso es 80 / min. Mano bilateral articulaciones y son
levemente sensibles e hinchadas. Los campos pulmonares son claros para la
auscultación y los ruidos cardíacos son normales El abdomen es suave y no
sensible, sin organomegalia. Hay una disminución bilateral leve edema de
extremidades El paciente no tiene erupción de la familia ni linfadenopatía. Los
resultados de laboratorio son los siguientes: Hemoglobina 12.2 g / dL Plaquetas
98,000 / mm3 Leucocitos 3,300 / mm Creatinina sérica 0.8 mg / dL. Proteína de
análisis de orina 2, 5-10 eritrocitos / HPF Cuál de los siguientes es el mejor paso
siguiente en evaluando a este paciente?

A. Anticuerpo peptídico citrulinado cíclico 15%


B. Anti-bicatenario Anticuerpo de ADN 45%
C. anticuerpo anti-histona 17%
D. anticuerpo anti-topoisomerasa 12%
E. antinuclear anticuerpo [39%

Un hombre de 68 años acude al servicio de urgencias debido a un


empeoramiento progresivo de la insuficiencia del hombre respiración por 2 días. Ha
tenido una semana de fiebre baja, secreción nasal y tos productiva. los otros
problemas médicos del paciente incluyen enfermedad arterial coronaria,
hipertensión y hiperlipidemia. Fue hospitalizado hace un año por un infarto agudo de
miocardio y fue tratado con colocación del stent liberador de fármaco en la arteria
descendente anterior izquierda. El paciente tiene un paquete de 40 años
antecedentes de tabaquismo y dejar de fumar después del infarto de miocardio. Su
temperatura es de 37.2 C (99 F), sangre la presión es de 140/90 mm Hg, el pulso es
de 90 / min y las respiraciones son de 22 fmin. El paciente parece ser en dificultad
respiratoria leve. Utiliza los músculos respiratorios accesorios para respirar, pero
puede hablar en oraciones completas. Las venas del cuello están levemente
distendidas, especialmente durante la espiración. Auscultación pulmonar muestra
disminución de los ruidos respiratorios y sibilancias bilaterales. Los sonidos del
corazón son distantes. Plasma B-type el nivel de péptido natriurético es de 88 pg /
ml (0-100 pg / ml normal). La radiografía de tórax se muestra a continuación. ¿Cuál
de la siguiente es la causa más probable de la presentación de este paciente?

A. Asma aguda exacerbación 13%)


B. Insuficiencia cardíaca descompensada aguda [5%]
C. embolia pulmonar aguda 12%]
D. Exacerbación de la enfermedad pulmonar obstructiva crónica 68%
E. Neumonía adquirida en la comunidad 15%
F. Taponamiento cardíaco subaguda [15%]

Un hombre hospitalizado de 46 años es evaluado por confusión. Fue admitido


hace 3 días después de colisión del vehículo a motor en el que era pasajero. Él
sufrió fracturas de la tibia derecha y peroné, que se redujeron y arreglaron
quirúrgicamente. El paciente tenía un pequeño hematoma subdural sin cambio en la
línea media, que se manejó de forma no operativa. Él ha estado recibiendo morfina
intravenosa por control de dolor. Esta mañana, el paciente estaba agitado y
perturbador y estaba hablando con personas que No estaban ahí. Él no tiene
antecedentes médicos significativos y no toma medicamentos regulares.
Temperatura es 37.5 c (99.5 F), la presión arterial es 170/100 mm Hg, el pulso es
120 / min y regular, y respiraciones son 22 min. La oximetría de pulso muestra un
96% en el aire ambiente. El paciente parece diaforético y trémulo y está tratando de
sacar las líneas intravenosas. Él no coopera para el examen, pero los alumnos
están igual y reactivo bilateralmente Los sonidos de la respiración son normales y
no se escuchan los soplos cardíacos. los el abdomen es suave y no sensible. La
incisión quirúrgica no muestra eritema circundante. El paciente mueve todas las
extremidades sin limitaciones. ¿Cuál de las siguientes es la causa más probable de
esto? confusión aguda del paciente?

A. Delirium tremens 84%


B. Presión intracraneal elevada 4%
C. Síndrome de embolia grasa 15%
D. Hipertermia maligna (0%)
E. Síndrome maligno neuroléptico 10%]
DE intoxicación por opiáceos [2%]
G. embolia pulmonar [0%]

 Un varón de 50 años presenta poliuria y polidipsia. Ha fumado 1 paquete de


cigarrillos da para los últimos 30 años. Él niega tener problemas médicos pasados.
Su madre y un tío compañero son diabéticos. Su altura es de 58 "(172 cm), el peso
es de 180 lb (81,6 kg), la temperatura es de 37 ° C (98,6 pulso) es de 75 / min, la
presión arterial es de 150/90 mm Hg, y las respiraciones son 15 / min. Examen de
un sistemas no tiene nada especial. El panel de química muestra: Sodio 140 mEq /
L Potasio 4.1 mEq / L Bicarbonato 26 mEq / L. Glucosa en sangre 210 mg / dL BUN
12 mg / dL Creatinina 0.9 mg / dL El paciente es diagnosticado con tipo 2 diabetes
mellitus. Se le aconseja el ejercicio y la modificación de la dieta. Él es referido a una
oftalmólogo y adecuadamente seleccionados para la retinopatía diabética. ¿Cuál es
el más sensible? prueba para detectar nefropatía en este paciente?

A. aclaramiento de creatinina 5%
B. prueba con varilla de medición Orina para proteína
C. Orina al azar para proporción de microalbúmina / creatinina [76%]
D. Ecografía renal [1%]
E. Prueba de tolerancia oral a la glucosa [1%]

Explicación: El desarrollo de la nefropatía es precedido por el desarrollo de excesiva


excreción de proteínas, cuyas etapas iniciales se denominan microalbuminuria. Los
pacientes con microalbuminuria generalmente tienen un valor de excreción de
albúmina en la orina entre 30-300 mg / 24 h (La excreción normal de proteína en la
orina es de 30 mg / 24 h). Recogida puntual de orina y la recolección de orina
programada para la medición de la microalbúmina en orina a la relación de
creatinina en general aceptado como buenos métodos de detección para la
microalbuminuria. Aunque la recolección de orina de 24 horas es un poco más
preciso en la detección de microalbuminuria.

Una mujer de 21 años llega a la oficina debido a un historial de 7 días de dolor de


garganta, fatiga extrema, mialgia y dolores de cabeza. Recientemente regresó de un
receso de invierno en Jamaica donde tuvo "la momento de mi vida ". La paciente no
tiene un historial médico significativo y no toma medicamentos. fuma medio paquete
de cigarrillos al día y ocasionalmente consume alcohol. La temperatura es 38 C
(100.4 F), la frecuencia cardíaca es de 78 / min, la presión arterial es de 114/76 mm
Hg y las respiraciones son de 14 / min. El examen revela amígdalas eritematosas
ampliadas con exudados, petequias palatinas, generalizadas linfadenopatía y
esplenomegalia leve. El conteo sanguíneo completo muestra lo siguiente:
Leucocitos 16,000 / mm3 con 55% de linfocitos Hemoglobina 13.5 g / dL Plaquetas
216,000 / mm3 Muchas formas variantes de se ven linfocitos, incluidas las células
con núcleos convolucionados y citoplasma altamente vacuolado. Los resultados de
la prueba rápida de antígeno estreptocócico y la prueba de anticuerpos heterófilos
son negativos. ¿Cuál es el diagnóstico más probable?

A. Leucemia linfoblástica aguda (7%)


B. Leucemia mieloide aguda [1%]
C. Infección por adenovirus [7%]
D. Infección por estreptococos del grupo A (1%)
E. linfoma de Hodgkin (6%)
F. Mononucleosis infecciosa 73%
G. Infección por influenza 12%]

Un hombre de 45 años acude al servicio de urgencias quejándose de disnea,


fatiga, falta de apetito, y aumento de peso en las últimas 2 semanas. Sus síntomas
comenzaron con un empeoramiento de la falta de aliento con esfuerzo. Se ha
estado despertando por la noche sin aliento durante los últimos días. Él también lo
encuentra difícil abrir los ojos por la mañana debido al edema facial. El paciente no
tiene otro médico problemas y no toma medicamentos. Su presión arterial es de
200/120 mm Hg y el pulso es de 100 / min. El examen físico muestra anasarca con
venas yugulares dilatadas mientras está sentado en posición vertical. Pulmón la
auscultación muestra crepitantes bibasilares. El análisis de orina muestra 1
proteína, sin nitrito, traza de leucocitos esterasa, y gt; 50 glóbulos rojos, cilindros de
glóbulos rojos y neutrófilos ocasionales. ¿Cuál de el siguiente es el mecanismo más
probable del edema de este paciente?

A. Hipertiroidismo 10%]
B. Hipoalbuminemia [17%]
C. Hipertensión portal [3%]
D. Daño glomerular primario [52%]
E. Renal hipoperfusión [10%]
F. Fallo ventricular derecho [16%]

Se realizaron dos estudios en diferentes muestras de la misma población para


evaluar la relación entre el uso de anticonceptivos orales y el riesgo de trombosis
venosa profunda (TVP). El estudio A mostró una aumento del riesgo de TVP entre
los usuarios de anticonceptivos orales, con un riesgo relativo de 2.0 y un 95%
intervalo de confianza de 1.2-2.8. El estudio B mostró un riesgo relativo de 2.01 y
una confianza del 95% intervalo de 0.8-3.1 ¿Cuál de las siguientes afirmaciones es
más probable que sea cierto con respecto a estos 2 ¿estudios?

A. El valor de p en el estudio B es probable y lt; 0.05 [11%]


B. El resultado en el estudio A no es precisión [1%] O
C. El resultado en el estudio A no es estadísticamente significativo [5%]
D. El resultado en el estudio B probablemente esté sesgado [8% l vy O
E. El tamaño de la muestra es probablemente menor en el estudio B que
en el estudio A [73%]

 Las Naciones Unidas un ensayo clínico grande, eran pacientes con infarto de
miocardio reciente e insuficiencia cardíaca aleatorizados a placebo o medicamento
del estudio. La fracción de eyección media del ventrículo izquierdo de los sujetos de
estudio fue del 33%. Mayoría de los pacientes en ambos grupos fueron tratada
concomitantemente con un bloqueador beta y un as en el inhibidor de angiotensina
o un antagonista. las curvas de mortalidad a largo plazo, todas las causas receptor
se muestra al mago por debajo, con la línea continua que representa el
medicamento del estudio. Que de la lo que sigue es el medicamento más probable
en el tiempo clínico, años

A. Amlodipine 6%
B. digoxina 7%
C. Diltiazem 15%
D. espironolactona 70%
E. hidroclorotiazida 18%
F. ranolazina 12%

Explicación: Los medicamentos que han demostrado mejorar la supervivencia a


largo plazo en pacientes con la izquierda disfunción ventricular (LV) son inhibidores
de la ECA, los bloqueadores de los receptores de angiotensina II (ARBs), beta
bloqueadores (BB), antagonistas del receptor de mineralocorticoides (ARM) y (en
pacientes afroamericanos) un combinación de hidralazina y nitratos. En este
ensayo, los pacientes tomando el medicamento de estudio (línea sólida) ha
mejorado la supervivencia. Como pacientes en ambos grupos tomaban las
municiones y los inhibidores de la ACE/ARB, el

Una mujer de 25 años de edad llega a la oficina debido a la pérdida de cabello.


Durante los últimos 6 meses, ella se ha dado cuenta adelgazamiento del cabello en
su cabeza y un aumento en la cantidad de pelo en el desagüe de la ducha. El
paciente no es pero actualmente sexualmente activa ha tenido 5 socios de toda la
vida. Menarquia fue a los 11 años. Ella tiene luz periodos cada 35-50 días. Ella
necesita una ayuda para dormir sin receta médica, pero no usa tabaco, alcohol o
drogas ilícitas. La madre del paciente edad 56 y carcinoma endometrial; su abuela
paterna murió de cáncer de mama metastásico diagnosticado en la edad 72.
Revisión de los sistemas es negativo excepto acné facial y posterior extenso. Índice
de masa corporal es de 33 kg/m2. El examen físico muestra adelgazamiento del
cabello en la frente con una rayita del retroceso. La tiroides es nonenlarged con sin
nódulos. La examinación cardiaca muestra regular y el ritmo; hay no hay soplos. El
abdomen es obeso no masas palpables. A prueba de embarazo es negativa. Suero
prolactina y tiroides función las pruebas son normales. ¿Cuál de las siguientes
pruebas adicionales se indica en este paciente?

A. BRCA mutación de pruebas [12%]


B. laparoscopia diagnóstica [4%]
C. hierro estudios [7%]
D. prueba para tolerancia a la glucosa oral [70%]
E. Rapid plasma reagin [3%]

Un hombre de 67 años de edad llega a la oficina debido a empeoramiento de la


frecuencia urinaria, nicturia y disuria inicial durante el año pasado. Se despierta 2 o
3 veces por noche a void. El paciente también dice que la fuerza de su secuencia se
disminuye y se siente que su vejiga no se evacua totalmente después de anular.
Tiene antecedentes de hipertensión y osteoartritis. El paciente tiene un 35-embalar-
años historia de fumar pero dejó de fumar hace 10 años. Temperatura es de 37,1 C
(98,8 F), la presión arterial es 130\/80 mm Hg, pulso es 78\/min el examen físico
muestra un abdomen blando y no quirurgico. Rectal el examen revela una próstata
agrandada, lisa sin nódulos y tono del esfínter rectal normal. Exploración de vejiga
post micción muestra 75 mL de orina (normal y lt; 12 mL). Análisis de orina y
creatinina sérica son normales. El antígeno prostático específico suero es 2,8
ng\/mL (valor de referencia ajustados por edad y lt; 4.5 ng\/mL). ¿Cuál de los
siguientes es el mejor siguiente paso en la gestión de este paciente?

A. bloqueador de inhibidor de la 5-alfa-reductasa [33%]


B. a adrenorreceptores α [55%]
C. intermitente urinaria Cateterismo [3%] Trans
D. biopsia rectal de la próstata [1%]
E. próstata resección transuretral [1%]
F el. urodinámico estudia [4%]

Un estudiante de escuela superior 17 años llega a la oficina para la evaluación del
dolor de la rodilla. Ella tiene un historia progresiva de 3 meses de dolor mal
localizado, dolor en la rodilla derecha que es peor con correr, sentada durante un
largo período y subir o bajar escaleras. El paciente ha estado entrenando intensivo
de danza competitiva y ha tenido que limitar sus sesiones de práctica debido a una
sensación que la rodilla está "regalando" o pandeo. Inicialmente tenía el alivio
parcial de medicamentos de venta libre antiinflamatorios no esteroideos, pero su
efectividad ha disminuido. Su historial médico es unremarkable, y ella no tome
medicamentos de prescripción. El examen muestra paso normal y ninguna
deformidad visible en las rodillas. Hay leve crepitación con rango de movimiento en
la rodilla derecha. Con la rodilla extendida, compresión de la rótula en el surco
troclear reproduce el dolor. Cuál de los siguientes es el paso siguiente más
adecuado en la gestión de este paciente ¿síntomas?

A. articulada rodillera [11%]


B. glucocorticoides intraarticulares inyección 19%
C. rodilla inmovilizador [14%]
D. orales glucocorticoides [1%]
E. patelar artroplastia [4%]
F. cuadriceps Ejercicios de fortalecimiento

Un hombre de 45 años de edad llega a la oficina para la evaluación de la pérdida


excesiva de su extremidad músculos, que es más evidente en el lado del extensor.
La debilidad comienza distalmente y asimétricamente. Recientemente comenzó a
tener dificultades con la deglución, masticación y habla. Él se siente algunos
movimientos en su cara y lengua. También tiene rigidez muscular. Su intestino,
vejiga, las funciones cognitivas y sensoriales están intactas. El examen físico revela
pérdida excesiva de sus músculos, que es más prominente en las extremidades
inferiores. Fasciculaciones e hiperreflexia del se observan todas las extremidades.
Se disminuyen sus reflejos bulbares. Qué camino neural es más probable ¿dañado?
A. zona piramidal [4%]
B. inferior motoneurona [6%]
C. motoneurona superior [13%]
D. motoneurona superior e inferior [73%]
E. corteza Cerebral 12%

Este paciente tiene amiotrófica esclerosis lateral (ELA), que es una de la peor y más
devastador neurodegenerativas trastornos. Se caracteriza por la presencia de las
lesiones de motoneurona superior e inferior. Síntomas bulbares, la espasticidad y
reflejos tendinosos profundos y exagerados son signos de motoneurona superior
dañar, mientras que las fasciculaciones indican un menor daño de la neurona de
motor. Debilidad y atrofia de la los músculos pueden ser debido a las lesiones de
motoneurona superior e inferior.

Una mujer de 28 años, mujer embarazada Grava 2 para 0 aborta 2, viene a la


oficina después de un aborto espontáneo. Ella tuvo un aborto de primer trimestre
anterior hace 9 meses. El paciente no tiene otros médicos conocido problemas. Ella
no ha experimentado ningún cambio reciente en el apetito o peso, diarrea,
estreñimiento, intolerancia a calor o frío, disnea y ronquera. Madre de la paciente
tiene hipotiroidismo tratado con levotiroxina y la hipertensión tratan con
hidroclorotiazida. El paciente no utiliza tabaco o alcohol. El examen muestra un
tiroides agrandado simétricamente, nontender, firme. No hay ninguna linfadenopatía
cervical, y el resto de la exploración física es normal. Su TSH nivel es de 7.2 HU/mL
y T4 libre es normal. Sonografía pélvica es normal. Cuál de las siguientes
¿autoanticuerpos se asocia probablemente con dolencia de este paciente?

A. Anticardiolipin anticuerpos [39%]


B. Antimitochondrial anticuerpo [2%]
C. antitiroideo peroxidasa
d. inmunoglobulina estimulante de tiroides
E. anticuerpo de TSH-receptor-bloqueo E. [14%]

Una mujer de 33 años viene a la oficina con mareos intermitentes. El paciente


describe un sensación de giro severo acompañado de náuseas intensas que duran
1-2 horas. Ella se siente inestable durante estos episodios y tiene que acostarse
con los ojos cerrados para aliviarse. El paciente ha tenido varios episodios similares
durante los últimos 2 años y no ha notado ningún factor particular que precipitar los
síntomas. También informa haber escuchado un "zumbido mecánico" en su oído
derecho durante estos episodios, causando distorsión del habla. El paciente no
tiene dolores de cabeza asociados, oreja dolor o secreción del oído Su temperatura
es de 36.6 C (98 F), la presión arterial es de 130/84 mm Hg, y el pulso es 86 /
min. En el examen, la conducción de aire es mayor que la conducción ósea en
ambos oídos. Cuando el la base de un diapasón se coloca contra su frente, el
sonido se escucha más prominente en el oreja izquierda. ¿Cuál de las siguientes es
la causa más probable de la condición de este paciente?

A. Benigno tumor que comprime un nervio craneal [17%]


B. Desmielinización del sistema nervioso central [4%]
C. Elevado presión endolinfática 49%]
D. Inflamación del laberinto membranoso [10%]
E. Calcio suelto restos en el canal semicircular [13%]
F. Movilidad reducida de los huesecillos 13%]

Una mujer de 58 años sometida a quimioterapia sistémica para cáncer de mama


avanzado con metástasis a el hueso llega a la oficina para un seguimiento de
rutina. Tiene un dolor de huesos leve y vago que se alivia con el uso ocasional de
paracetamol. El examen físico muestra una cicatriz de mastectomía bien curada en
el derecho y un ganglio linfático supraclavicular derecho palpable. Las membranas
mucosas están húmedas. Laboratorio los resultados son los siguientes: Química
sérica 144 mEq / L, Sodio 3.8 mEq / L. Cloruro de Potasio 102 mEq / L, 24 mEq / L
Bicarbonato Nitrógeno ureico en sangre 14 mg / dL Creatinina 0.8 mg / dL Calcio
11.4 mg / dL Glucosa 146 mg / dL Estudios de función hepática Albúmina 3.4 g / dL
¿Cuál de los siguientes es el mejor paso siguiente en manejo de este paciente?

A. Terapia con bisfosfonatos [62%]


B. Terapia con corticosteroides [2%]
C. Diuréticos de asa 19%]
D. Terapia con opioides [4%]
E. Radioterapia [20%]
Un hombre de 47 años acude a la oficina a mediados de enero debido a una
fiebre persistente de alto grado, dolor de cabeza, y severas mialgias que
comenzaron abruptamente hace 4 días. El paciente también tiene dolor de garganta,
no productivo tos y congestión nasal leve, pero falta de aliento o dolor en el pecho.
Ha sido y gt; 5 años desde su última cita, y él no está al día sobre el mantenimiento
de la salud o las inmunizaciones. El paciente no tiene problemas médicos crónicos,
no toma medicamentos y no tiene alergias medicamentosas conocidas. La
temperatura es de 38.5 C (101.3 F), la presión arterial es de 135/80 mm Hg, el pulso
es de 88 / min y las respiraciones son 16 / min. la saturación de oxígeno es del 98%
en el aire ambiente. El examen físico muestra un leve eritematoso orofaringe con
amígdalas normales. No hay linfadenopatía cervical significativa. La respiración
suena son claros e iguales bilateralmente sin matices a la percusión o egofonía Los
sonidos cardíacos son normales. ¿Cuál de los siguientes es el siguiente paso más
apropiado en el manejo de este paciente?

A. Administrar vacunación contra la influenza 5%


B. Realizar una prueba rápida de antígeno estreptocócico 19%
C. Prescribir azitromicina 1%
D. Prescribir oseltamivir 11%
E. Recomendar tratamiento sintomático 72

Un varón blanco de 44 años presenta una larga historia de dolores en las
articulaciones en varias articulaciones. Él tiene médico antes, pero no se hizo un
diagnóstico. Él ha estado tomando ibuprofeno con alivio parcial. Él ahora ha
desarrollado fiebre, diarrea y pérdida de peso. Él niega cualquier genitourinary o
síntomas de ojo. Él no usa tabaco, alcohol o drogas. El es un granjero. En el
examen, él ha generalizado linfadenopatía y artritis no deformante. La biopsia del
intestino delgado revela ácido periódico-Schiff (PAS) macrófagos positivos. ¿Cuál
de los siguientes es el diagnóstico más probable?

A. Reactivo artritis 3%]


B. Sarcoidosis 12%]
C. Enfermedad inflamatoria intestinal 4%
D. enfermedad de Whipple [82%]
E. Enfermedad celíaca (4%)
F. Linfoma intestinal 12%]
G. Infección por VIH [0%]
Explicación: el paciente descrito es muy probable que padezca la enfermedad de
Whipple. La enfermedad de Whipple es una enfermedad multisistémica que se
caracteriza por una multitud de posibles manifestaciones. Más comúnmente,
pacientes con enfermedad de Whipple presentan antecedentes de diarrea
malabsortiva crónica (esteatorrea, flatulencia, distensión abdominal), enteropatía
con pérdida de proteínas, pérdida de peso, migración artritis no deformante,
linfadenopatía y fiebre baja. La enfermedad también puede causar daño a la vista
del SNC y el miocardio. La enfermedad de Whipple es causada por una infección
con grampositivos bacilo Tropheryma whippelii, pero un mecanismo por el cual los
pacientes contraen esta infección aún no se ha ser determinado. El diagnóstico se
realiza con una biopsia de intestino delgado y PCR en pacientes con síntomas
clínicos consistentes con la enfermedad. La biopsia del intestino delgado muestra
PAS-positivo macrófagos en la lámina propia que contienen bacilos grampositivos
no ácidos-rápidos

Un hombre de 28 años acude a la oficina para un examen de rutina de


mantenimiento de la salud. El paciente tiene un Historial de 2 años de asma
bronquial, para el cual usa un albuterol en El experimenta asma síntomas un
promedio de dos veces a la semana, por lo que su inhalador de albuterol
proporciona alivio. El paciente ha tenido despertares durante la noche durante el
último mes y nunca ha sido hospitalizado por asma exacerbación. Él no usa tabaco,
alcohol o drogas ilícitas. Su historia familiar es significativa para el asma en su
abuelo. Los signos vitales son normales La auscultación pulmonar indica sonidos
normales sin sibilancias Los sonidos del corazón son normales. ¿Cuál de los
siguientes es el siguiente paso más apropiado? en el manejo de este paciente?

A. Agregue corticosteroides inhalados [14%]


B. Agregue beta-2 de acción prolongada inhalador agonista [4%]
C. Agregar prednisona oral [0%]
D. Agregar teofilina oral [0%]
E. Continuar con la corriente régimen médico (80%)

Una mujer afroamericana de 33 años de edad llega a la oficina debido a las


lesiones de piel y dolor en los tobillos durante una semana. Ella ha tenido no hay
tos, dolor de garganta, dificultad para respirar, dolor abdominal o arco síntomas. Ella
no divulga ningún viaje reciente o enfermedad. El paciente no tiene otros historiales
médicos significativo y no medicamentos. Ella no utiliza tabaco, alcohol o drogas
ilícitas. Ella no tiene ningún historia de las enfermedades de transmisión sexual y
está casado con una pareja monógama durante los últimos 8 años su madre fue
diagnosticada con cáncer de ovario a los 65 años. Temperatura es de 37,2 C (98,9
F) y la presión arterial es 126/76 mmHg. En la examinación, el paciente tiene
múltiples tierno rosado a rojizo nódulos debajo de la rodilla, como se muestra en la
exposición. Cuál de las siguientes es la más apropiada ¿siguiente paso en la
administración?

A. Los anticuerpos antinucleares 36%


B. B. biopsia de las lesiones de la piel [5%]
C. Rayos x de Tórax [47%]
D. D. colonoscopia y biopsias al azar 13%]
E. E. tomografía computarizada del abdomen 12%]
F. la prueba del VIH 12%]
G. Frotis uretral y rectal

Explicación: esta imagen representa a eritema nodoso (EN), que se caracteriza


por nódulos subcutáneos, dolorosos que son más comunes en la parte anterior
inferior piernas. Artralgias y malestar general pueden desarrollar junto a los
nódulos. EN se piensa para representar una reacción de hipersensibilidad
retardada a antígenos asociados con varias condiciones y suele relativamente
benigna con la uno mismo-resolución en varias semanas. Sin embargo, EN
puede ser una señal temprana de más enfermedad grave y la identificación de la
causa pueden evitar morbilidad. Enfermedades asociadas EN incluyen infección
estreptocócica, sarcoidosis, tuberculosis (TB), coccidioidomicosis, inflamatoria
enfermedad intestinal (lBD) y enfermedad de Behcet.

Un hombre de 51 años de edad se queja de dificultad para caminar y dolor en el


pie derecho a un lado suave para el pasado varias semanas. Historia clínica del
paciente es importante para diabetes mellitus tipo 1, hipertensión y la
hipercolesterolemia. El examen físico muestra que un nificantly deformado pie
derecho y un pie izquierdo ligeramente deformado. rayos x del pie derecho indican
efusiones en varios de los articulaciones tarsometatarsianas, grandes osteofitos y
varios fragmentos de hueso articular. Que de la ¿siguientes es la causa más
probable de las quejas de este paciente?

A. La necrosis Avascular 16%]


B. Desmineralización del hueso 2%
C. infección bacteriana [17%]
D. hiperuricemia (10%)
E. daño al nervio [50%]
F La artritis reumatoide [14%]

Más personas parecen estar sufriendo de leucemia en la ciudad A que en la


ciudad B. ¿Cuál de los siguientes Los diseños de estudio serían los más adecuados
para determinar si existe una diferencia en la incidencia de leucemia en estas 2
ciudades?
A. Serie de casos 4%]
B. Caso-control 11%]
C. Ensayo clínico 0%]
D. Cohorte 49%]
E. Transversal 33%]

Una mujer de 35 años acude a la consulta debido a úlceras orales. Ella tiene
múltiples llagas dolorosas en su mucosa oral que comenzó hace unos días. El
paciente tuvo lesiones similares hace 3 meses que sanó sin cicatrices Además,
recientemente fue evaluada por un oftalmólogo para visión borrosa y fue
diagnosticado con uveítis anterior. Ella también ha tenido lesiones genitales
recurrentes que requieren frecuentes citas ginecológicas durante el último año. En
el examen, hay visible oral ulceraciones, así como también lesiones cutáneas
hiperpigmentadas dispersas y áreas sensibles e induradas en ella piernas. ¿Cuál
de los siguientes es el diagnóstico más probable?

A. Síndrome de Behcet [72%]


B. Herpes infección simple [7%]
C. Artritis reactiva [6%]
D. Sarcoidosis (6%)
E. Lupus sistémico eritematoso [6%]

Una mujer de 65 años acude al servicio de urgencias debido a un historial de


estreñimiento de 2 semanas y dolor abdominal. El paciente también tiene
frecuencia urinaria y sed constante. Su historial médico es significativo para la
fibrilación auricular persistente tratada con diltiazem y rivaroxaban, hipertensión
tratados con hidroclorotiazida e hipotiroidismo tratados con levotiroxina. además, el
el paciente fue diagnosticado con osteoporosis hace 3 meses; ella rechazó el
tratamiento con bisfosfonatos y en su lugar optó por tratar solo con suplementos de
vitaminas y minerales de venta libre. Ella no usa tabaco o alcohol En el examen
físico, la temperatura es de 36.8 C (98.2 F), la presión arterial es 120/70 mm Hg, el
pulso es 90 / min e irregular, y las respiraciones son 12 / min. Las membranas
mucosas están secas, y el abdomen es suave y no doloroso sin sensibilidad o
rigidez de rebote. Los sonidos intestinales son disminuidos pero presentes. El
análisis de orina con tira reactiva está dentro de los límites normales. Cuál de los
siguientes es la causa más probable de los síntomas actuales de este paciente?

A. Efecto adverso de diltiazem 17%


B. Cetoacidosis diabética 12%
C. Hipotiroidismo [6%]
D. Síndrome de leche-álcali [33%]
E. Hipercalcemia inducida por tiazida 43%
F. Toxicidad de vitamina A [6%]

Un hombre de 29 años llega a la oficina debido a 10 días de fatiga y copiosa


diarrea acuosa. Él tuvo sin fiebre, heces con sangre o moco, tenesmo o vómitos. El
paciente regresó hace 9 días de un Viaje de 3 semanas a Europa del Este. Durante
este viaje, pasó mucho tiempo caminando y nadando en lagos y corrientes. Él no
tiene problemas médicos crónicos y no toma medicamentos. El paciente no usa
tabaco, alcohol o drogas ilícitas. La temperatura es 37.2 C (98.9 F), la presión
arterial es 120/74 mm Hg, el pulso es 80 / min, y las respiraciones son 15 / min. El
cofre está claro para la auscultación. El abdomen es suave y no sensible con
aumento de los ruidos intestinales y sin organomegalia. El taburete es negativo
leucocitos y sangre oculta en materia fecal. ¿Cuál de los siguientes patógenos es el
responsable más probable de los síntomas de este paciente?

A. Clostridium difficile (2%)


B. Cryptosporidium parvum [46%]
C. Entamoeba histolytica [27%]
D. Shigella flexneri 15%]
E. Strongyloides stercoralis [17%

Explicación: Aunque los patógenos bacterianos y virales son la causa más común
de los viajeros diarrea, organismos parásitos como Cryptosporidium parvum,
Cyclospora y Giardia son responsable de la mayoría de los casos de diarrea acuosa
profusa y prolongada. C parvum es un intracelular protozoario transmitido a través
de la ingestión de agua contaminada (beber, nadar). Lanzamiento de ovocitos
esporozoitos que penetran las células epiteliales intestinales, lo que resulta en una
arquitectura vellosa alterada. Aunque una minoría de pacientes permanece
asintomática, la mayoría se desarrolla leve o profusa, acuosa Diarrea. Malestar,
náuseas, dolor abdominal tipo calambre y fiebre leve también pueden ocurrir las
pruebas de laboratorio rara vez son positivas para los leucocitos o la sangre, pero la
microscopía con las manchas usualmente son diagnósticas (los óvulos fecales de
rutina y las pruebas de parásitos no identifican a C parvum). Los adultos sanos
generalmente tienen una resolución espontánea de los síntomas dentro de los 10 a
14 días; sin embargo, los pacientes inmunocomprometidos (p. ej., SIDA) corren el
riesgo de padecer una enfermedad crónica grave.

Un hombre de 36 años de edad llega a la clínica con una historia de 2 días de
rastros de sangre en el papel higíenico después de la defecación. No tiene bruto
sangre mezclada con heces, pero en una ocasión se eran gotas de sangre en el
inodoro después de la defecación. El paciente se siente bien. Médicos del pasado la
historia es unremarkable, y el paciente no tiene ningún antecedente de malignidad.
Signos vitales son normal. El abdomen es suave y nontender con sonidos
intestinales normales y no hay hepatoesplenomegalia o masas. Examen rectal
digital es unremarkable salvo rastros de sangre roja visible en la guante. ¿Cuál de
los siguientes es el paso siguiente más adecuado en la gestión? C

A. Anoscopia [75%]
B. Prueba de sangre oculta
C. colonoscopia [10%]
D. Fecal de B. bario enema [1%]]
E. sigmoidoscopia [7%]

Una mujer de 45 años de edad llega a la oficina debido a una historia de disnea
progresiva del exertional 3 meses y tos no productiva. Ella no ha tenido edema de
pedales o dolor de pecho, pero informes de episodios intermitentes de dificultad
para tragar. El paciente tiene antecedentes de hipertensión y gastroesofágico
severo reflujo y tiene fenómeno de Raynaud por 2 años. Ella no usa tabaco o
alcohol. Sangre presión es 142/86 mmHg y pulso 80 por minuto el examen físico
muestra la piel sobre su cara, manos, brazos y tronco grueso y firme. Crepitaciones
dispersas se escuchan en la auscultación pulmonar. ¿Cuál de las siguientes es la
causa más probable de los síntomas actuales de este paciente?

A. químico enfermedad de pulmón


B. intersticial neumonitis irritante [67%]
C. izquierda ventricular diastólica disfunción [4%]
D. obstrucción de la pared de pecho restringido
E. medianos y pequeños bronquiolos [3%] [22%]

Una mujer de 35 años llega al médico con una historia de 5 meses de dolor
retrosternal episódica irradia a la región Interescapular. Los episodios suelen duran
15 minutos, aunque se OCC puede durar unas horas y son precipitados por el
estrés emocional y alimentos calientes o fríos. Ella regurgita el alimento
intermitentemente. Antecedentes del paciente es irrelevante, y ella lleva ninguna
medicación allí es no hay historia familiar de enfermedad coronaria. Sus signos
vitales están dentro de límites normales. Phys examen no demuestra ninguna
anormalidad. Un perfil lipídico está dentro de límites normales. Un
Electrocardiograma (ECG) durante un episodio de dolor muestra un ritmo sinusal
normal sin segmento ST cambios. Tabletas de nitroglicerina sublingual alivia el
dolor. Una prueba de esfuerzo es incapaz de reproducir el síntomas o inducir STIT.
cambios de la onda. Radiografía de tórax, endoscopia digestiva alta, y la
ecocardiografía no muestra ninguna anormalidad. Cuál de los siguientes es el paso
siguiente más adecuado en ¿gerencia de este paciente?

A. pH de 24 horas, monitoreo 12%]


B. angiografía coronaria 4%]
C. tomografía computarizada de tórax con contraste 13%]
D. estudios de Motilidad esofágica de D. [79%]
E. manometría de esófago

Explicación: los síntomas de esta paciente (es decir, dolor espontáneo, odinofagia
para alimentos fríos y calientes) son sugestivos de espasmo del esófago difuso.
Resolución de su dolor en el pecho después de tomar nitroglicerina también es
consistente con el diagnóstico. Nitratos (y antagonistas del calcio) relajación no sólo
miocitos en los vasos coronarios, sino también aquellos en el esófago, aliviando así
el dolor. Esophagography puede o no puede mostrar otras anomalías (por ejemplo,
forma de sacacorchos). Manometría esofágica debe mostrar repetitiva, no
peristálticas, de gran amplitud contracciones, ya sea espontáneamente o después
de la estimulación de la ergonovina. (Opción A) Radiación del dolor a la espalda y
en su precipitación por estrés emocional hacen el diagnóstico de la motilidad
trastorno más probable que la enfermedad por reflujo gastroesofágico (ERGE).
GERD commonl más incómodo ardor (acidez) en lugar de dolor que se irradia y se
asocia con esofagitis en la endoscopia. Además, si se sospecha de ERGE, el
paciente debe iniciar un juicio empírico de un protón inhibidor de la bomba en lugar
de monitoreo del pH de 24 horas.

 1 de 2 un hospital de atención terciaria somete a una revisión independiente de


las reclamaciones de negligencia relacionados con diagnosis faltadas o retrasadas
que han ocurrido en los últimos 10 años en el servicio de urgencias. Una gran
proporción de estos errores condujeron a graves daños, con aproximadamente la
mitad dando por resultado muerte. La revisión concluye que la tasa de estos errores
en el servicio de urgencias del hospital 25% mayor que en los hospitales
comparables en todo el país. La administración del hospital convoca un Comité
interno para abordar este tema. Cuál de los siguientes es el subyacente más
probable ¿causa de la mayoría de estos errores médicos?
Fallas en la

A. comunicación entre los proveedores [76%]


C. Departamento de la emergencia alta urgencias
B. alta agudeza 13%]
D. Falta de computarizado el paciente entrega herramientas [2%]
E. falta de respuesta correcta de signout detalladas notas [7%]

Explicación: Discontinuidad es una realidad lamentable pero inevitable de la


atención hospitalaria, con múltiples proveedores de asumir la responsabilidad de
cuidado del paciente en diferentes momentos del día. El proceso de transferir la
responsabilidad para la atención médica se conoce como una entrega paciente, con
"signout" refiriéndose al proceso de transmitir información sobre el paciente y el
seguimiento necesarios. Descuidos y fallas en la comunicación durante el proceso
de signout y entrega se han relacionado con eventos clínicos adversos en una gama
de ajustes del hospital. Omisión de información clave durante las entregas a
menudo resulta en percances, escalamientos evitables en el cuidado (por ejemplo,
la transferencia a la intensa unidad de cuidados), las ineficiencias en la atención y
retrasos en el diagnóstico o tratamiento.

 EM 2 de la 2 después de una prolongada investigación, el hospital determina que


la mayoría de los perdidas o dela urgencias diagnósticos están directamente
relacionadas con fallas en la comunicación entre médico durante el proceso de
signout. Cuál de las siguientes se ha encontrado que la mayoría efectivamente
¿mejorar la comunicación de información relevante durante las transferencias de
pacientes?

A. implementar un signout lista de verificación 55%


B. inclusión de enfermeras y trabajadores sociales signout [9%]
C. aumento de la duración del tiempo asignado para signout 1 %
D. ordene un procedimiento signout cara a cara [27%]
E. Minimizar la respuesta correcta interrupciones [6%]

Explicación: errores médicos derivados de la comunicación fallas entre proveedores


de servicios médicos se tratan más eficazmente mediante la implementación de una
sistemática SignOut proceso que incluye listas de comprobación para mejorar la
eficacia y precisión. Las transferencias de pacientes debe ser realizada verbalmente
y complementado con un componente escrito actualizado y claro. Idealmente, las
transferencias ocurre cara a cara (con interrupciones mínimas), pero esto no se ha
encontrado para ser tan efectiva como la aplicación de listas de comprobación
sistemáticas como parte del procedimiento signout (opciones D y E). componentes
esenciales de una lista de comprobación eficaz signout incluyen no resucitar /
intubar Estado, curso del hospital y los acontecimientos recientes, condición actual e
información anticipada (por ejemplo, si cambios en el estado mental del paciente,
comprobar un gas de sangre y considerar categorías sistemáticamente a asegurar
que ninguna información importante se pierde. (Opción B) Aunque realizar signout
con otros los miembros del equipo pueden proporcionar información adicional o
dudas sobre el paciente, esto sería poco práctico. Rondas interdisciplinarios sería
marco más adecuado para todo el equipo a examinar al paciente en mayor longitud,
incluyendo un cuidado de seguimiento y planificación de descarga (opción C)
Aunque los médicos deben designar una cantidad adecuada de tiempo para salidas
paciente aumentó tiempo para las transferencias de pacientes no necesariamente
conduce a signout mejora calidad o mejores resultados

Un nuevo régimen de la quimioterapia del multifarmacia prolonga


significativamente la supervivencia en pacientes con pulmón cáncer. Si este nuevo
régimen se aplica ampliamente, qué cambios en la prevalencia e incidencia de
¿cáncer de pulmón le esperan?

A. disminuirá la incidencia de, la prevalencia reducirá 1%


B. Incidencia aumentará, voluntad de prevalencia no aumento 1%
C. incidencia no va a cambiar, la prevalencia aumentará el 92%
D. la prevalencia no va a cambiar [2%] voluntad de incidencia de
E. disminución, prevalencia aumentará [1%] tiempo:
Explicación: es importante entender el diferencia entre prevalencia e incidencia, las
dos medidas básicas de ocurrencia. Incidencia es la medida de los nuevos casos, la
rapidez con que son enfermedad medida número de casos en un determinado punto
de tiempo diagnosticado. La prevalencia es la incidencia de la respuesta a la
pregunta: Cómo ¿muchos nueva enfermedad se convirtió en una población durante
un periodo determinado de tiempo? Respuestas de prevalencia el pregunta: muchos
de la existencia de una población en una punto particular relación prevalencia ¿y la
incidencia puede ser en el tiempo? Lo expresado como prevalencia (incidencia) x
(tiempo) la incidencia de un la enfermedad no se modifica por cualquier tipo de
tratamiento, porque la enfermedad ya se ha producido cuando se inicia tratamiento.
Por otro lado, la prevalencia puede verse afectada por el tratamiento de la
enfermedad En este caso, el tratamiento de una enfermedad aguda y rápidamente
mortal (p. ej., cáncer de pulmón) en prolongada el supervivencia de los pacientes;
sin embargo, este tipo de tratamiento no cura la enfermedad. Esto resultará más
personas que tienen dicha enfermedad en un momento en el tiempo; por lo tanto,
aumentará la prevalencia

Una mujer de 62 años de edad llega a la oficina por 4 días de diarrea acuosa y
calambres abdominales. Ella ha tenido 4 o 5 deposiciones sueltas diarias sin sangre
ni moco. Ella tiene antecedentes de estreñimiento que requieren el uso frecuente de
laxante pero último tomó un laxante hace 6 días. El paciente también tiene una
historia de diabetes mellitus tipo 2, hipertensión, hiperlipidemia y reflujo
gastroesofágico. Varios pólipos de colon benignos fueron quitados durante una
colonoscopia hace 2 años. Ella toma omeprazol todos los días y recibido antibióticos
orales para la sinusitis aguda hace 3 semanas. El paciente no ha viajado
recientemente. Ella fuma un paquete de cigarrillos diario. Su temperatura es de 38 C
(100,4 F). El abdomen es suave, con distensión leve y suave ternura difusa. No
guardar o rebote ternura está presente. Taburete por Clostridium difficile es positivo.
Uso de antibiótico además de reciente, que de la ¿siguiente más probable es que
predispuso a este paciente a su condición actual?

A. uso de laxante crónico de (34%]


B. polyps colónicos de la
C. del cigarrillo fumar [6%]
D. gástrico supresión ácida [55%] toma omeprazol todos los días
E. Nervio esplácnico daño 12%]

Un hombre de 51 años de edad llega a la a picazón frecuente, especialmente


después de bañarse. De lo contrario se siente bien salvo ocasionales dolores de
cabeza y mareos. No tiene ninguna pérdida de fiebre o peso. El paciente ha no
visitado un médico en 10 años y no toma medicamentos. Él no usa tabaco, alcohol o
drogas ilícitas. Hermana de la paciente tiene un desorden bipolar, y ambos padres
murieron en un accidente de coche hace muchos años. Temperatura es de 36,7 c
(98 F), la presión arterial es 140/80 mm Hg, pulso es 76/min, y respiraciones son
14/min IMC 30 kg/m2. Su rostro tiene un aspecto cianótico ruddy. orofaringe es
Claro venosa yugular. No hay ninguna linfadenopatía en su cervical, supraclavicular
o axilar cadenas. es suave y la presión es normal. Examen cardiopulmonar no
demuestra ninguna anormalidad. Abdomen objetiva; el bazo es palpable 4 cm por
debajo de la costilla inferior izquierda. Resultados de laboratorio son como sigue:
hemoglobina 21,5 g/dL hematocrito 64% 521.000/mm3 plaquetas leucocitos
14.800/mm que de ¿el siguiente es más probable ser visto en workup adicional?
Sedimentación de eritrocitos elevados

A. tasa de hierro 8%
B. elevada del suero los niveles de saturación de oxígeno arterial de reposo
C.
D. nivel de eritropoyetina sérica o proteína de fusión de
E. positivo BCR-ABL [74%]

Un hombre de 56 años oliguria tres días después de un trasplante. Su curso


postoperatorio era sin complicaciones. Su presión arterial es 160/100 tasa 90/min
palpación del trasplante revela suave dulzura. Estudios de laboratorio muestran: 145
suero sodio mEq/L. suero potasio 5.5 mEq suero calcio 8,6 mg/dL. Creatinina sérica
3,2 mg/dL. 30 mg/dL BUN su nivel de ciclosporina sérica es normal. Ecografía renal
no detecta la dilatación de los con cálices. Biopsia del trasplante muestra infiltración
linfocitaria pesado y la hinchazón de la implicación vascular de íntima. Que de la
¿siguiente es el paso siguiente más adecuado en la gestión? La

A. disminuir la dosis de ciclosporina 12%.


B. dar diuréticos de ASA
D. administrar IV de IV esteroides [77%]
C. ureterography 12%]
E. Preparar para cirugía [15%] tiempo: segundos 1 respuesta correcta 77% es
omitieron R última actualizada: 12/08/2017 este correctamente contestado
explicación: disfunción del trasplante Renal en los primeros postoperatorio se
manifiesta como oliguria, hipertensión y aumento de creatinina/BUN. Puede ser
explicada por un número de causas, que incluyen la obstrucción ureteral, rechazo
agudo, ciclosporina obstrucción vascular, necrosis tubular aguda, toxicidad, etcetera
en este caso, del paciente clínico signos, síntomas y hallazgos de laboratorio son
sugestivos de rechazo agudo. Institución rápida de terapia antirrechazo, incluyendo
los esteroides de la alto-dosis IV, es importante.

Un hombre de 24 años llega al servicio de urgencias debido a unas horas de


fiebre con rigores, dolor de cabeza y mialgia. Mientras que en un centro de atención
de urgencia ayer por la noche, se le diagnosticó sífilis y recibió una dosis de
benzatina intramuscular penicilina G. El paciente tiene antecedentes de VIH
infección por relaciones heterosexuales sin protección y un reciente recuento de
cD4 era 540/mm3. Lo es no está tomando terapia antirretroviral y no sobre cualquier
otro medicamento. Continúa tener relaciones sin protección relaciones
heterosexuales con varios socios a pesar de la asesoría. El paciente no usar tabaco,
alcohol o drogas ilícitas. Temperatura es de 38 C (100,4 F), la presión arterial es
90/60 mm Hg y el pulso es 110/min. El paciente es diaforético y aparece incómodo.
Una difusa, existe débil erupción macular con palmas y plantas. Examen genital
muestra una objetiva úlcera con induración y varios había agrandado nodos de linfa
inguinales. Cuál de las siguientes es la más ¿causa probable de la condición actual
de este paciente?
A. reacción anafiláctica a la penicilina [6%]
B. Co infección con Neisseria gonorrhoeae [4%]
C. progresión debido a la resistencia [7%]
D. Lisis rápida
E. de los síndrome inflamatorio [12%]
F. posttreatment reconstitución inmunitaria de la espiroqueta 68%

Una mujer de 24 años de edad llega a la oficina con una historia de 4 semanas de
dolor en las articulaciones. Ella ha tenido moderado, dolor e hinchazón en múltiples
articulaciones metacarpofalángicas de ambas manos, con menor dolor en las
muñecas, rodillas y codos. Además, el paciente experimentó fiebres intermitentes en
la Inicio de su dolor, pero ha estado afebril desde entonces. Ella ha tenido alivio de
sus síntomas con ibuprofeno sin receta, pero en la semana ya que ella ha
programado su cita, ella no ha necesaria la medicación como ha resuelto el dolor. El
paciente está casado y tiene 2 hijos. Vital las muestras son normales. El examen
muestra el rango normal de movimiento en todas las articulaciones, probadas con
ninguna rojez, calor, o inflamación en las articulaciones de cualquier mano.
Radiografías de las manos son normales. Cuál de las siguientes es ¿el diagnóstico
más probable?

A. artritis cristalina [1%]


B. fibromialgia [4%]
C. gonocócica artritis [2%]
D. Lyme enfermedad [1%]
E. polimialgia reumática 15%]
F. reumatoide artritis [13%]
G. espondiloartropatía [0%]
H. sistémico lupus eritematoso [6%]
i. artritis Viral I. [65%]

hombre de 62 años con antecedentes de cáncer de vejiga y recurrente h la


emergencia Departamento después de "pasar hacia fuera." El paciente fue
levantarse a orinar poco después el waki cuando él fieltro pudo volver a su cama
antes de perder por varios minutos paciente Electrocardiograma (ECG) en el
momento del ingreso se muestra en la exposición. Cuál de las siguientes
probablemente representa el observado ECG cambios

A. la fibrilación [1%]
B. auricular prematura Beats (4%]
c. deteriorada nodo atrioventricular conducción [83%]
D. deteriorado sinoauricular respuesta correcta automaticidad [7%]
E. Ventricular preexcitación 13%] es 83% tiempo: 2 segundos Omitida de la
última actualización: 06/10/2017 respondió correctamente esta explicación: este
paciente Electrocardiograma (ECG) muestra prolongación progresiva del intervalo
PR a una P no llevó a cabo onda y una "caída ' complejo QRS compatible con
Mobitz auriculoventricular de segundo grado de tipo I Bloque (Av). Bloque de Av
Mobitz tipo l (referido a menudo como bloque de Wenckebach) es una Av
intermitente bloque debido a problemas de conducción en el nodo AV.
Características de Mobitz tipo I AV bloque son: constante P P intervalo PR aumentar
intervalo disminuyendo R-R intervalo grupo paliza (racimos de compás de repetición
seguida de un QRS caído) La prolongación del intervalo PR es el mejor apreciado
por la medición de intervalos PR justo antes y después de la caída ' complejos QRS.
Mobitz bloqueo Av de tipo l es a menudo asintomático y tiene un pronóstico benigno
en pacientes jóvenes con buena función ventricular. Sin embargo, ocasionalmente
puede causar bradicardia significativa con síntomas de gasto cardiaco inadecuado
(por ejemplo, fatiga, mareos, síncope, insuficiencia cardíaca), especialmente en más
viejos pacientes con isquemia aguda o con comorbilidad cardíaca adicional

Una mujer de 31 años de edad llega a la oficina debido a un aumento de peso de


6,8 kg (15 lb) en los últimos dos meses. Ella ha sido incapaz de bajar de peso a
pesar de una dieta rigurosa y ejercicio regular. El paciente también ha
experimentado la debilidad y no se puede levantar pesas que ella era capaz de
levantar antes de la Inicio de sus síntomas. Sus períodos menstruales han sido
irregulares durante los últimos meses y ella ha tenido cada vez mayor ansiedad e
insomnio que se empezó a ver a un psicólogo clínico. Historia médica es irrelevante
.Ella toma vino sólo en raras ocasiones y dejar de fumar 7 años hace después de
una historia 5-embalar-años. En el examen, la presión arterial es 160/100 mm Hg y
el pulso es 88/min y regular. El examen neurológico muestra debilidad muscular
proximal. Morena terminal es en la parte inferior del abdomen. Ayuno los resultados
de laboratorio son los siguientes: 130 mg/dL glucosa sodio 142 mEq/L potasio 3.6
mEq/L. cloruro 98 mEq/L, bicarbonato 28 mEq/L calcio 9,2 mg/dL. TSH 2.2 mIU/L,
cuál de las siguientes es el paso siguiente más adecuado en la evaluación de este
paciente ¿condición?
B. supresión de baja dosis de dexametasona durante la noche
A. temprano-mañana cortisol nivel 39%
D. aldosterona en suero [15%]
C. suero ACTH plasma renin actividad proporción 15%]
E. Testosterona en suero

Un hombre de 22 años acude al médico para una visita preventiva. Él no tiene
médicos significativos historia y no está tomando ningún medicamento. Bebe de 2 a
3 bebidas alcohólicas por semana, no fumar, y no tiene historial de uso de drogas
ilícitas. El paciente no es sexualmente activo, pero dice que su novia quiere que se
realice un examen de detección de enfermedades de transmisión sexual (ETS)
antes de "tomar su relación con el siguiente nivel. "No ha tenido ningún examen
STD previo. ¿Cuál de los siguientes las pruebas de detección están indicadas para
este paciente en este momento?

A. Anticuerpo contra la hepatitis C 12%]


B. Herpes virus simplex 1 y 2 anticuerpos 17%]
C. VIH antígeno p24 y pruebas de anticuerpos 41%]
D. Plasma VIH Prueba de ARN [12%]
E. Reacción en cadena de la polimerasa de la orina para la gonorrea 24%]
F. Investigación de enfermedades venéreas prueba de laboratorio (VDRL)
[12%]
Un hombre de 35 años acude a la consulta con un historial de 5 días de dolor de
espalda, que comenzó al final de un largo día de llevar cajas pesadas mientras se
mudaba a un apartamento nuevo. Desde entonces, ha tenido dolor en la columna
lumbar que irradia hacia la cara posterior de la pantorrilla izquierda y el pie. El
paciente ha asociado con la incontinencia urinaria o intestinal. La historia médica es
notable solo por el veneno recurrente dermatitis de la hiedra. Fuma marihuana con
frecuencia los fines de semana pero no usa tabaco, alcohol o otras drogas ilícitas.
En el examen, los signos vitales son normales. La percepción del dolor y la
temperatura son conservado en la extremidad afectada, así como en el área
perineal. Reflejo anal es normal. Levantamiento la pierna izquierda a 70 grados con
la rodilla retenida provoca dolor ardiente que irradia desde el bajo volver al pie
izquierdo ¿Cuál de los siguientes es el mejor paso siguiente en el manejo de este
paciente?
A. Resonancia magnética de la columna lumbar [39%]
B. Estudios de conducción nerviosa [0%]
C. Programa de ejercicios supervisados [7%]
D. Descompresión quirúrgica (0%)
E. Prueba de fármacos antiinflamatorios no esteroideos (41%)
F. Radiografía de la columna lumbar (10%)

Explicación: Este paciente tiene radiculopatía lumbosacra aguda (ciática), muy


probablemente debido a la compresión de la raíz nerviosa por una hernia discal.
Pacientes mayores o aquellos con una lesión traumática previa también puede tener
compresión de la raíz nerviosa debido a la espondilosis lumbar. Menos causas
comunes incluyen trastornos infecciosos o inflamatorios, lesiones masivas,
trastornos vasculares, y anomalías del desarrollo. Los pacientes con una distensión
lumbar no complicada pueden tener radiación de dolor las nalgas o el muslo
posterior, pero la radiación en la pantorrilla y el pie es más consistente con ciática.
La tracción en la raíz del nervio durante la prueba de elevación de pierna recta
causa empeoramiento o reproducción del dolor La mayoría de los pacientes con
ciática aguda experimentarán una resolución espontánea; por lo tanto, el tratamiento
inicial se centra principalmente en el alivio agudo de los síntomas. Nonsteroidal los
medicamentos antiinflamatorios y el paracetamol son los medicamentos de primera
elección preferidos. Uso a corto plazo de opioides o relajantes musculares se
pueden considerar en pacientes con dolor persistente, pero está asociado con
sedación significativa. La modificación de la actividad es a menudo recomendable,
pero los pacientes deben animado a mantener una actividad física moderada.

Una mujer de 78 años de edad es llevada a urgencias por insomnio y frecuentes


episodios de agitación en los últimos 2 días. El paciente estaba confundido pero
tranquilo al llegar; pero ahora ella es grita en voz alta y trata de sacar sus líneas
intravenosas, quitarse la ropa y huir mientras está siendo examinado. Ella es
residente de un asilo de ancianos. Su historial médico reciente incluye severo
pérdida de memoria y alteración de la marcha. Los problemas médicos crónicos del
paciente incluyen hipertensión, tipo 2 diabetes mellitus, enfermedad de úlcera
péptica y pielonefritis crónica. Su temperatura es 37.2 C (99 F), la presión arterial es
162/96 mm Hg, y el pulso es 95 / min y regular. La oximetría de pulso muestra un
96% saturación de oxígeno en el aire ambiente. El ECG muestra ritmo sinusal
normal, hipertrofia ventricular izquierda, intervalo QTc normal, y sin cambios
isquémicos agudos. Los estudios de laboratorio son los siguientes: 41% Hematocrito
12,000 células / uL Leucocitos 160,000 / uL Plaquetas Sodio 137 mEq / L. 4.8 mEq /
L de potasio 95 Cloruro mEq / L 1,2 mg / dL Creatinina Nitrógeno ureico en sangre
25 mg / dL Análisis de orina muestra trazas de proteína, numerosos leucocitos y
eritrocitos ocasionales. Cuál de las siguientes es la mejor inicial tratamiento para
este paciente?

A. Amitriptilina [1%]
B. Clopidogrel (0%)
C. Haloperidol [68%]
D. Lorazepam 25%
E. Memantina (3%)

Un hombre de 34 años de edad es evaluado por malestar general y palpitaciones


durante las 4 semanas. Él también informa fiebres de bajo grado. Él niega cualquier
contacto enfermo o viaje reciente. El paciente tiene una historia o diabetes mellitus y
una fuerte historia familiar de cáncer de colon. Un soplo holosistólico 2/6 es hea
apéndice. La ecocardiografía muestra una masa móvil de 5 mm unida a la valva
anterior de la mitral y regurgitación mitral leve a moderada. Los hemocultivos crecen
Eikenella corrodens. Cuál o siguiente ¿Es la condición predisponente más probable
a la condición actual de este paciente?

A. Comunicación auricular defecto [7%


B. Poliposis colónica [15%]
C. Jeringas de insulina contaminadas [15%]
D. Periodoncia infección [58%]
E. Infección del tracto urinario [4%

La respuesta correcta es 58% omitida D Tiempo transcurrido 1 segundo Última


actualización: 10/10/201 respondió esto correctamente Explicación: Este paciente
ha desarrollado una infección endocarditis (IE) de la válvula mitral debido a la
infección con Eikenella corrodens. E corrodens pertenece al grupo de organismos
HACEK, que incluye Haemophilus aphrophilus, Aggregatibacter
Actinomycetemcomitans (anteriormente Actinobacillus actinomycetemcomitans),
Cardiobacterium hominis, E corrodens, y Kingella kingae. Estos organismos son
responsables solo de una minoría (aproximadamente 3%) de los casos de IE. E
corrodens es un anaerobio Gram-negativo y un componente común de la
normalidad flora oral humana. IE debido a E corrodens se ve generalmente en el
contexto de una dentición pobre y / o infección periodontal, junto con procedimientos
dentales que implican la manipulación de la encía o mucosa oral (Opción A) Una
variedad de lesiones cardíacas congénitas (p. ej., válvula aórtica bicúspide, patente
ductus arterioso, tetralogía de Fallot, defectos del tabique ventricular) predisponen a
los pacientes a una aumento del riesgo de E. Este aumento del riesgo no es
específico de E corrodens o cualquier otro microorganismos. La presencia de un
defecto del tabique auricular no se considera que aumente el riesgo de desarrollo de
IE (Opción B) Pacientes con lesiones ulcerosas del colon por neoplasia de colon o
La enfermedad inflamatoria intestinal tiene una mayor predilección por desarrollar EI
por Streptococcus gallolyticus (S bovis biotype (Choice C) jeringas de insulina
contaminadas (como puede ser usado por usuarios de drogas intravenosas) puede
causar.
Un hombre de 32 años se queja de dolor y secreción en el ojo derecho. Sus
síntomas comenzaron agudamente en despertar en la mañana. Utiliza lentes de
contacto de uso extendido y ha tenido dificultades para quitarse lentes para la última
semana. Su historial médico pasado es significativo para la obesidad, el dolor de
espalda crónico, asthm y enfermedad de reflujo ácido. En el examen, la descarga
amarilla globular gruesa está presente en el esquina medial e y en los márgenes del
párpado. La córnea está edematosa, turbia y ulcerada, y hay inyección escleral
extensiva, como se muestra en la imagen. ¿Cuál es el diagnóstico más probable en
este ¿paciente?

A. Glaucoma de ángulo cerrado [3%


B. Uveítis anterior [15%]
C. Epiescleritis [13%]
D. Conjuntivitis neumocócica [12%]
E. Queratitis pseudomonal [50%]
F. Hemorragia subconjuntival [1%
G. conjuntivitis viral 12%)

Una mujer de 89 años acude al médico quejándose de una decoloración


episódica de la piel últimos meses Ella informa áreas dispersas de color púrpura
oscuro en sus manos y antebrazos sin dolor asociado o picazón. No tiene fiebre,
escalofríos o pérdida de peso reciente. El paciente es otro Los problemas médicos
incluyen la diabetes controlada por dieta y la osteoartritis de la rodilla derecha
tratada con acetaminofén. El examen físico muestra una piel delgada e
hiperpigmentada con varios tonos púrpura oscuro áreas equimóticas sobre el dorso
de ambos antebrazos. Su abdomen es suave y no sensible. El hígado el lapso es de
8 cm sin un bazo palpable. Los resultados de laboratorio son los siguientes:
Hematocrito 47% 5,800 / UL Glóbulos blancos Plaquetas 220,000 / uL 0.6 mg / dL
Creatinina sérica 350 mg / dL (200-400 normales) Fibrinógeno mg / dL) Tiempo de
protrombina 11 sec International Normalized Ratio 1.0 Tromboplastina parcial
tiempo 25 segundos ¿Cuál de las siguientes es la causa más probable de la queja
de este paciente?

A. Un hueso insuficiencia medular (0%)


B. anticoagulante lúpico 12%]
C. atrofia del tejido conectivo perivascular [72%]
D. Pobre adherencia plaquetaria [14%]
E. Deficiencia de vitamina K 19%]

Un hombre de 68 años acude a la oficina con un historial de dolor e hinchazón en


el pie derecho de 2 días. El dolor comenzó cuando se despertó en la mañana.
Inicialmente, el paciente tenía dolor solo en la dorsiflexión del pie, pero
progresivamente desarrolló dolor con cualquier movimiento en el tobillo o peso. El
paciente no ha tenido síntomas similares antes. Él es normalmente muy activo, y
había pasado una semana andar en bicicleta en Utah 3 semanas antes de esta
presentación. La historia médica es notable para el tipo 2 diabetes mellitus,
hipertensión e hiperlipidemia. La temperatura es 37 C (98.6 F), la presión arterial es
140/90 mm Hg, y el pulso es 90 / min. El examen del pie derecho muestra calor,
hinchazón y sensibilidad sobre la superficie dorsal del pie y alrededor del tobillo
derecho. Tanto la dorsiflexión y La flexión plantar está restringida debido al dolor. La
piel está intacta, sin úlceras ni rupturas. Los pulsos son completo e igual en ambos
pies, y las sensaciones sensoriales son normales. Los rayos X revelan la hinchazón
de los tejidos blandos pequeño derrame de la articulación tibioastragalina y
calcificación crónica del cartílago articular. No existen fracturas o dislocación ¿Cuál
de los siguientes es el diagnóstico más probable?

A. Bacterial infección de la articulación del tobillo 15%]


B. artritis por Pirofosfato de calcio [55%]
C. Ure monosódico artritis (11%)
D. articulación neuropática [5%]
E. fascitis plantar 18%]
F. fractura por estrés [13%]

Un hombre de 21 años acude al servicio de urgencias para evaluar el dolor en el


muslo. Esta mañana, él chocó contra el borde de una mesa en su sala de estar y
ahora tiene hinchazón y dolor en su muslo derecho. Tomó una dosis de
paracetamol, que no alivió el dolor. La historia familiar es significativo para un tío
materno con "problemas de sangrado" que murió después de una lesión menor en
la cabeza año. La presión arterial es de 120/80 mm Hg y la frecuencia cardíaca es
de 98 / min. El examen muestra un Hombre de aspecto incómodo con equimosis
marcada e hinchazón en el cuádriceps derecho. ¿Cuál de lo más probable es que
se encuentre en la historia y el examen posterior de este paciente?

A. Antecedentes familiares de una hermana con fácil contusión [3%]


B. Historial familiar de una hermana con menorragia [5%]
C. Heces positivas para sangre [1%]
D. Incremento de la creatinina [0%]
E. recurrente hemartrosis (87%)
F. Pápulas rojas en el tronco y los labios [1%]

1. Una joven de 14 años es llevada al médico por severa dismenorrea desde el pasado año. La
dismenorrea se acompaña de náusea y vómitos los primeros 2 días de su período menstrual y le
causa pérdida de 2 días de escuela cada mes; la aspirina no le alivia el dolor. Menarquia a los 11
años de edad. Ella es sexualmente activa con un compañero y usa condom para anticoncepción. El
examen pélvico no muestra anormalidades. Los estudios serológicos muestran una hormona
folículo estimulante en una concentración de 7 mlU/mL, la concentración de hormona folículo
estimulante de 7 mlU/mL, ky la hormona estimulante del tiroides de 4.0 uU/mL. Una prueba de
embarazo es negativa. ¿Cuál de los siguientes es el paso siguiente mas apropiado en el manejo?

A) Doxyclina por 10 días


B) Acetaminofén antes de esperada la menstruación
C) Ibuprofén antes de esperada la menstruación
D) Codeína durante la menstruación
E) Terapia con Danazol diariamente

2. Una enfermera es hospitalizada en el centro médico donde trabaja por una apendicectomía. Una
semana después de la cirugía, el asistente del administrador del hospital, pregunta al cirujano quien
realizó el procedimiento acerca del diagnóstico final. Cuál de los siguientes es la respuesta más
apropiada de parte del cirujano?

A) Respuesta ,por medio del acceso del seguro médico al centro médico
B) Respuesta, porque como un empleado del centro médico el administrador tiene acceso a la
información de los pacientes.
C) Respuesta, por la posibilidad de difundir desinformación acerca del paciente.
D) Declina en responder, porque el administrador no es un doctor en medicina.
E) Declina en responder, porque la información es confidencial.

3. Un niño de 8 años con Leucemia linfoblástica aguda ha tenido 3 recaídas en los pasados 2
años. El único tratamiento disponible es quimioterapia experimental. Sin tratamiento el niño
es posible que no sobreviva más de 6 semanas; con el tratamiento, su pronóstico es
desconocido. Los padres no quieren más tratamiento para su hijo y desean llevarlo a casa, el
niño también desea irse a su casa. ¿Cuál de los siguientes es el más apropiado curso de
acción?

A) Dar de alta al niño en contra de la orden médica.


B) Dar de alta al niño como en la forma rutinaria
C) Pedir a la corte una orden de tratamiento
D) Informar a los padres del niño sobre los servicios de protección para negligencia médica.

4. Investigadores de la conducta realizan un estudio de cohorte observacional del efecto de


la detección temprana de pacientes con cáncer de pulmón. Los resultados muestran
pacientes con cáncer del pulmón detectados a través de un grupo de larga vida, siguiendo
el diagnóstico, con otros pacientes con cáncer de pulmón. Un estudio randomizado
separado de cáncer de pulmón no demostró ningún aumento en la sobrevida. Cuál de los
siguientes puntos explica estos resultados dispares?

A) El estudio randomizado crea confusión para indicación de tratamiento.


B) En el estudio de cohorte no hay suficiente peso estadístico .
C) Existe sesgo por el tiempo corto en el estudio observacional.
D) Sesgo por el tiempo largo en el estudio randomizado.
E) Misclasificación de cáncer de pulmón en el estudio randomizado.

5. Se efectúa un estudio para establecer la relación entre la toxicidad del aluminio y el


desarrollo de Enfermedad de demencia tipo Alzheimer. Se escogen 400 pacientes con la
enfermedad y 400 controles no afectados, los cuales fueron entrevistados acerca del uso
de vajillas de aluminio, tanto de los pacientes como de los controles en la 4ª. década de la
vida. Los resultados indican que el relativo uso de vajillas de aluminio es de 2.6 (95% de
intervalo de confidencia de 1.9 – 3.2) Los investigadores concluyen que las vajillas de
aluminio causan demencia tipo Alzheimer. ¿Cuál de siguientes fallas potenciales es la más
probable para invalidar esta conclusión?

A) Sesgo en el diagnóstico
B) Falacia ecológica
C) Pobre peso estadístico
D) No es estadísticamente significativo.
E) Sesgo en los pacientes que fueron llamados.
6. En un estudio sobre administración profiláctica de Isoniazida, los niños de una escuela ,
que tenían prueba de tuberculina+, fueron randomizados y asignados para 2 grupos , uno
con tratamiento con la droga y otro con placebo. Un tercer grupo, consistió en aquellos
no elegidos para entrar en el estudio, que fueron observados. Después de varios años, el
tercer grupo tuvo una alta prevalencia de tuberculosis comparado con el grupo placebo.
¿Cuál es la causa más probable de estos resultados?

A) Sesgo del observado.


B) Efecto placebo
C) Pobre randomización
D) Sesgo de auto selección
E) Sesgo en un estudio no ciego.

7. Una mujer de 18 años, primigrávida con 18 semanas de gestación se le encuentra


antígeno de superficie para hepatitis B+ (HBsAg) en pruebas de rutina. Su esposo fue
diagnosticado con hepatitis B, 3 meses atrás. Ella refiere que le intenta dar leche materna
a su infante. El examen muestra un hígado firme de 12 cms de largo. Las pruebas
sexológicas para hepatitis muestran:

Anti HBsAg negativo


IgM-antiHBcAg positivo
Anti-HaV negativo

¿Cuál de las siguientes opciones terapéuticas se debe administrar al recién nacido para
prevenir la transmisión de hepatitis?

A) Inmunoglobulina de hepatitis B solamente al nacimiento.


B) Vacunación contra Hepatitis B sólo al nacimiento.
C) Inmunoglobulina y vacuna sólo al nacimiento.
D) Inmunoglobulina al nacimiento y vacuna con la cesación de la leche materna.
E) Inmunoglobulina de hepatitis B y vacunación sólo si las pruebas sexológicas
hechas al nacimiento para HBsAg y anti HBsAg son negativas.

8. Un hombre de 55 años tiene disnea de esfuerzo en los últimos 10 meses que le impide
viajar o subir escaleras. El examen del tórax muestra un aumento del diámetro
anteroposterior, existe hiperresonancia a la percusión. Los ruidos respiratorias están
distantes y sibilancias al final de la expiración, respira con dificultad. ¿Cuál de las
siguientes vacunas debe recibir este paciente anualmente?

A) Clostridium tetanii
B) Hemophilus influenza tipo B.
C) Virus de la Hepatitis B
D) Virus de la influenza
E) Estreptococo pneumoniae.

9. Una mujer de 79 años se encuentra en un lugar para cuidos de enfermos crónicos; tiene
fiebre, cefalea, dolor de garganta, mialgias y tos no productiva. T° 39.2°C (102.6 °F).
pulso 104 por min. ,R: 22/ min, P.A. 136/76 mmHg. Apariencia de enferma. Leucocitos
6,200/mm3, N (54%, banda 1%, eos 3%, bas 2%, L 28, M 12%)Una radiografía del tórax no
muestra anormalidades. ¿Con cuál de las siguientes vacunas debe realizarse la
inmunización?

A) BCG
B) Hemophilus influenza tipo b
C) Influenza
D) Pertusis
E) Pneumococo.

10. Un niño de 13 años tiene descarga nasal y prurito de los ojos , cada año al inicio del
verano. Se le practica una prueba de alergia al polen y a los 15’ desarrolla eritema de 15
mm en el sitio de la prueba. ¿ Cuál de los siguientes mecanismos es más probable para el
desarrollo de esta prueba dérmica?

A) Complejos antígeno / anticuerpo formados en los vasos sanguíneos de la piel.


B) Instilación de células fagocíticas en respuesta a la inyección de proteínas extrañas.
C) Liberación de histamina de los mastocitos.
D) Liberación de linfoquinas por los linfocitos sensibilizados reaccionando con
antígenos.
E) Liberación de linfoquinas de los mastocitos.

11. Varón de 23 años quien es HIV+, presenta desde hace 2 semanas historia de dolor
torácico en mitad del esternón que se agrava con las comidas picantes, el dolor no está
relacionado con el ejercicio o posición, y no refiere disfagia. El tratamiento con
bloqueadores H2 no le alivia. Ingiere pastillas de Clotrimazol para candidiasis y
Zidovudina (AZT). Tiene CD4+ y cuenta de linfocitos de 220/mm3. ¿Cuál de los siguientes
pasos es el más apropiado para su manejo?

A) Curso terapéutico con Acyclovir


B) Investigar pH 24 horas
C) Prueba de perfusión ácida
D) Manometría esofágica
E) Esofagoscopía

12. Una infante de 9 meses es llevada al médico a visita de control. Nació a las 33 semanas
de gestación. Su dieta consiste de leche entera y comida de vegetales de baby comercial,
frutas y carne. S estatura es la del 50% de su edad y su peso el 75%. El examen no
muestra anormalidades a no ser por la palidez. Su hematocrito es de 25%. Un frotis de
sangre muestra eritrocitos hipocrómicos, microcíticos. ¿Cuál de las siguientes opciones es
la que mayormente puede prevenir esta condición?

A) Suplemento dietético con vitamina C


B) Suplemento dietético con vitamina E
C) Introducir frutas frescas y vegetales a su dieta a los 3 meses de edad.
D) Regular consumo de fórmula que contenga hierro.
E) Regular consumo de 2% de leche.

13. Una niña africana americana de 3 años de edad, previamente sana, es llevada al Cuarto de
urgencias por epistaxis severa que se controla pobremente al aplicarle presión. Hace 3
meses tuvo otitis media que mejoró después de tratamiento con cloranfenicol oral. Su T°
es 37°C (98°F). Está pálida y tiene numerosas petequias y equimosis. No tiene
linfadenopatía o hepatoesplenomegalia. Estudios de laboratorio muestran:

Hb 4.5 g/dL
Leucocitos 2000/mm3 con 80% de linfocitos maduros.
Reticulocitos 0.2%
Plaquetas 12,000/mm3.

¿Cuál de los siguientes diagnósticos es el más probable para explicar estos hallazgos?

A) Anemia debida a pérdida de sangre


B) Anemia aplásica
C) Crisis aplásica de anemia falciforme
D) Mononucleosis infecciosa.
E) Pancitopenia inducida por virus

14. Un hombre de 40 años es llevado al Cuarto de Urgencias por alucinaciones desde hace 24
horas; él dejó la bebida 3 días atrás. Tiene historia de Diabetes mellitas e Hipertensión.
Ingiere medicación pero no recuerda los nombre de las drogas. Su T° es 37.8°C (100°F),
pulso 110/min, P.A. 165/100 mmHg. Está desorientado, tembloroso y diaforético. La
medida del hígado es de 16 cm y está sensible a la palpación. Los laboratorios muestran:

Hb: 11.2 g/dL


VCM 103 Um3
GB: 5,000/mm3
Plaquetas: 50,000/mm3
Suero:
Bilirrubina 1.3 mg/dL
AST 120 U/L

¿Cuál de los siguientes puntos es el que explica estos hallazgos?

A) Interacción adversa de medicamentos.


B) Hiperesplenismo
C) Médula mieloptísica
D) Destrucción periférica de células
E) Bloque de producción de la médula ósea.

15. Varón de 54 años, se presenta con debilidad generalizada desde hace 6 meses, durante
este período , ha habido una pérdida de peso de 9 kg (20 libras). Si T°37°C (98.6°F) pulso
86/min, mientras que posición de pie 88/min y P.A en decúbito supino es de 146/80
mmHg , de pie 142/80 mmHg. El hígado mide 16 cm en la línea media clavicular derecha.
Hb 10 g/dL, Ht 31%, VCM 72 U3, FSP muestra microcitosis e hipocromia. Prueba de
sangre oculta es +. ¿Cuál de las siguientes causas es la más probable para explicar estos
hallazgos?

A) Pérdida aguda de sangre


B) Leucemia mieloide aguda
C) Leucemia linfocítica crónica
D) Leucemia mieloide crónica
E) Infección por Diphyllobotrium latum
F) Deficiencia enzimática del eritrocito
G) Deficiencia de ácido fólico
H) Hemocromatosis
I) Deficiencia de hierro
J) Rasgo de Beta talasemia

16. Un niño de 6 meses sangra con historia de sangrado fácil, está sangrando por las encías en
varias ocasiones desde hace 2 meses. Una tía materna tiene enfermedad hemorrágica. El
examen físico revela pequeñas equimosis en las piernas. TPT está prolongado, y el TP es
normal. ¿Cuál de los siguientes es el factor deficiente más probable?

A) Factor III
B) Factor VII
C) Factor VIII
D) Factor X
E) Factor XIII

17. Varón , 18 años con enfermedad falciforme acude al Cuarto de Urgencias por respiración
corta desde hace 6 horas. Tiene fiebre de bajo grado, dolores articulares, y rash desde
hace 3 días. Estuvo previamente bien y ha tenido varias crisis vasoclusivas. Toma ácido
fólico regularmente y no ha requerido transfusiones de sangre por 8 años. Su Ht ha
estado estable en 32%. Su T° es 38,1°C (1000.6°F), pulso 120/min, R 36/min, P.A. 120/86
mmHg. Los pulmones están claros para percusión y auscultación. Examen cardíaco no
muestra hallazgos anormales. No tiene hepato ni esplenomegalia. Tiene dolor con el
movimiento de las articulaciones, pero no hay evidencia de artritis. Su Ht es 21%,
leucocitos 11,5000 / mm3, cuenta plaquetaria 450,000 /mm3. Una radiografía de tórax es
normal.
Para llegar a un diagnóstico, ¿cuál es el siguiente paso más apropiado?.

A) Electroforesis de Hemoglobina
B) Tiempo de sangría
C) Cuenta de reticulocitos
D) Prueba de HIV
E) CAT de abdomen

18. En las últimas 8 semanas, una mujer de 66 años con una estenosis severa de la aorta , ha
sentido empeoramiento de su respiración e ingurgitación venosa yugular. Se auscultan
traquidos bilaterales. Se escucha un soplo sistólico, duro, áspero, en la punta del corazón,
se escucha mejor en la base del corazón. Examen para sangre oculta en el excremento es
+. Los estudios de laboratorio muestran:

A) Hemoglobina 7.4 g(dL


VCM 70 Um3
Cuenta de leucocitos 5400/mm3
Cuenta plaquetaria 580,000/mm3
Eritrosedimentación 33mm / H
Un frotis de sangre periférica muestra eritrocitos microcíticos, hipocrómicos, con
moderada poiquilocitosis. ¿Cuál es el diagnóstico más probable?

A) Anemia de la enfermedad crónica


B) Anemia hemolítica autoinmune
C) Anemia por deficiencia de folatos
D) Anemia por deficiencia de hierro
E) Anemia hemolítica microangiopática.

19. A) Pérdida aguda de sangre


B) Leucemia linfoblástica aguda
C) Leucemia mieloide aguda
D) Anemia de la enfermedad crónica
E) Anemia aplásica
F) Hemólisis autoinmune
G) Deficiencia de ácido fólico
H) Deficiencia de hierro
I) S-talasemia
J) Anemia sideroblástica
K) Beta talasemia menor
L) Deficiencia de vitamina B12 (cobalamina)

Para cada paciente con anemia, seleccione el diagnóstico más probable:

Una mujer de 22 años con esquizofrenia ha tenido fatiga y disminución de la energía desde hace 6
meses.
Tiene buen apetito, pero ha rehusado comer vegetales frescos por 1 año porque ha esta oyendo
voces que le dicen que los vegetales podrían ser venenosos para ella. T° 37°C (98.6 °F), pulso 76 /
min, R 18 / min, P.A. 122 /70 mm Hg. Luce pálida. El examen físico y neurológico es normal. Hb 9.8
g/dL, leucocitos 6,000 /mm3.

R: G

Un hombre áfrico americano viene al Cuarto de Urgencias 6 horas después del inicio de un
dolor severo en el tórax, y dolor en ambos muslos, los síntomas ocurrieron mientras estaba
esquiando a una altura visitaba Colorado. Tiene dolor obvio. T° 38°C (104.4 ° F), pulso 110
/min, , R 24 / min, y P.A. 8000 pies de altura. Vive en New York y ha tenido un episodio
previo hace 3 años, mientras 136 / 84. No tiene dolor a la palpación de las vértebras o
muslos.No hay anormalidades al examen cardíaco o pulmonar. Los estudios de laboratorio
muestran:

Hemoglobina 10.5 g/dL


Leucocitos 16,000/ mm3
Reticulocitos 20%
R: I

20. Una mujer de 34 años, previamente en buen estado de salud, es llevada al médico por
fiebre y cefalea de 1 semana. No ha tenido antes ninguna enfermedad. No ingiere
medicamentos. T° 39.3°C (102.8°F), pulso 104 / min. Ella está confusa y orientada sólo en
persona. Al examen físico muestra ictericia de la piel y conjuntivas. Presenta algunas
petequias en el tronco y espalda. No hay linfadenopatía. Examen físico y neurológico no
muestra anormalidades Sangre oculta en heces es +. Los estudios de laboratorio
muestran:
Hematocrito: 32% con eritrocitos fragmentados y
nucleados
Leucocitos: 12,5000/mm3
Cuenta plaquetaria 20,000/mm3
Tiempo protrombina 10”
Tiempo parcial de tromboplastina 30”
Productos de degradación de la fibrina negativo
Suero
Nitrógeno de urea 35 mg/dL
Creatinina 3.0 mg/dL
Bilirrubina
Total 3.0 mg/dL
Directa 0.5 mg/dL
Dehidrogenasa láctica 1000 U/L

Cultivos de sangre y orina son negativos. CAT cerebral no muestra anormalidades.


¿Cuál de los siguientes es el diagnóstico más probable?

A) Coagulación intravascular diseminada


B) Púrpura trombocitopénica inmune
C) ;Meningitis meningocócica
D) Sarcoidosis
E) Lupus eritematoso sistémico
F) Púrpura trombocitopénica trombótica

21. Un varón de 55 años es llevado a una resección quirúrgica de urgencia de 40 cm de


yeyuno proximal gangrenado debido a una trombosis mesentérica. Fumador de 2
paquetes de cigarrillos diariamente por 35 años. Laboratorios preoperatorios mostraron:

Hematocrito 40%
Leucocitos 19,000/mm3
Cuenta plaquetaria 240,000/mm3
Tiempo de protrombina 12”
Tiempo de tromboplastina parcial 30” (N=21-36)

Postoperatoriamente, él recibe heparina y antibióticos de amplio espectro. Los


estudios al 3er día postoperatorios muestran:

Hematocrito 32%
Leucocitos 12,000/mm3
Cuenta plaquetaria 30,000/mm3
Tiempo de protrombina 12.8”
Tiempo de tromboplastina parcial 65”

¿Cuál de los siguientes es el diagnóstico más probable?

A) Insuficiencia adrenal aguda


B) Coagulación intravascular diseminada
C) Trombocitopenia inducida por heparina
D) Púrpura trombocitopénica inmune
E) Púrpura trombocitopénica trombótica

22. Un varón de 65 años es llevado al Cuarto de Urgencias por confusión la cual ha ido en
aumento en los pasados 4 días. En los últimos 3 meses ha perdido 9 kg de peso y dolor de
espalda progresivamente severo. Toma diurético tiazídico para hipertensión moderada.
Luce letárgico. Su P.A. 156/84. El examen muestra dolor a la palpación de la espina
lumbosacra. Los estudios de laboratorio muestran:

Hematocrito 26%
Leucocitos 3200/mm3
Suero
Na+ 144 mEq/L
Cloro 102 mEq/L
K+ 3.7 mEq/L
HCO3- 24 mEq/L
Calcio 16.8 mg/dL
Nitrógeno de urea 9.8 mg/dL
Creatinina 5.9 mg/dL
Fósforo 4.0 mg/dL
Fosfatasa alcalina 30 U/L
Acido úrico 9.8 mg/dL

Una radiografía de tórax no muestra anormalidades. ¿Cuál de los siguientes es el


diagnóstico más probable?

A) Carcinoma de páncreas
B) Hipercalcemia inducida por drogas
C) Mieloma múltiple
D) Hipertiroidismo primario
E) Carcinoma de células renales.

Para cada paciente con linfadenopatía seleccione el diagnóstico más probable.

A) Leucemia linfocítica crónica


B) Reacción a droga
C) Enfermedad de Hodgkin
D) Mononucleosis Infecciosa
E) Carcinoma metastático
F) Sarcoidosis
G) Lupus eritematoso sistémico
H) Toxoplasmosis
I) Tuberculosis
J) Tularemia

Un varón previamente saludable ha tenido fiebre, sudores nocturnos, prurito y una masa arriba de
su clavícula izquierda, desde hace 3 semanas. El examen muestra un nódulo linfático supraclavicular,
de 3 cms. no doloroso, cauchoso.
Respuesta: C

Una mujer de 41 años acude al médico para examen de control. Está tomando aspirina por cefaleas
crónicas y fenitoína por convulsiones desde hace 2 años. El examen muestra dolor epigástrico
moderado a la palpación y nódulos linfáticos no dolorosos de 3 cms en ambas axilas. Una biopsia de
uno de los nódulos revela hiperplasia.

Respuesta: B

23. un hombre de 68 años , es hospitalizado para tratamiento de neumonía neumocócica. El


examen muestra signos de una consolidación del lóbulo inferior derecho del pulmón y
nódulos discretos, generalizados, pequeños. La punta del bazo se palpa a 7 cms debajo
del reborde costal izquierdo. Los estudios de laboratorio muestran:

Hemoglobina 10.1 g/dL


Cuenta de leucocitos 84,0000/mm3
Cuenta plaquetaria 110,000/mm3

Un frotis de sangre se muestra. ¿Cuál de los siguientes es el diagnóstico más probable?

A) Metaplasma mieloide agnogénica


B) Leucemia linfocítica crónica
C) Leucemia mieloide crónica
D) Infección por el virus de Epstein Barr
E) Enfermedad de Hodgkin
F) Granulocitosis con desviación a la izquierda

24. Una mujer de 28 años, acude al Cuarto de Urgencias debido a que 6 hrs. después del
inicio de insomnio, malestar en el flanco, y constipación que atribuye a cálculo renal. Es
un residente de otro estado y está visitando a unos parientes. Toma oxycodona (6-8
tabletas diariamente) por dolor crónico de espalda, sumatriptan para migrañas ,
amitriptilina (25 mg en la noche) y paroxetine (30 mg / día) por bulimia nervosa. Pulso
100/min, P.A. 130/80; examen muestra diaforesis y pupilas dilatadas. Está alerta y
cooperadora pero luce inquieta. No ha tenido alucinaciones ni ideas suicidas. Se torna
enojada cuando se le pregunta por números telefónicos de sus médicos. La
concentración de amitriptilina sérica es 150 mg/dL. Urinálisis muestra 0-2 eritrocitos
/hpf.
¿Cuál de los siguientes diagnósticos es el más probable para explicar los síntomas?

A) Envenenamiento por anticolinérgicos.


B) Deprivación de opiáceos
C) Cálculo renal
D) Síndrome de serotonina
E) Intoxicación tricíclica Respuesta: B
25. Un hombre de 18 años es traído al Cuarto de Urgencias por unos amigos estar
tambaleante y sentir pesadez. No tiene antecedentes de problemas médicos. Tiene
aliento alcohólico sin evidencia de trauma. Pulso 70/min, R 6/min, P.A. 110/60. Los
pulmones son claros a la auscultación. S1 y S2. normales. Soplo sistólico de eyección. No
se palpa el hígado ni el bazo. Pruebas de heces para sangre oculta negativas. Examen
neurológico difícil de realizar. Reflejos+ bilateralmente. Todas las extremidades
responden al estímulo doloroso. Su concentración de alcohol en la sangre es de 200
mg/dL; las pruebas de toxicología son + para benzodiacepinas en la sangre y embolitos de
cocaína en la orina. El paciente no responde a la infusión con Naloxone, glucosa o
vitamina B1 (tiamina). ¿Cuál de las siguientes es la más probable causa de estos
hallazgos?

A) Solamente alcohol
B) Alcohol y benzodiacepinas
C) Alcohol y cocaína
D) Alcohol y feniciclidina
E) Alcohol y salicilatos.

26. A) Episodio de amnesia inducido por el alcohol


B) Deprivación de alcohol
C) Hipertiroidismo apatético
D) Enfermedad bipolar , depresiva
E) Delirio por condición médica
F) Demencia relacionada con el alcohol
G) Demencia tipo Alzheimer
H) Desorden de ansiedad generalizado
I) Depresión enmascarada
J) Toxicidad medicamentosa
K) Pérdida de la memoria asociada a la edad
L) Hidrocéfalos con presión normal
M) Enfermedad de Parkinson
N) Enfermedad de Pick
O) Pseudodemencia
P) Esquizofrenia residual
Q) Demencia por multi-infartos (vascular)

Para cada paciente con un problema de memoria, seleccione el diagnóstico más probable.

Una mujer de 29 años , con una historia de enfermedad bipolar de 11 años , acude al médico por
pérdida de la memoria en las últimas 2 semanas. Ha tenido dificultad en recordar citas que ella ha
hecho, y en una ocasión, llegó a olvidar el club de salud en el cual ha sido miembro por años. Ha
estado tomando carbonato de litio por 8 años, y ha estado tomando un diurético tiazídico por
aumento de peso perimenstrual en los pasados 3 meses. El examen físico muestra un temblor de
reposo en ambas manos y ataxia moderada. El examen mental demuestra que está orientada en
persona, lugar y tiempo pero retiene solamente 1 objeto de 3, en 5 minutos.

Respuesta: J toxicidad al carbonato de litio

Un hombre de 63 años es llevado al médico por su hija debido a pérdida de la memoria en el


pasado año. Ayer no pudo recordar el nombre de su nieto de 18 meses . Aunque niega que
exista algún problema, ella dice que él se ha vuelto olvidadizo y se torna fácilmente confuso.
No hay historia de abuso de alcohol. Su temperatura es 37°C (98.6°F), pulso 77/min,
respiraciones son 12 / min y P.A.118/84 mmHg. Al examen mental su humor es normal. Está
orientado en persona y lugar, pero inicialmente da el mes errado, el cual es capaz de corregir.
Retiene la memoria desde su juventud en gran detalle, pero sólo retiene una de 3 palabras
después de 5 min. Tiene dificultad reteniendo nombre de objetos comunes y no recuerda e
nombre del actual Presidente de los Estados Unidos. El examen físico , laboratorios y pruebas
de función tiroidea no muestran anormalidades.

Respuesta: G demencia tipo Alzheimer

28. Una adolescente de 17 años previamente saludable es llevado al médico por pérdida del
apetito, insomnio y por irritabilidad extrema desde hace 3 semanas. Se siente con
frecuencia cansada y tiene dificultad para permanecer sentada y concentrarse en su
trabajo escolar. Su menstruación ocurre a intervalos regulares. Mide 168 cm (5’6”) y pesa
50 kg (110 lbs); su índice de masa corporal (BMI) 18 kg/m2. Pulso 74 / min, R 16 / min y
P.A. 110 /70 mmHg.
¿Cuál es el diagnóstico más probable?

A) Reacción de ajuste con disturbio mixto emocional y de conducta


B) Anorexia nervosa
C) Déficit de atención con hiperactividad
D) Enfermedad distímica
E) Enfermedad depresiva mayor

Respuesta: E

29. Una mujer de 54 años acude al médico 2 meses después de que su esposo murió de infarto
del miocardio. Se cansa fácilmente y tiene dolor torácico cuando inhala profundamente en
reposo. Ha tenido frecuentes llantos cuando habla, asociados con pensamientos con su
esposo y siente que no se siente comprometida con sus actividades diarias. ¿Cual es el
diagnóstico más probable?

A) Desorden de conversión
B) Desorden de ansiedad generalizada
C) Desorden depresivo mayor
D) Reacción de duelo o pena normal
E) Reacción de duelo o pena patológica.

Respuesta: D

30. Una mujer de 45 años tiene historia de 2 semanas de ansiedad, dolor abdominal,
irritabilidad, y dificultad en la concentración; a ella le robaron con un cuchillo en un
estacionamiento, hace 3 semanas. Toma Levotirosina para hipotiroidismo y usa un
inhalador para asma. P.A. 140/80 mmHg, y pulso 100/min. El examen muestra pie y
cabello secos. Es cooperadora, pero luce ansiosa, y se sobresalta cuando un ruido se
escucha fuera de la oficina. Leucocitos 12,000/mm3, TSH 5.0 uU/mL. Un ECG muestra
taquicardia sinusal ¿ Cuál de los siguientes es el diagnóstico más probable?

A) Estrès agudo
B) Agorafobia
C) Desorden de ansiedad generalizada
D) Hipotiroidismo
E) Desorden de pànico Respuesta: A

Para cada paciente con ansiedad, seleccione el diagnóstico más probable.

A. Síndrome de Abstinencia Alcohólica G. Taquicardia Paroxística Atrial


B. Insuficiencia Coronaria H. Feocromocitoma
C. Hipertiroidismo I. Estrés Postraumático
D. Hipoglicemia J. Embolismo Pulmonar
E. Trastorno Depresivo Mayor K. Trastorno Somatoforme
F. Trastorno de Pánico

1. Una mujer de 33 años acude al cuarto de urgencias 30 minutos después por presentar dolor
torácico súbito, palpitaciones, respiración entrecortada, adormecimiento y punzadas en ambos
brazos y miedo a volverse loca. Ha visitado el cuarto de urgencias local varias veces los últimos 3
meses por presentar síntomas similares que resuelven a la hora. Utiliza anticonceptivos orales.
Bebe dos cervezas diariamente y seis cervezas los fines de semana. No tiene historia de
problemas médicos. Su madre y hermanas tienen historia de ansiedad. Su pulso es de 90 por
minuto, su respiración de 18 por minuto, su presión arterial de 130/90 mmHg. El examen físico,
los estudios de laboratorio y el electrocardiograma no muestra alteraciones.
¿Cuál es el diagnóstico más probable? Respuesta F (Trastorno de Pánico)

2. Una mujer de 35 años acude al médico por nerviosismo, temblores, labilidad emocional y
excesiva sudoración desde hace 3 semanas; ha tenido una perdida de peso de 4.5Kg (10lbs)
durante este periodo. No tiene antecedentes personales o familiares de enfermedades
psiquiátricas. Su pulso es de 95 por minuto, con 12 respiraciones por minuto y una presión
sanguínea de 120/80 mmHg. El examen muestra piel caliente y húmeda, un fino temblor en sus
dedos y lengua e hiperreflexia.
¿Cuál es el diagnóstico más probable? Respuesta C (Hipertiroidismo)

3. Un niño de 10 años es traído al médico por haber incrementado sus problemas de


comportamiento en la escuela desde el comienzo del quinto grado hace 3 meses. Su maestra
refiere que el no puede permanecer sentado quieto durante el periodo de clases y
frecuentemente interrumpe la clase y a otros niños mientras están hablando. Sus padres
reportan que él ha sido siempre un niño activo y se han interesado porque es inatento cuando
camina o corre. Durante el examen, tiene el afán de mover constantemente sus manos y pies y
es fácilmente distraído para completar una tarea.
¿Cuál de los siguientes es la más apropiada farmacoterapia?

A. Amitriptilina
B. Fluoxetina
C. Haloperidol
D. Imipramina
E. Metilfenidato
Respuesta E Diagnóstico Déficit Atencional/Trastorno de Hiperactividad

4. Un infante de 3 meses de edad es traído al médico por sus padres porque continúa teniendo
fiebre, dolor de oído e irritabilidad después del tratamiento con amoxicilina por 72 horas. Al
examen se evidencia una membrana timpánica inmóvil, roja y opaca.
¿Cuál de los siguientes es el agente etiológico más probable?

A. Chlamydia trachomatis
B. Estreptococo del Grupo A
C. Haemophilus influenzae
D. Mycoplasma pneumoniae
E. Staphylococcus aureus

Respuesta C Diagnóstico Otitis Media

5. Un estudiante de 19 años es traído al cuarto de urgencias por su compañero de cuarto por


presentar dificultad para despertarlo en la mañana. Ha tenido algo parecido a la gripe con
fiebre, dolores musculares y articulares desde hace 12 horas. Su temperatura es 39.5°C
(103.1°F), pulso de 120 por minuto, una presión arterial de 90/60 mmHg. Hay un rash petequial
difuso sobre el tronco y las extremidades. Tiene rigidez nucal que no puede flexionar
pasivamente.
¿Cuál de los siguientes es el agente etiológico más probable?

A. Coxsackievirus B
B. Echovirus
C. Haemophilus influenzae
D. Neisseria meningitidis
E. Streptococcus pneumoniae

Respuesta D Diagnóstico Meningitis

6. Cerca de la semana pasada, una mujer de 55 años ha tenido vértigo extremo al acostarse o
levantarse. Refiere que se siente “como si el cuarto diera vueltas”. El examen no muestra
alteraciones excepto por un nistagmo fatigable hacia el lado izquierdo cuando la paciente se
deja caer sobre su lado izquierdo.
¿Cuál de los siguientes es el diagnóstico más probable?

A. Vértigo Posicional Benigno


B. Enfermedad de Meniere
C. Fístula Perilinfática
D. Tumor del glomus yugular
E. Neuronitis Vestibular

Respuesta A Diagnóstico Vértigo Posicional Benigno


Para cada paciente con ansiedad, seleccione el diagnóstico más probable.

A. Sinusitis Aguda C. Cefalea en Racimos


B. Sinusitis Crónica D. Tumor Intracraneal
E. Meningoencefalitis I. Hemorragia Subaracnoidea
F. Migraña J. Arteritis Temporal
G. Feocromocitoma K. Síndrome Temporomandibular
H. Cefalea Postraumática

7. Un hombre de 43 años acude al cuarto de urgencias a las 3:00am por presentar una cefalea
severa constante desde hace 1 hora. El dolor es localizado detrás de los ojos. También ha
tenido secreción acuosa por la narina izquierda. Ha tenido episodios similares nocturnos desde
de la semana pasada. Tiene historia de síntomas similares en periodos de cada 3 semanas por
los últimos 2 años. Su pupila izquierda es más pequeña en comparación con la derecha y hay
una ptosis palpebral en el lado izquierdo. Se observa una secreción hialina en la narina
izquierda.
¿Cuál es el diagnóstico más probable? Respuesta C (Cefalea en Racimos)

8. Una mujer de 19 años acude al médico por presentar un cuadro de cefaleas recurrentes desde
hace 1 año. Las cefaleas son unilaterales y pulsátiles acompañadas de náuseas, vómitos y
fotosensibilidad. Las cefaleas ocurren una a dos veces al mes alcanzando su máxima intensidad
a la hora con una duración de 12 a 24 horas. No hay aura. El examen no muestra
anormalidades.
¿Cuál es el diagnóstico más probable? Respuesta F (Migraña)

9. Una mujer de 28 años de edad tiene palpitaciones que ocurren aproximadamente desde hace 1
semana, duran aproximadamente de 1 a 5 minutos y consisten de un latido rápido y regular. Los
episodios inician y terminan súbitamente y no son asociados con molestias torácicas o disnea.
No hay historia de problemas cardíacos. Toma 2 a 3 tazas de café diariamente. Raramente
toma alcohol y no fuma. Su pulso es de 96 por minuto y regular, su presión arterial es de 120/88
mmHg. Al examen se ha notado exoftalmos, la glándula tiroidea es firme y 1.5 veces su tamaño
normal. Hay un clic medio sistólico en la punta y un soplo sistólico 2/6 en el borde esternal
izquierdo. El electrocardiograma es normal excepto por taquicardia sinusal.
¿Cuál de los siguientes es el paso más apropiado en el diagnóstico?

A. Monitoreo Electrocardiográfico Ambulatorio


B. Medición Sérica de la Hormona Estimulante del Tiroides
C. Medición de la Concentración de Catecolaminas
D. Centelleo MUGA
E. Ecocardiograma

Respuesta B Diagnóstico Enfermedad de Graves


Para cada paciente con ansiedad, seleccione el diagnóstico más probable.

A. Polineuropatía Diabética G. Enfermedad de Parkinson


B. Enfermedad de Huntington H. Glioma Pontino
C. Síndrome Medular Lateral I. Tabes dorsalis
D. Encefalopatía por Plomo J. Encefalopatía de Wernicke
E. Meduloblastoma
F. Esclerosis Múltiple

10. Un hombre de 50 años es llevado al médico por su esposa por presentar historia de 1 mes de
incremento gradual de la confusión mental, desorientación y perdida de la memoria a corto
plazo. El examen no muestra anormalidades excepto por dejar caer su pie izquierdo. Un frotis
de sangre periférica muestra microcitosis y eritrocitos con moteado basofílico.
¿Cuál es el diagnóstico más probable? Respuesta D (Encefalopatía por Plomo)

11. Una joven de 20 años es llevada al cuarto de urgencias 30 minutos después de haber presentado
visión doble en forma aguda en su ojo izquierdo. Hace 2 meses, ha tenido un episodio agudo de
dolor al movimiento del ojo izquierdo con perdida de la visión tanto central como del reflejo
pupilar a la luz; estos síntomas mejoraron espontáneamente. El examen muestra un retraso en
la adducción del ojo izquierdo y nistagmo en el ojo derecho al voltear hacia el lado derecho.
¿Cuál es el diagnóstico más probable? Respuesta F (Esclerosis Múltiple)

12. Un hombre de 57 años acude al cuarto de urgencias por presentar un esputo sanguinolento
desde hace 2 semanas. Tiene una historia de 6 meses presentando disnea excepcional
especialmente cuando camina cuesta arriba o sube las escaleras. No toma ninguna medicación y
no fuma. Es alérgico a la penicilina. Su pulso es de 88 por minuto, respiraciones de 16 por
minuto, con presión arterial de 120/80 mmHg. Se auscultan crépitos en ambas bases
pulmonares y un soplo diastólico puede ser auscultado en el ápice cardíaco. El
electrocardiograma muestra una ancha muesca de la onda P. La radiografía de tórax muestra
redistribución de los vasos pulmonares hacia los lóbulos superiores.
¿Cuál de los siguientes es el paso más apropiado en el diagnóstico?

A. Broncoscopía
B. Angiografía Coronaria
C. Angiografía Pulmonar
D. Ecocardiograma
E. Cateterización de la Arteria Pulmonar

Respuesta D Diagnóstico Estenosis Mitral

13. Un joven de 15 años es traído al cuarto de urgencias una hora después de haber presentado un
episodio de síncope mientras corría en una carrera de 400 metros. Ha tenido episodios similares
hace 2 años. Su madre y primo hermano materno fallecieron súbitamente a las edades de 32 y
17 años, respectivamente. El examen muestra abrasión de su cara, manos y rodillas. El examen
cardiológico y neurológico no evidencia anormalidades.
¿Cuál de los siguientes es el paso más apropiado en el diagnóstico?

A. Prueba de Inclinación
B. Tomografía Cerebral
C. Electrocardiograma
D. Electroencefalograma
E. Punción Lumbar

Respuesta C Diagnóstico Síndrome del Intervalo QT Prolongado

14. Un niño de 10 años con sinusitis crónica es traído al cuarto de urgencias seguido de 3 minutos de
convulsiones tónico-clónicas generalizadas. Tiene una historia de 3 días de incremento severo
de cefaleas y 2 días de vómitos y temperatura de 38.7°C (101.7°F), pulso de 80 por minuto y
respiraciones de 16 por minuto con presión arterial de 135/90 mmHg. Está ligeramente
letárgico pero responde a las preguntas apropiadamente. El examen muestra papiledema. No
se evidencian otras anomalías.
¿Cuál de los siguientes es el paso más apropiado en el manejo?

A. Medición de la Concentración Sérica de Amonio


B. Radiografía de Seno Paranasales
C. Electroencefalograma
D. Tomografía Cerebral
E. Punción Lumbar

Respuesta D Diagnóstico Absceso Cerebral

15. Un hombre de 32 años ha tenido debilidad progresiva en sus brazos y piernas los pasados 4 días.
Se ha sentido bien excepto por una infección del tracto respiratorio superior hace 10 días. Su
temperatura es 37.8°C (100°F), pulso de 94 por minuto, respiración superficial de 42 por minuto
y presión arterial de 130/80 mmHg. Hay una debilidad simétrica en ambos lados de su cara y de
los músculos proximales y distales de las extremidades tanto superiores como inferiores. La
sensibilidad está intacta. No hay reflejos tendinosos profundos; la respuesta plantar es flexora.
¿Cuál de los siguientes es el diagnóstico más probable?

A. Encefalomielitis Aguda Diseminada


B. Síndrome de Guillain – Barré
C. Miastenia Gravis
D. Poliomielitis
E. Polimiositis

Respuesta B Diagnóstico Síndrome de Guillain – Barré


Para cada paciente con ansiedad, seleccione el diagnóstico más probable.

A. Pericarditis Aguda G. Pleuritis


B. Shock Cardiogénico H. Embolismo Pulmonar
C. Pericardiopatía Constrictiva Crónica I. Fiebre Reumática
D. Enfermedad de la Válvula Mitral J. Neumotórax Espontáneo
E. Miocarditis
F. Taponamiento Pericardiaco

16. Un joven de 18 años previamente saludable acude al cuarto de urgencias 12 horas después de
presentar dolor torácico en el área del músculo trapecio. Ha tenido una infección del tracto
respiratorio superior por 9 días. Se auscultan crépitos sobre el precordio. Un
electrocardiograma muestra incremento del punto J en todas las derivadas con excepción de
aVR y V1. Después de la administración de aspirina, el dolor cede.
¿Cuál es el diagnóstico más probable? Respuesta A (Pericarditis Aguda)

17. Una mujer de 42 años acude al cuarto de urgencias 1 hora después de presentar súbitamente
dolor torácico, tos, disnea, taquipnea y marcada ansiedad. Hace 2 días, se le realizó una
hemicolectomía por cáncer en el colon ascendente. Se ausculta un acentuado ruido S2 en la
pulmonar. El electrocardiograma muestra cambios no específicos del segmento ST y de la onda
T. El conteo leucocitario es de 12000/mm3. La radiografía de tórax no muestra infiltrados
pulmonares ni efusión pleural. La gasometría arterial al aire ambiente muestra una PCO2 en 30
mmHg y una PO2 en 55 mmHg.
¿Cuál es el diagnóstico más probable? Respuesta H (Embolismo Pulmonar)

18. Un hombre de 58 años acude al médico por presentar fatiga extrema e indisposición por 3
semanas. Se ha sentido bien excepto por un dolor de muelas hace 5 semanas manejado con un
procedimiento en la raíz del canal. Tiene una historia de un soplo cardíaco notado la primera
vez a la edad de 19 años. Su temperatura es de 37.8°C (100°F), pulso de 110 por minuto y
presión arterial de 120/80 mmHg. Los pulmones se auscultan claros. El examen cardíaco
muestra un soplo sistólico de eyección 2/6 escuchado mejor en el segundo espacio intercostal
derecho también como un S4 y un clic de eyección. Los laboratorios muestran:
Hemoglobina 9.3h/dl Leucocitos 10000/mm3
Veloc. de Sedimentación 90mm/h Neutrofilos Segmentados 90%
Hematuria Positivo Bandas 10%

El Hemocultivo es obtenido. ¿Cuál tinción de Gram del Hemocultivo es el más probable?

A. Cocos en Cadenas Gram Positivos


B. Cocos en Racimos Gram Positivos
C. Diplococos Gram Positivos
D. Bacilos en cadenas Gram Negativos
E. Diplococos Gram Negativos

Respuesta A Diagnóstico Endocarditis Infecciosa

19. Una joven de 14 años moderadamente obesa tiene historia de 2 semanas presentando cefaleas
bifrontal severa y vómitos temprano en la mañana. Aparece alerta y cooperadora. Ella es
diestra. Su pulso es de 82 por minuto y la presión arterial es 112/76 mmHg. El fondo de ojo
evidencia lo de la figura. Su agudeza visual es de 20/20 bilateralmente; el examen neurológico y
el CAT cerebral no evidencia anormalidades.
¿Cuál de los siguientes es el diagnóstico más probable?

A. Migraña
B. Neuritis Óptica
C. Tumor de Fosa Posterior
D. Pseudotumor cerebri
E. Cefaleas Tensional

Respuesta D Diagnóstico Pseudotumor cerebri

20. Un joven de 18 años jugador de básquetbol en la secundaria acude al médico para un examen
físico de rutina. Mide 193cm (6 pies 4 pulgadas); su brazada mide 201cm (79 pulgadas). Tiene
largos dedos en las manos y pies. Su pulso es 64 por minuto, su presión arteria es 146/62
mmHg. Presenta un soplo diastólico de alta frecuencia 2/6 en el segundo espacio intercostal
derecho.
¿Cuál de los siguientes es el diagnóstico más probable?

A. Regurgitación Aórtica
B. Defecto Septal Atrial
C. Coartación de la Aorta
D. Anomalía Ebstein
E. Estenosis Mitral

Respuesta A Diagnóstico Síndrome de Marfan

21. Una mujer de 54 años es traída al cuarto de urgencias por sus familiares por presentar
adormecimiento y parálisis de sus extremidades superior e inferior izquierdos desde hace 2
horas. Tiene historia de hipertensión y abuso de alcohol y ha fumado 2 paquetes de cigarrillos
diarios desde hace 30 años. La sensación está disminuida en la extremidad superior e inferior
izquierdo y la fuerza muscular es 0/5. Se escucha un soplo carotídeo derecho.
¿Cuál de los siguientes es el estudio inicial más apropiado?

A. Electroencefalograma
B. Doppler Carotídeo
C. Tomografía Cerebral
D. Ecocardiograma
E. Angiografía Carotídea

Respuesta C Diagnóstico Infarto Cerebral


Para cada paciente con ansiedad, seleccione el diagnóstico más probable.

A. Angina pectoris M. Hernia Hiatal


B. Disección Aórtica N. Cardiomiopatía Hipertrófica
C. Estenosis Aórtica O. Cáncer Pulmonar
D. Artritis de la Espina P. Enfermedad Metastásica Pulmonar
E. Enfermedad Disco Cervical Q. Válvula Mitral Prolapsada
F. Pericardiopatía Crónica Constrictiva R. Infarto al Miocardio
G. Costocondritis S. Miocarditis
H. Aneurisma Disecante de la Aorta T. Taponamiento Cardíaco
I. Úlcera Duodenal U. Neumonía
J. Espasmo Esofágico V. Neumotórax
K. Síndrome de Atrapamiento de Aire W. Embolismo Pulmonar
L. Herpes zoster

22. Un hombre de 37 años acude al cuarto de urgencias 1 hora después de presentar un dolor
torácico anterior. El dolor se irradia a la espalda y brazo izquierdo. Mide 203cm (6 pies 7
pulgadas) y pesa 86Kg (190lbs); IMC es 21Kg/m2. Su temperatura es 36.7°C (98°F), pulso de 116
por minuto y respiración de 20 por minuto, con presión arterial en 115/70 mmHg. Los pulmones
se auscultan claros. El electrocardiograma es normal.
¿Cuál es el diagnóstico más probable? Respuesta B (Disección Aórtica)

23. Un hombre de 72 años acude al cuarto de urgencias 2 horas después de haber presentado
súbitamente dolor torácico anterior y respiración entrecortada. El dolor es exacerbado por la
respiración profunda. Le fue diagnosticado cáncer prostático metastásico hace 6 meses atrás.
Su temperatura es 37.2°C (98.9°F), pulso de 136 por minuto, respiración de 28 por minuto y la
presión arterial es 100/56 mmHg. A la auscultación cardiopulmonar no se encuentran
anormalidades. El electrocardiograma muestra una desviación hacia el axis derecho que no se
evidenciaba en el electrocardiograma de hace 1 mes atrás.
¿Cuál es el diagnóstico más probable? Respuesta W (Embolismo Pulmonar)
24.Un recién nacido de 2 semanas es traído al médico porque sus labios se han
tornado azules en tres ocasiones durante la alimentación; además suda
mucho durante la alimentación. Nació a las 38 semanas de gestación y peso
2466g (5lbs 7oz); corrientemente pesa 2778g (6lbs 2oz). Su temperatura es
37.8°C (100°F), pulso de 170 por minuto, respiración de 44 por minuto y
presión arterial de 75/45 mmHg. Se ausculta un soplo rudo sistólico de
eyección 3/6 en el borde esternal superior izquierdo. La radiografía de tórax
evidencia una pequeña figura de botón en el corazón y disminución de la
trama vascular pulmonar.
¿Cuál de los siguientes es el diagnóstico más probable?

A. Vasos Coronarios Anómalos


B. Defecto Atrioseptal
C. Fibroelastosis Endocardial
D. Tetralogía de Fallot
E. Anomalía Total del Retorno Venosos Pulmonar

Respuesta D Diagnóstico Tetralogía de Fallot

25.Una mujer de 64 años de edad viene al médico por historia de 5 meses de


evolución de incremento en dificultad respiratoria, dolor de garganta y tos
productiva con esputo blanco escaso. La pasada semana, ella había tenido
náuseas relacionada a tos excesiva.
En el año pasado, ella ha perdido 7 lbs. Ella tiene asma tratada con teofilina
y agonistas B adrenérgicos y corticoides. Ella ha fumado un paquete de
cigarrillos diario por 44 años y toma una bebida alcohólica diaria. Se observa
delgada. El examen muestra un nódulo de 2cm no doloroso en el área
supraclavicular derecha. El examen no muestra otras anormalidades. Una
radiografía de tórax muestra una gran densidad en lóbulo inferior derecho.
Un CAT de tórax muestra una masa en lóbulo inferior derecho de 7.5 x 7.5 x
6cm con algunas calcificaciones especuladas. La lesión se acerca a la pared
posterior del tórax sin una clara invasión. Hay nódulos linfáticos
peritraqueales derechos, precarinales, hiliares derechos y subcarinales. Hay
una masa de 1.5cm en la glándula adrenal derecha. Una biopsia de la masa
pulmonar demuestra lo siguiente:

A. Linfoma de Células B
B. Absceso Pulmonar
C. Aesotelioma
D. Adenocarcinoma de Mama Metastásico
E. Neoplasia Endocrina Múltiple
F. Carcinoma de Pulmón de Células No Pequeñas
G. Sarcoidosis
H. Tuberculosis

Respuesta F Diagnóstico Cáncer de Pulmón

26.Dos horas después de una colecistectomía sin eventualidades, una mujer de


50 años de edad tuvo extrasístoles y una disminución en la presión sistólica
de 110 a 90 mmHg. Los gases arteriales a aire ambiente muestran: pH
7.30, Pco2 52 mmHg, pO2 58 mmHg
¿Cuál de los siguientes es el diagnóstico más probable?

A. Hipoventilación Alveolar
B. Hemorragia Oculta
C. Fallo e irritabilidad Cardíaca
D. Hipoxemia Primaria Causada por Anestésicos
E. Embolismo Pulmonar

Respuesta A Diagnóstico Hipoventilación Alveolar.

27.Dos días después de una fractura femoral, un hombre de 25 años de edad


llega confundido y disnéico. El examen muestra petequias sobre el tórax. Su
temperatura es 38.2°C, pulso 115 por minuto, y presión sanguínea 110/70
mmHg.
¿Cuál de los siguientes es la causa más probable de estos hallazgos?

A. Reacción adversa a Analgésicos


B. CID (Coagulación Intravascular Diseminada)
C. Embolia Grasa
D. Shock Hipovolémico
E. Trombocitopenia

Respuesta C Diagnóstico Embolia Grasa

28.Un hombre de 21 años de edad es traído al cuarto de urgencias 45 minutos


después de una colisión frontal en vehículo de motor, en la cual el era el
conductor. A su llegada, el está alerta y coherente, respirando oxígeno 100.
El examen muestra equimosis en la porción central del tórax extendido a
ambos lados, las venas del cuello no están distendidas, su tráquea está
desviada a la derecha. Los ruidos respiratorios están presentes en el lado
derecho y ausentes en el izquierdo, luego de la administración de 2 litros de
Lactato Ringer, su presión sistólica es 80 mmHg.
¿Cuál de los siguientes es el más apropiada próximo paso en el manejo?

A. Intubación
B. Inserción de Tubo de Toracostomía Izquierda
C. Infusión de Sangre No Cruzada.
D. Lavado Peritoneal
E. Pericardiocentesis

Respuesta B Diagnóstico Hemotórax

29.Un hombre de 19 años de edad ha tenido malestar general, dolor de


garganta recurrente, anorexia, edema de glándulas, y pérdida de peso de 9
lbs en las últimas 5 semanas, un cultivo de garganta creció Streptococcus
pyogenes (grupo A), y el paciente fue tratado con penicilina. Su temperatura
es de 38.5°C. El examen demuestra faringitis exudativa, linfadenopatía
anterior y cervical posterior, y esplenomegalia. Los laboratorios demuestran:
Hcto 37 %, GB 3200/mm3, Neutrófilos 55%, Linfocitos 34 %, Plaquetas
84,000/mm3, AST sérica 75 u/L.
¿Cuál de los siguientes es el paso diagnóstico más apropiado?
A. Estudios de Anticuerpos Antiplaquetarios
B. Estudios de Antiestreptolisina O
C. Examen de Médula Ósea
D. Tamizaje por Hepatitis
E. Serología por Ebstein Barr Virus

Respuesta B Diagnóstico Infección por Ebstein


Barr virus

30.Un hombre de 22 años de edad con un desorden convulsivo, ha tenido un


incremento en tos y respiración corta, por 3 días y fiebre por 1 día. El tiene
esputo maloliente. El tuvo una convulsión tónico – clónico generalizada una
semana antes. Su temperatura es 39.4°C. Se auscultan crépitos en tórax.
Una radiografía de tórax muestra un infiltrado en lóbulo superior derecho.
¿Cuál de los siguientes es el diagnóstico más probable?

A. Neumonitis Química
B. Neumonía por Anaerobios
C. Neumonía Secundaria a Aerobios Gram Negativos
D. Neumonía Secundaria a Aerobios Gram Positivos
E. Neumonía Secundaria a Micoplasma pneumoniae

Respuesta B Diagnóstico Neumonía por


Aspiración
Para cada paciente de los siguientes con tos, selecciones el Diagnóstico más
probable:

A. Bronquitis Aguda
B. Asma
C. EPOC
D. Falla Cardíaca Congestiva
E. Fibrosis Quística
F. Cáncer Metastásico a Pulmón
G. Neumonía
H. Carcinoma Primario de Pulmón
I. Hipertensión Pulmonar
Primaria
J. Embolismo Pulmonar
K. Sarcoidosis
L. Apnea del Sueño
M. Tuberculosis
N. Pleuritis Viral
31.Un hombre de 59 años de edad ha tenido tos y esputo productivo con estrías de sangre
por 8 horas. Su temperatura 38.9°C, malestar, respiración corta, tos productiva de
esputo amarillo por 2 días. El ha fumado 2 paquetes diarios por 40 años. Roncus,
sibilancias y egofonía se escuchan en el hemotórax derecho. Una radiografía de tórax
muestra un infiltrado lobar denso en hemotórax derecho.
¿Cuál es el diagnóstico más probable? Respuesta G (Neumonía)

32.Un hombre de 25 años de edad ha tenido tos productiva con esputo con estrías de
sangre por 5 días. En los pasados 6 años, el ha tenido síntomas similares una o dos
veces por año, que han sido tratadas satisfactoriamente con antibióticoterapia. el ha
tenido tos por 12 años, recientemente productiva de una a dos tazas de esputo verde
amarillo diarias. No fuma, toma lipasa, proteasa y amilasa por mala absorción. Con
roncus bilaterales y sibilancias. Una radiografía muestra incrementote marcas
intersticiales bilaterales, lesiones nodulares e hiperinflación.
¿Cuál es el diagnóstico más probable? Respuesta E (Fibrosis Quística)

33.una mujer de 47 años de edad ha tenido tos productiva de escasa cantidad de esputo
amarillo, su temperatura de 38.3°C por 3 meses. En una ocasión, el esputo fue con
estrías de sangre. Ha perdido 21 lbs de peso, en los últimos 4 meses, ella no fuma. Es
inmigrante de Vietnam hace 20 años su registro médico no está disponible. Se ausculta
crépitos en lóbulo superior y roncus bilaterales. Una radiografía mostró infiltrados
múltiples, bilaterales y cavitaciones en lóbulo superior rodeados de infiltrados
pulmonares.
¿Cuál es el diagnóstico más probable? Respuesta N (Tuberculosis)

34.Un hombre de 50 años de edad con hipertensión es traído al cuarto de urgencias 30


minutos después del inicio súbito de un dolor torácico que se irradia a espalda y brazos.
Su presión sanguínea es 180/80 en su brazo izquierdo no se puede obtener PA en su
brazo derecho. Al examen cardíaco se ausculta un soplo de insuficiencia aórtica.
¿Cuál de los siguientes es el diagnóstico más probable?

A. Disección Aórtica Aguda


B. Infarto Agudo del Miocardio
C. Embolismo de la Arteria Subclavia Derecha
D. Embolismo Pulmonar
E. Neumotórax Espontáneo

35.Un hombre de 24 años de edad operador de un jackhammer, viene al médico por


historia de una semana de dolor, edema que afecta todo la extremidad superior
derecha; el dolor empeora con el ejercicio. Al examen se observa crecimiento de las
venas cutáneas sobre el tórax anterior.
¿Cuál de los siguientes es el diagnóstico más probable?

A. Trombosis de la Vena Axilar – Subclavia


B. Insuficiencia Valvular Venosa de la Extremidad
C. Tromboflebitis Superficial de la Mama
D. Tromboflebitis Superficial de la Vena Cefálica
E. Trombosis de Varicosidades Preexistentes de la Extremidad Superior.
36.Un infante de 2 meses de edad aparece bien hasta hace 3 semanas atrás cuando
empezó a tener disnea y dificultad para la alimentación. Se ausculta un bajo soplo
holosistólico en el borde esternal izquierdo bajo. El electrocardiograma muestra
hipertrofia ventricular izquierda.
¿Cuál de los siguientes es el diagnóstico más probable?

A. Bronquiolitis Aguda
B. Alergia a la Fórmula
C. Defecto Atrioseptal
D. Hipertensión Pulmonar
E. Defecto Septal Ventricular

37.Un niño de 3 años de edad es traído al médico por fiebre, tos y dificultad respiratoria por
2 semanas. Una radiografía de tórax muestra un infiltrado en lóbulo medio derecho y
una gran efusión pleural. La toracocentesis muestra un fluido purulento, el cultivo del
líquido crece Bacteroides melaninogenicus.
¿La infección es más probable una complicación de cuál de los siguientes?

A. Fibrosis Quística
B. Aspiración de Cuerpo Extraño
C. Defecto Inmunológico
D. Inhalación de un Hidrocarbono Tóxico
E. Apendicitis Subaguda

Cual de los siguientes esta más bajo en la sangre arterial umbilical de un feto sano que en
la sangre arterial de la madre?

a- Hematocrito
b- Afinidad del oxigeno
c- Capacidad del oxígeno
d- P02
e- Viscosidad

Tanto un paciente X como uno Y tienen frecuencia respiratoria de 12/min. Y un volumen


tidal de 500 ml. Sin embargo, el paciente X tiene una PC02 arterial de 35 mm Hg,
mientras el paciente Y tiene una PC02 arterial de 45 mm Hg. La diferencia en PC02 entre
estos pacientes esta mejor explicada por una diferencia en cual de los siguientes?
a- Gasto Cardiaco
b- Espacio muerto ventilatorio
c- FEV1
d- Capacidad residual funcional
e- Expansión pulmonar

Una paciente con un aneurisma del arco aórtico desarrolla ronquera. Parálisis de cual de los
siguientes músculos sobre el lado izquierdo es más probable?

a- Banda anterior del digástrico


b- Cricotiroideo
c- Omohioideo
d- Banda posterior del digástrico
e- Cricoaritenoideo posterior
Si la arteria axilar es ligada proximalmente a su paso sobre la primera costilla, la sangre
puede ser suplida a la arteria axilar distal a través de cual de las siguientes arterias?
a- Braquial
b- Torácica interna
c- Subclavia
d- Subescapular
e- Vertebral

Cual de las siguientes figuras (A-E) ilustra mejor le relación entre flujo sanguíneo (blood
flow) y presión arterial media (MAP) en un lecho vascular que demuestra auto regulación
del flujo sanguíneo?

Respuesta: d

Se realiza una cateterización cardiaca en una persona sana. La muestra sanguínea extraída
desde un catéter muestra 60% de saturación de oxígeno y la presión obtenida muestra
oscilación de un máximo de 26 mm Hg a un mínimo de 14 mm Hg. La punta del catéter fue
localizada en cual de las siguientes áreas?
a- Ductus arterioso
b- Foramen ovale
c- Atrio izquierdo
d- Arteria pulmonar
e- Atrio derecho

El procedimiento para medir el gasto cardiaco usando el principio de Fick involucra medir el
consumo de oxígeno por el pulmón y medir la diferencia de la tensión de oxígeno arterio-
venosa. Por la diferencia existente en el contenido de oxígeno de sangre emergiendo desde
diferentes órganos, cual de los siguientes es la mejor procedencia de sangre venosa para
esta medición?
a- Vena Yugular
b- Arteria pulmonar
c- Vena pulmonar
d- Vena Safena
e- Vena cava superior
Luego del ayuno, un paciente no exhibe secreción gástrica cuando mastica ni presenta
relajación proximal del estómago cuando deglute. Cuál de las siguientes explica mejor estos
hallazgos.

a. ausencia de células G
b. ausencia de células principales
c. concentración aumentada de secretina
d. concentración aumentada de somatostatina
e. estómago denervado

Varón de 75 años de edad con historia de constipación y dolores abdominales episódicos,


padece de diverticulosis. Esta paciente exhibe un riesgo aumentado de presentar cuál de las
siguientes condiciones?:

a. angiodisplasia
b. carcinoma de colon
c. hemorragia gastrointestinal
d. colitis granulomatosa
e. malabsorción

Niño de 6 años de edad presenta al examen físico una masa suprapúbica en la línea media.
Durante la cirugía se encuentra una masa quística dependiente del ombligo y que se ubica
en la base de la vejiga. Cuál de las siguientes condiciones es el diagnóstico más probable?:

a. hidrocele
b. divertículo de Meckel
c. quiste de Meckel
d. onfalocele
e. quiste del uraco

Al cabo de 10 minutos, luego de una biopsia hepática, un paciente exhibe un dolor severo
en el hombro derecho. Cuál de las siguientes vías nerviosas está involucrada en la
fisiopatología del dolor que tiene el paciente?:

a. axilar
b. intercostal
c. frénico derecho
d. cadena simpática derecha
e. vago derecho

Una mujer de 25 años, presenta una historia de tres días de vómitos y evacuaciones
líquidas. Presenta al examen físico tensión arterial baja y pobre turgor en tegumentos. El
sodio sérico es de 130 mEq/L. Cuál de los siguientes hallazgos es el más probable?:

a. concentración sérica disminuida de aldosterona


b. incremento en las concentraciones séricas del péptido natriurético atrial
c. incremento en el volumen circulante efectivo
d. incremento en la concentración sérica de vasopresina
e. orina con osmolalidad inferior a la sérica.
Un hombre de 70 kg con in ingreso diario fijo de 200 mmol/día, se le está inyectando
diariamente un potente mineralo corticoide por espacio de cuatro días. El tiene una ingesta
de agua apropiada, a discreción del paciente y no presenta restricción dietética alguna. La
excreción de sodio es como sigue:

Día NaCl (mmol)


01 30
02 90
03 180
04 200

si asumimos que un litro de NaCl al 0.9 % contiene 150 mmol de NaCl y pesa 1 kg, cuánto
pesará este paciente al finalizar el cuarto día?:
a. 66
b. 68
c. 70
d. 72
e. 74

Un recién nacido masculino (atendiendo a la impronta genética) ha desarrollado


completamente los conductos sexuales masculinos y trompas de Falopio. Cuál de los
siguientes procesos se vio probablemente alterado durante el período embrionario?:

a. producción de estrógeno por los testículos


b. producción del factor inhibidor de Muller por los testículos
c. producción testicular de testosterona
d. la respuesta de los conductos paramesonéfricos a los estrógenos.
e. Respuesta de los conductos paramesonéfricos a la testosterona.

Muestras séricas en una mujer sana con historia de ciclos menstruales de 28 días muestra
un pico sérico en la concentración de 17 β – estradiol en las pasadas 12 horas. No hay
progesterona sérica detectable. Dentro de los siguientes 3 días, cuál de los siguientes
eventos se espera que tenga lugar?:

a. cese de la menstruación
b. disminución de la temperatura basal
c. inicio de la menstruación
d. ovulación
e. regresión del cuerpo lúteo.

La lactancia no acontece durante la gestación debido a que la acción de la prolactina es


bloqueada por el incremento de cuál de las siguientes hormonas?:

a. estrógeno y progesterona
b. oxitocina y hormona del crecimiento
c. prostaglandinas y hormona coriónica del crecimiento
d. insulina y cortisol
e. tiroxina y dopamina
La gráfica muestra la osmolaridad orina/plsma versus el flujo urinario en pacientes
humanos sanos. Cuál de las siguientes situaciones puede producir un cambio de X a Y?:
a. ingestión oral de un litro de agua
b. ingesta oral de 200 cc de salina al 0.9 %
c. ingesta oral de 200 cc de solución de glucosa hipertónica
d. ingestión oral de 200 cc de salina al 3 %
e. inyección de ADH

Un infante de 3 meses de edad presenta una masa en el cuello. Al examinar el tejido de la


masa, se establece que se trata del timo. Atendiendo al origen embrionario, cuál de las
siguientes estructuras se encontrará en un sitio ectópico?:
a. nódulo linfático jugulo digástrico
b. amígdalas linguales
c. glándula paratiroides
d. glándula submandibular
e. glándula tiroides

El dominio intracelular del receptor insulínico cuenta con cual de las siguientes actividades
enzimáticas?:
a. adenilato ciclasa
b. fosfodiesterasa
c. fosfolipasa C
d. fosfoproteinfosfatasa
e. tirosina kinasa

Un adolescente de 10 años de edad presenta quemaduras corporales severas. La pérdida de


nitrógeno ocurre en los primeros días luego de la quemadura. Durante este período agudo,
cuál de los siguientes sustancias juega un papel importante en la pérdida del nitrógeno?:

a. cortisol
b. eritropoyetina
c. insulina
d. hormona paratiroidea
e. tiroxina.

91. Un jugador de básquetbol colegial de 22 años de edad acude al médico por una masa
escrotal izquierda; él se notó por primera vez dicha masa hace 2 semanas después de
golpearse con la bola en la ingle izquierda durante un juego. El exámen abdominal no
muestra anormalidades. Una masa no dolorosa de 3 cm es palpada en relación al polo
superior del testículo izquierdo; la prueba de transiluminación es negativa. ¿Cuál de los
siguientes es el diagnóstico más probable?.
a. Epididimitis.
b. Hematoma.
c. Hernia.
d. Hidrocele.
e. Tumor.
Respuesta correcta: E (Cáncer Testicular).

92. Un joven de 17 años de edad acude al médico por un dolor en la ingle del lado derecho
de 2 horas de evolución; la elevación del escroto no alivia el dolor. No hay historia de
trauma. Él es sexualmente activo y ha tenido múltiples parejas sexuales en los últimos 3
años. Él no usa condón regularmente. Él tuvo uretritis por Clamidia hace 1 año tratada con
Doxiciclina. El exámen muestra un escroto aumentado de volumen, edematoso,
eritematoso y doloroso. El testículo izquierdo está en posición horizontal; el reflejo
cremastérico está ausente del lado derecho. ¿Cuál de los siguientes es el diagnóstico más
probable?.
a. Epididimitos.
b. Tumor hemorrágico.
c. Hernia incarcerada.
d. Torsión testicular.
e. Torsión del apéndice testicular.
Respuesta correcta: D (Torsión testicular).

93. Un hombre de 70 años de edad acude al médico con una historia de 18 meses de
evolución caracterizada por un aumento en la frecuencia urinaria, vacilamiento en el inicio
de la micción y una disminución en el calibre del chorro urinario. El examen rectal revela
una próstata ligeramente aumentada de tamaño y firme. Después que él orina, se le pasa
una sonda Foley recolectándose 500mL de orina residual. El urianálisis está dentro de
límites normales. ¿Cuál de los siguientes es el diagnóstico más probable?.
a. Prostatitis aguda.
b. Hiperplasia prostática benigna.
c. Vejiga neurógena.
d. Cáncer de próstata.
e. Estrechez uretral.
Respuesta correcta: B (Hiperplasia Prostática Benigna).

94. Un joven de 16 años de edad es traído al Cuarto de Urgencias con historia de 1 hora de
evolución de dolor abdominal y escrotal de inicio súbito. Hay un dolor severo a la palpación
en la región del canal inguinal derecho; el escroto del lado derecho está vacío. El urianális
es normal. ¿Cuál de los siguientes es el manejo inicial más efectivo?.
a. Administración de analgésicos y observación.
b. Administración de hormonas gonadotróficas.
c. Inserción de una sonda nasogástrica.
d. Cistoscopía.
e. Operación inmediata.
Respuesta correcta: E (Torsión Testicular con Criptorquídea).

95. Un hombre de 60 años de edad, quien está hospitalizado, no orina desde hace 8 horas.
A él se le realizó un reemplazo total de cadera hace 36 horas. Él tiene una historia de
diabetes mellitas tipo 1 y toma nitratos por angina de pecho estable. Durante la cirugía, la
pérdida de sangre estimada fue de 1500mL. Intraoperatoriamente, él tuvo un episodio
corto de hipotensión que resolvió espontáneamente. Su temperatura es de 37.2º (99ºF),
tiene un pulso de 80 LPM, una FR de 16x’, y una presión sanguínea de 130/85 mmHg. Se
observa un aumento del volumen suprapúbico y una herida limpia en la cadera izquierda.
El exámen físico no muestra otras anormalidades. Su hemoglobina es de 9g/dL, los
glóbulos blancos están en 11000/mm3, y el conteo de plaquetas en 150000/mm3. La
glicemia capilar está en 190mg/dL. ¿Cuál de los siguientes es el paso inicial más apropiado
en el manejo de este paciente?.
a. Colocación de una sonda Foley.
b. Ultrasonido Renal.
c. CAT abdominal.
d. Transfusión de GRE.
e. Administración de 20 UI de insulina regular.
Respuesta correcta: A (Oliguria aguda).

96. Un hombre de 25 años de edad es traído al Cuarto de Urgencias después de estar en los
escombros debajo de un material de acero de un edificio que se derrumbó. Su pulso es de
140/min, la FR es de 22x’, y la presión sanguínea de de 80/60 mmHg. Él tiene lesiones
extensas por aplastamiento de los tejidos blandos en ambos muslos sin fracturas. No hay
otras lesiones obvias. Después de la administración IV de 3L de Lactato Ringer, su pulso
es de 110/min, la FR es de 20x’, y la presión sanguínea es de 100/70 mmHg. Con la
colocación de una sonda Foley se obtienen 30mL de una orina vino-oscura; la prueba en
orina por mioglobina es positiva. Además de la hidratación IV, ¿Cuál de los siguientes es el
próximo paso más apropiado en el manejo de este paciente?.
a. Administración IV de ácido hidroclórico hasta obtener un pH <4.
b. Administración IV de Manitol.
c. Infusión contínua de Dopamina.
d. CAT con contraste oral e IV.
e. Arteriografía.
Respuesta correcta: B (Mioglobinuria).

97. Una mujer de 54 años con una historia de 10 años de evolución de una Esclerosis
Sistémica progresiva (Esclerodermia) es llevada a una laparotomía de urgencias por una
apendicitis perforada con peritonitis. Durante el período postoperatorio inmediato, ella
tenía una presión sanguínea de 180/110mmHg. En los siguientes 3 días, su creatinina
sérica se incrementó, y su gasto urinario disminuyó a 250mL/día. Al cuarto día
postoperatorio, presentó disnea leve. Su saturación periférica de Oxígeno a aire ambiente
era de 89%. Los estudios en sangre mostraron un potasio en 6.2mEq/L, un BUN en
34mg/dL, y una creatinina en 3.9mg/dL. ¿Cuál de los siguientes es el próximo paso más
apropiado en el manejo de ésta paciente?.
a. Administración IV de IECA.
b. Administración IV de Morfina.
c. Lactato Ringer a chorro 2 L.
d. Hemodiálisis.
e. Diálisis Peritoneal.
Respuesta correcta: D (Falla Renal Aguda).

98. Una mujer de 26 años de edad G3, P2 acude al médico para su primer control prenatal
a las 36 semanas de embarazo. Ella ha estado tomando sulfato ferroso sin receta médica.
Aunque su ingesta de proteínas y calorías ha sido adecuada, ella come muy poco frutas o
vegetales frescos. Esta paciente está en riesgo incrementado de la siguiente deficiencia:
a. Calcio.
b. Ácido fólico.
c. Vitamina A.
d. Vitamina B12 (Cobalamina).
e. Vitamina D.
Respuesta correcta: B (Deficiencia de Ácido Fólico).

99. Una mujer de 39 años de edad G2, P1 tuvo un recién nacido de 3090g (6 Lb con 13
onzas) por parto vaginal espontáneo. El bebé tiene un puente nasal aplanado, pliegues
epicánticos prominentes, y un defecto septal ventricular. Su embarazo se complicó con
pielonefritis a las 16 semanas de gestación y por un resfriado a las 32 semanas. ¿Cuál de
los siguientes estudios prenatales es el más apropiado para diagnosticar la condición de
éste recién nacido?.
a. Amniocentesis para cariotipo.
b. Prueba de no estrés anteparto.
c. Títulos de Citomegalovirus.
d. Medición de alfa-fetoproteína sérica materna.
e. Títulos de Rubéola.
Respuesta correcta: A (Síndrome de Down).

100. Una mujer primigesta con 42 semanas de gestación da a luz un recién nacido de 4000
g (8Lb con 13 onzas) después de una inducción de la labor de parto con Oxitocina. El
estadío 1 y 2 de la labor de parto duró 14 y 3 horas, respectivamente. Se le realizó una
episiotomía media, y la placenta estaba completa. Diez minutos después del parto, ella
presentó un sangrado vaginal abundante estimado en 500mL en un período de 5 minutos.
El fondo uterino se palpa suave. ¿Cuál de las siguientes es la causa más probable de la
hemorragia?.

a. Laceración cervical.
b. Coagulación intravascular diseminada.
c. Tejido placentario retenido.
d. Atonía uterina.
e. Inversión uterina.
Respuesta correcta: D (Atonía uterina postparto).

101. Una mujer de 32 años de edad G5, P4 es admitida al hospital con un embarazo de 32
semanas de gestación por presentar sangrado transvaginal rojo rutilante ligero de 2 horas
de evolución. Ella no tiene dolor ni contracciones. Sus 2 últimos embarazos fueron
cesáreas. Los signos vitales son estables, y la FC fetal es de 140/min y es regular. Se
toman muestras de sangre para un hemograma completo, tipaje, Rh y cruce, y se
administran líquidos. ¿Cuál de los siguientes es el próximo paso más apropiado?.
a. Estudio Doppler del flujo sanguíneo umbilical.
b. Localización placentaria por exámen bimanual.
c. Localización placentaria por ultrasonido.
d. Amniocentesis para estudios de madurez pulmonar.
e. Cesárea inmediata.
Respuesta correcta: C (Placenta Previa).

102. Una mujer primigesta de 18 años de edad inició labor de parto espontáneamente
luego de una ruptura de membranas de 14 horas de evolución. La dilatación cervical es de
7cm. El monitoreo de la FC fetal muestra una FC basal de 180/min con una variabilidad
disminuída. Su temperatura es de 38.5º (101.3ºF), y su útero es doloroso a la palpación.
El frotis Gram del líquido amniótico muestra múltiples microorganismos. ¿Cuál de las
siguientes es la farmacoterapéutica más apropiada?.
a. Ampicilina y Gentamicina.
b. Ciprofloxacina y Clindamicina.
c. Eritromicina.
d. Metronidazol.
e. Penicilina.
Respuesta correcta: A (Corioamnionitis).

103. Una mujer primigesta de 20 años de edad con embarazo a término da a luz por parto
vaginal y anestesia epidural un recién nacido de 3200g (7 Lb con 1 onza). Se requirió de
una episiotomía media grado 2. Durante el primer día postparto, ella se queja de dolor
perineal. El periné está ligeramente edematoso sin evidencia de drenaje purulento o de
dehiscencia de la episiotomía. ¿Cuál de los siguientes es el próximo paso más apropiado en
el manejo de esta paciente?.
a. Baños de asiento 2 veces al día.
b. Injección local de lidocaína en el periné.
c. Tratamiento con Ampicilina IV.
d. Morfina IM.
e. Anestesia epidural.
Respuesta correcta: A ( Molestías normales postepisiotomía).

104. Una mujer primigesta de 24 años de edad con diabetes mellitas tipo 1 con un
embarazo de 38 semanas de gestación da a luz un recién nacido de 3856g (8Lb con 8
onzas). El embarazo se caracterizó por un pobre control de su diabetes. La labor de parto
duró 4 horas. El Apgar fue de 7/7 al primer y tercer minuto, respectivamente. ¿Cuál de las
siguientes es la prueba en sangre neonatal más apropiada en los primeros 30 minutos
luego del nacimiento?.
a. Determinación del grupo sanguíneo y el factor Rh.
b. Medición del hematocrito.
c. Medición del pH.
d. Medición de la bilirrubina sérica.
e. Medición de la glicemia.
Respuesta correcta: E (Recién nacido de madre diabética).
105. Un hombre de 65 años cuadripléjico como resultado de una esclerosis múltiple es
hospitalizado para el tratamiento de una neumonía del lóbulo inferior izquierdo. Su
temperatura es de 38.1º (100.5ºF), su pulso es de 95/min, su FR de 12x’, y la presión
sanguínea es de 120/80 mmHg. Él se ve malnutrido. Se auscultan roncus en el lóbulo
inferior izquierdo del pulmón. El exámen del corazón, ganglios, abdomen, y las
extremidades no demostró anormalidades. Hay un área eritematosa de 1 cm en el sacro
con piel intacta y sin induración. El exámen neurológico muestra cuadriparesia. La sangre
oculta en heces fue negativa. ¿Cuál de las siguientes es la intervención más efectiva para
la lesión en la piel de este paciente?.
a. Cambios frecuentes de posición.
b. Uso de vendajes húmedos y secos.
c. Terapia de remolino.
d. Terapia antibiótica de amplio espectro.
e. Debridamiento quirúrgico.
Respuesta correcta: A (Úlcera de decúbito).

106. Desde hace 1 semana, una joven de 17 años de edad ha tenido prurito y rash que se
han incrementado. Ella no tiene historia de problemas en la piel o síntomas asociados. Ella
no toma medicamentos. Su hermana con quien comparte el cuarto tuvo una condición
similar hace una semana. La temperatura de la paciente es de 36.8ºC (98.2ºF). Se
observan múltiples pápulas eritematosas en el tronco de 2-5mm, especialmente en los
pliegues, antebrazos, manos y dedos. No hay linfadenopatías o hepatoesplenomegalias.
¿Cuál de los siguientes es más probablemente el organismo causal?.
a. Virus Epstein-Barr.
b. Estreptococo del grupo A.
c. Virus del Sarampión.
d. Sarcoptes scabies.
e. Virus de la varicela-zoster.
Respuesta correcta: D (Escabiosis).

107. Un joven de 17 años de edad tiene un rash que involucra la cara, el cuello, el tórax
superior, y las extremidades superiores e inferiores de 5 días de evolución. Durante un
viaje reciente a México, él estuvo buceando encontrándose con muchos peces “gelatinosos”
(Jellyfish). Él también tuvo diarrea tratada con trimetoprim-sulfametoxazol. Su
temperatura es de 37.8ºC (100ºF). El exámen físico demuestra un rash concluyente,
eritematoso y pruriginoso; no se observan costras ni vesículas. El exámen es por lo demás
irrelevante. ¿Cuál de las siguientes es la causa más probable de esta condición?.
a. Eczema.
b. Fotosensibilidad.
c. Reacción alérgica al “Jellyfish”.
d. Salmonella Typhi.
e. Escherichia colli toxicogénica.
Respuesta correcta: B (Fotosensibilidad inducida por el TMP-SMX).

108. Un hombre de 48 años de edad conocido alcohólico acude al médico por fiebre, rash
facial, y edema rápidamente progresivo de la hemicara izquierda. El edema comenzó hace
12 horas luego que una costra en su barbilla izquierda le comenzó a picarse. Su
temperatura es de 39.2ºC (102.5ºF). Él es incapaz de abrir su ojo izquierdo debido al gran
edema. ¿Cuál de los siguientes es el organismo causal más probable?.
a. Estreptococo del grupo A.
b. Haemophilus influenzae.
c. Virus Herpes simples.
d. Neisseria meningitidis.
e. Streptococcus pneumoniae.
Respuesta correcta: A (Infección por estreptococo del grupo A).

109. Un niño de 7 años de edad es llevado al médico 3 días después de hacerse una
laceración pequeña en la ceja izquierda. Su temperatura es de 38.8ºC (101.8ºF). El
exámen físico muestra edema y eritema de la región periorbitaria y del párpado izquierdos
con proptosis moderada y movimientos oculares disminuídos y dolorosos. Los márgenes
del disco óptico son claros, y no hay anormalidades retinianas. ¿Cuál de los siguientes es el
diagnóstico más probable?.
a. Trombosis del Seno Cavernoso.
b. Trombosis del Seno Lateral.
c. Celulitis orbitaria.
d. Celulitis periorbitaria (Preseptal).
e. Trombosis del Seno Sagital.
Respuesta correcta: C (Celulitis Orbitaria).

110. Un hombre de 56 años de edad presenta un rash vesicular doloroso con formación de
costras de 2 días de evolución. Él recibió quimioterapia por un Linfoma No-Hodgkin hace 1
año. Su temperatura es de 36.7ºC, su pulso es de 80/min, y su presión sanguínea es de
138/76mmHg. El exámen físico por lo demás es irrelevante. ¿Cuál de los siguientes es el
diagnóstico más probable?.
a. Herpes zoster.
b. Impétigo.
c. Pioderma gangrenoso.
d. Sífilis.
e. Lupus eritematosos sistémico.
Respuesta correcta: A (Herpes Zoster).

111. a. Dermatitis por contacto alérgica.


b. Dermatitis atópica.
c. Eczema dishidrótico.
d. Dermatitis por contacto irritante.
e. Liquen simple crónico (Neurodermatitis localizada).
f. Eczema numular.
g. Pitiriasis rosada.
h. Psoriasis.
i. Escabiasis.
j. Tiña corporis.

Para cada paciente con una condición de la piel pruriginosa, seleccione el diagnóstico más
probable.
Una mujer de 30 años de edad acude al médico por un rash caracterizado por costras
eritematoescamosas y pruriginosas de 1 año de evolución; el rash es más severo en los
codos y rodillas. El rash ocurre en los meses de invierno y se alivia moderadamente con
emolientes. El exámen físico muestra placas ovales discretas de 4-6 cm de diámetro
sobre las rodillas y codos; ellas tienen una base eritematosa con una escama plateada.
Respuesta correcta: H (Psoriasis).

Una mujer de 23 años de edad acude al médico porque presenta manchas pálidas en su
espalda desde hace 2 meses. Ella primero notó la aparición de una placa ovala de 3-4 cm
en su espalda superior, seguida por la aparición de manchas más pequeñas con escozor
ocasional. Ella ha utilizado algunos medicamentos tópicos sin éxito. El exámen muestra
máculas múltiples de 3-5mm en su espalda con una distribución en “arbolito de navidad”;
las máculas son más pálidas que la piel circundante.
Respuesta correcta: G (Pitiriasis rosada).

112. Un hombre de 67 años de edad acude al médico por una lesión no dolorosa en la
porción superior de su frente que ha sangrado recientemente. Él piensa que la lesión está
allí desde que el se golpeó en la cabeza con una botella hace 18 meses. El exámen físico
muestra áreas eritematosas en el resto de su frente. La lesión es de 1cm de diámetro con
un borde definido oscuro y un centro ulcerado más claro. ¿Cuál de los siguientes es el
diagnóstico más claro?.
a. Keratosis actínica.
b. Epiteliona de células basales.
c. Sarcoma de Kaposi.
d. Melanoma.
e. Keratosis seborreica.
Respuesta correcta: B (Epitelioma de células basales).

113. Una mujer afroamericana de 30 años de edad previamente sana acude al médico por
fatiga, artralgias, y un rash nodular sobre el tronco y las extremidades superiores de 3
semanas de evolución. Se observan 12 placas nodulares induradas de 0.3-0.8cm, pálidas
en el tórax, espalda, y en las extremidades superiores. El hígado es palpable 2 cm debajo
del reborde costal derecho con una zona de matidez hepática de 14 cm, y el polo inferior
del baso es palpable a 3cm por debajo del reborde costal izquierdo. No hay dolor o
limitación en la movilidad de las articulaciones. El resto del exámen muestra hallazgos
normales. Un Rx de tórax demuestra adenopatías hiliares bilaterales. Una biopsia de las
lesiones en piel, ¿Qué es lo más probable que demuestre?.
a. Infiltración dérmica con monolitos y células de Reed-Sternberg.
b. Histiocitos cargados de grasa.
c. Granulomas no caseosos.
d. Infiltrado eosinofílico homogéneo paravascular.
e. Vasculitis con células gigantes.
Respuesta correcta: C (Sarcoidosis).

114. Una mujer de 43 años de edad que trabaja como costurera tiene una historia de 3
meses de dolor, debilidad, y adormecimiento de la mano derecha que empeora en la
noche. El exámen físico muestra hipoestesia y atrofia de la eminencia tenar. Estos
hallazgos son más probablemente causados por compresión de ¿cuál de los siguientes
nervios?.
a. Axilar.
b. Cutáneo braquial.
c. Mediano.
d. Radial.
e. Ulnar.
Respuesta correcta: C (Síndrome del Túnel Carpal).

115. Una mujer de 50 años de edad acude al médico por una historia de 4 semanas de
evolución de dolor en las extremidades, dificultad en subir escaleras, y dificultad para
ponerse de pie. El exámen demuestra dolor a la palpación de los músculos cuadríceps.
Hay decoloración rojo-púrpura de la piel en la frente, párpados y mejillas, así como
nódulos purpúreos en los codos y rodillas. ¿Cuál de los siguientes es el diagnóstico más
probable?.
a. Dermatomiositis.
b. Enfermedad mixta del tejido conectivo.
c. Psoriasis.
d. Artritis Reumatoidea.
e. Lupus eritematoso sistémico.
Respuesta correcta: A (Dermatomiositis).

116. Una mujer de 60 años de edad viene al médico por dolor severo y rigidez de su
cuello, hombros, y caderas de 2 meses de evolución. Sus síntomas son más pronunciados
en la mañana inmediatamente al despertarse. Ella ha tenido fatiga crónica y fiebre de
bajo grado durtante este período. El rango del movimiento del cuello, hombros y caderas
es normal. Los músculos están mínimamente dolorosos a la palpación. La fuerza
muscular, sensación y los reflejos tendinosos profundos son normales. La CPK es de 40
U/L y la velocidad de eritrosedimentación está en 80mm/h. El factor reumatoide y el ANA
son negativos. ¿Cuál de los siguientes es el diagnóstico más probable?.
a. Fibromiositis.
b. Osteoartritis.
c. Polimialgia reumática.
d. Polimiositis.
e. Artritis reumatoidea seronegativa.
Respuesta correcta: Polimialgia Reumática.

117. Una joven de 16 años de edad es traída al médico debido a dolor intermitente y
edema de ambos tobillos el mes pasado. Ella actualmente no tiene dolor. Cuando el
dolor ocurre, éste es tan severo que ella es incapaz de caminar. No hay fiebre ni
escalofríos asociados. Ella es sexualmente activa y ha tenido sólo un compañero sexual
en el último año. Su temperatura es de 37ºC (98.6ºF), su pulso es de 80/min, y la presión
sanguínea es de 145/87mmHg. El exámen no muestra anormalidades o dolor a la
palpación de las articulaciones de los tobillos. Hay una úlcera no dolorosa en la mucosa
oral. Los pulmones son claros a la auscultación. El exámen cardíaco no muestra
anormalidades. Los estudios de laboratorios demuestran:
GB 4000/mm3.
Neutrófilos segmentados 65%
Eosinófilos 3%
Linfocitos 25%
Monocitos 7%
Plaquetas 60000/mm3
VES 100mm/h.
ANA 1/320
AntiDNA positivo.
RPR (Reagina plasmática rápida) 1/16.
Factor reumatoide negativo.
Proteínas en orina 3+
Cilindros de GR negativo.
GR ninguno
GB 10-20/campo de alto poder.

Los Rx de los tobillos no muestran anormalidades más que edema de tejidos blandos.
¿Cuál de los siguientes es el diagnóstico más probable?.
a. Enfermedad gonocóccica diseminada.
b. Artritis poliarticular.
c. Artritis reactiva.
d. Sífilis secundaria.
e. Lupus eritematoso sistémico.
Respuesta correcta: E (Lupus eritematoso sistémico).
118. Un hombre de 70 años acude al médico debido a mialgias severas y malestar general
en los últimos 2 meses. Él ha perdido 2.7Kg (6Lb) durante este período. El exámen físico
no muestra anormalidades. Su hematocrito es del 30%, el conteo de GB es de
11300/mm3, y la VES es de 112mm/h. La CPK está dentro de límites normales. ¿Cuál de
los siguientes es el diagnóstico más probable?.

a. Miopatía hipertiroidea.
b. Miastenia gravis.
c. Polimialgia reumática.
d. Artritis reumatoide.
e. Miopatía seronegativa.
Respuesta correcta: C (Polimialgia Reumática).

119. Un niño de 4 años con fractura supracondílea desplazada del húmero sin
complicación neurovascular es colocado en tracción esquelética. Seis horas más tarde, él
tiene un dolor severo en el antebrazo que se incrementa con la extensión pasiva de la
muñeca y los dedos. ¿Cuál de los siguientes es el próximo paso más apropiado en el
manejo de éste niño?.

a. Incrementar el peso en el aparato de tracción.


b. Administración de medicamentos analgésicos.
c. Exploración de la fractura y fasciotomía del compartimento flexor del antebrazo.
d. Reducción cerrada con el paciente bajo anestesia.
e. Reducción abierta y fijación interna de la fractura.
Respuesta correcta: C (Síndrome compartamental).

120. Una adolescente de 15 años de edad es traída al médico por irritabilidad,


intranquilidad, dificultad para concentrarse y deterioro en la ejecución académica en los
últimos 2 meses. Ella ha tenido ocasionalmente palpitaciones y ha perdido 1Kg (2Lb) de
peso durante este período a pesar de un apetito incrementado. Menarquia ocurrió a los
12 años, y sus menstruaciones ocurren a intérvalos regulares. Ella no usa drogas y no ha
iniciado relaciones sexuales. Sus padres refieren que ella no tiene problemas en la casa.
El exámen no muestra anormalidades. La T3 y T4 séricas están aumentadas. ¿Cuál de los
siguientes es el diagnóstico más probable?.
a. Desorden tipo déficit de atención-hiperactividad.
b. Diabetes mellitus.
c. Hipertiroidismo.
d. Adenoma Pituitario.
e. Cáncer de Tiroides.
Respuesta correcta: C (Hipertiroidismo).
121. Un hombre de 49 años de edad es llevado al Cuarto de Urgencias por una historia de
2 días de evolución caracterizada por poliuria, polidipsia, naúseas, y confusión
incrementada. A él se le diagnosticó un carcinoma escamoso de pulmón hace 8 semanas.
Él está letárgico, orientado en persona pero desorientado en tiempo y lugar. Un EKG
muestra un intérvalo QT corto. Además de la terapia con líquidos IV, cuál es el manejo
próximo más apropiado en este paciente?.
a. Bifosfonato IV.
b. Corticoides IV.
c. Manitos IV.
d. Mitramicina IV.
e. Hidroclorotiazida oral.
Respuesta correcta: A (Hipercalcemia).

Das könnte Ihnen auch gefallen